You are on page 1of 167

DAFTAR ISI

Halaman
AcEPT (Academic English Proficiency Test) ............................................................................... 1
Vocabulary Practice ........................................................................................................................ 1
Grammar and Structure Practice ............................................................................................. 37
Reading Comprehension Practice ........................................................................................... 67
Composing Skills Practice ........................................................................................................... 86

GMST (Gajah Mada Scholastic Test) ............................................................................................... 100


Numeric, Verbal, Non Verbal, Reasoning Practice .......................................................... 100
Quantitative Practice....................................................................................................................... 136

vi
ACEPT
Latihan Vocabulary
SECTION 1
Time – 30 minutes
35 Questions

Directions: For each question in this section, select the best answer from among the choices given
and fill in the corresponding oval on the answer sheet.

Each sentence below has one or two blanks, 3. As a physicist, Veronica is a gifted ------; she
each blank indicating that something has loves to go beyond particular facts and
been omitted. Beneath the sentence are five speculate about general principles.
words or sets or words labeled A through E. A. Dogmatist
Choose the word or set of words that, when B. Consultant
inserted in the sentence, best fits the meaning C. prodigy
of the sentence as a whole. D. materialist
E. theorist
1. In the Renaissance, when few women were 4. Although hostile demonstrations and -------
formally educated and most were forced to marred James Meredith's 1962 enrollment
marry, he rebellious Cecilia Gonzaga at the University of Mississippi, the
succeeded in ------- scholarship and the commencement ceremony in which he
marriage planned for her. became the university's first African
1. obtaining . . succumbing to American graduate was surprisingly -------.
2. escaping. . subverting A. discord . . tranquil
3. pursuing . . avoiding B. pomp . . daunting
4. ignoring . . observing C. banality . . conventional
5. disavowing. . enjoying D. turmoil . . controversial
2. During the day, downpours were ------, E. serenity. . opportune
starting and stopping at nearly regular 5. The editor's comment was not intended as
intervals. a criticism, but as a ------- by which she
1. Unmediated sought further clarification.
2. Spontaneous A. query
3. pediodic B. confession
4. incidental C. dismissal
5. endemic D. condemnation

1
E. credo 10. The ------- of the art world, its "apparent
6. Although Clifton often appeared ------, he inviolability," was sullied in 1997 when
actually devoted ------ amount of time investigators uncovered several dubious
trying to keep up a neat appearance. art transactions.
A. orderly . . an enormous A. turpitude
B. disheveled . . an inordinate B. sacrosanctity
C. annoyed . . an unfortunate C. perspicuity
D. distracted . . an unrealistic D. verisimilitude
E. agitated . . a considerable E. duplicity
7. In 1991 salsa ------ ketchup as the best-
selling condiment in the United States, Each question below consists of a related pair
outselling ketchup by $40 million in retail of words or phrases, followed by five pairs of
stores. words or phrases labeled A through E. Select
A. Supplanted the pair that best expresses a relationship
B. redoubled similar to that expressed in the original pair.
C. augmented
D. brandished 11. COAL : MINERAL ::
E. evaded A. oxygen : water
8. The Earth’s oceans sustain a ------ of marine B. river : dam
creatures, an abundance that makes the C. gold : metal
seas teem with life and activity. D. silver : mine
A. Melee E. laboratory : chemical
B. Profusion 12. TRAIL : HIKERS ::
C. configuration A. airport : pilots
D. symmetry B. pasture : horses
E. dimension C. highway : motorists
9. The gentle flow of the speaker's words D. forest : animals
became increasingly balanced and E. track : footprints
rhythmic; such ------ oratory was quite 13. COMPETITION : CONTESTANT ::
hypnotic. A. trial : witness
A. Cadent B. journey : traveler
B. specious C. royalty : monarch
C. convoluted D. cures : patient
D. adulatory E. election : candidate
E. impassioned 14. GONG : MALLET ::
A. guitar : string

2
B. orchestral : baton B. letter : address
C. clarinet : reed C. contract : agree
D. maraca : rattle D. inscription : write
E. drum : stick E. invoice : pay
15. ARCHIVE: DOCUMENTS :: 21. DISINTERESTED : FAVORITISM ::
A. warehouse : merchandise A. urbane : civility
B. theater : plays B. modest : reserve
C. cinema : projector C. adversarial : cooperativeness
D. library : shelves D. dilatory : procrastination
E. farm : crops E. dissipated : pleasure
16. RIDDLE : PUZZLEMENT :: 22. WHEEDLE : CAJOLERY ::
A. comedy : stage A. deseive : subterfuge
B. clown : costume B. distribute : parity
C. quip: wit C. delight : mimicry
D. jest : laughter D. alienate : cohesion
E. pun : meaning E. dissemble : demeanor
17. INVENTORY : GOODS :: 23. REMISS : DUTIFULNESS ::
A. agenda : meeting A. redoubtable: awe
B. snapshot : image B. careful : compulsion
C. ballot : voters C. hysterical : calamity
D. compass : directions D. intemperate : moderation
E. roll : members E. diplomatic : tact
18. INNUMERABLE : QUANTITY ::
A. superficial : surface
B. impotent : strength
C. invaluable : worth
D. finite : size
E. inexpressive : feeling
19. REHASH : DISCUSS ::
17 reprimand : scold
18 reject : want
19 rejoice : praise
20 reiterate : state
21 relish : taste
20. EPITAPH : COMMEMORATE ::
A. badge: identify

3
The passage below is followed by questions based on its content. Answer the questions on the basis
of what is stated or implied in the passage and in any introductory material that may be provided.

Questions 24-35 are based on the following see a martial eagle, they move into thick
passage. 30 vegetation close to a tree trunk or at ground
level. Thus the tactics that help escape from a
This discussion of vervet monkeys is from a 1984
leopard make them highly vulnerable to a
book about animal coimmunication.
martial eagle, and vice versa. In response to the

Vervet monkeys have at least three threat of a large snake, they stand on their hind

different categories of alarm calls. When a 35 legs and look around to locate the snake, then

leopard or other large carnivorous mammal simply move away from it, either along the

approaches, the monkeys give one type of alarn ground or by climbing into a tree.
Line
Knowing that the monkeys give
5 call; quite a different call is used at the sight of a
martial eagle, one of the few flying predators different alarm calls when they see different

that captures vervet monkeys. A third type of 40 predators does not establish beyond a doubt

alarm call is given when a large snake that the calls actually describe the type of

approaches the group. This degree of predator. When the monkeys, which are usually

10 differentiation of alarm calls is not unique, close to each other, hear an alarm call, each one

although it has been described in only a few quickly looks around at the caller. Like many

kinds of animals. When ethologists, who study 45 other animals, they are adept at judging the

animal behavior, interpret data of this kind, direction in which another animal is looking, so

they require proof that variations in animal they can easily see what the caller is looking at.

15 communication signals convey anything more This serves much the same function as pointing.

than information about the communicator's When monkeys other than the caller take the

internal state. 50 appropriate action to avoid the danger, it is

The first and relatively simple question difficult to be sure whether they are acting

is whether the vervet monkey’s three types of solely on the basis of the call or whether the call

20 alarm calls convey to other monkeys simply led them to look at the source of the

information about the type of predator. Such danger.

information is important, because the animal's 55 To clarify this situation, researchers

defensive tactics are different in the three cases. conducted some carefully controlled playback

When a leopard approaches, the monkeys climb experiments under natural conditions. The

25 into trees. But leopards are good climbers, so basic idea was to play from a concealed

the monkeys can escape then only by climbing loudspeaker tape recordings of vervet alarm

out into the smallest branches, which are too 60 calls when vervets had just seen a leopard, a

weak to support a leopard. When the monkeys martial eagle, or a large python, and to inquire

4
whether these playbacks, in the absence of a interpretation necessarily ascribes three
predator, would elicit the normal response. The specific types of injunction to the vocabulary of
experiments required many precautions and vervet monkeys. Even such postulated
refinements. For instance, vervet monkeys 100 injunctions would be more than a simple
65 come to know each other as individuals, not reflection of the internal state of the
only by visual appearance but by minor communicator.
differences in their vocalizations. They might
24. The passage indicates that the calls
not respond even to an alarm call recorded
described in lines 1-9 are significant
from one of their own companions if that
primarily because they
70 individual was in plain sight some distance
A. show that animals are capable of
from the vegetation concealing the speaker. In
expressing emotion
all experiments, the loudspeaker reproduced
B. prove that some animals are more
calls of a member of the group. and the speaker
intelligent than others
was hidden in a place where the monkeys
C. noticeably improve the monkeys’ rate
75 would expect that individual to be. The
of reproduction
experiments had to be prepared with tape
D. represent a departure from the
recordings of a known member of a well-
monkeys' predictable patterns of
studied group or vervet monkeys and a hidden
communication
speaker located where this individual
E. prompt questions about the potential
80 frequently spends time.
extent of animal communication
When all these conditions wcre
25. ln lines 12-17 (“When . . . state”), the
satisfied, the playbacks of alarm calls did indeed
author's observation about ethologists
elicit the appropriate responses. The monkeys
implies that they
responded to the leopard alarm call by climbing
A. are dismissive of issues that concern
85 into the nearest tree; the martial eagle alarm
other biologists
caused them to dive into thick vegetation; and
B. limit themselves by their reliance on
the python alarm produced the typical behavior
traditional explanations of animal
of standing on the hind legs and looking all
behavior
around for the nonexistent snake.
C. fail to account for discrepancies
90 Not all ethologists have accepted the
between field and laboratory
straightforward interpretation that the alarm
observations
calls convey information about the type of
D. try to avoid unjustified conclusions
predator. One alternative interpretation is that
about the meaning of a phenomenon
the alarm calls are injunctions to behave in
E. use an approach that sometimes
95 certain ways. Thus the leopard alarm might
arouses resentment
mean "Go climb into a tree." But even this

5
26. What is the relationship between the first B. showing the necessity of multiple
paragraph (lines 1-17) and the “simple explanations
question” mentioned in lines 18-21? C. describing an alternate method of
A. The first paragraph contains evidence observation
that will answer the question. D. supporting a hypothesis with
B. The question arises from information observations from the field
in the first paragraph. E. drawing an analogy between animal
C. The question makes light of the view and human behavior
presented in the first paragraph. 29. When designing the experiments described
D. The first paragraph outlines the way in lines 55-80, researchers had to consider
the question will be answered in the all of the following EXCEPT
rest of the passage. A. the location of certain monkeys in the
E. The question defines an unorthodox group
view that was discounted in the first B. the monkeys’ familiarity with one
paragraph. another
27. In lines 24-31 (“When a leopard . . . level”), C. the location of the equipment
the author juxtaposes two kinds of D. the vocalization of predators
behavior in order to E. individual differences among the
A. show how the presence of more than monkeys’ calls
one observer in the field yields 30. According to lines 55-80, which action
conflicting information would likely keep the monkeys from
B. provide evidence that challenges an responding to the recorded calls?
accepted theory about monkey A. Locating the loudspeaker far from
communication where the individual whose voice it
C. compare a unique form of defense to a broadcasts can be seen
more common form of defense B. Playing the calls during feeding or
D. explain how the monkeys imitate grooming periods
behavior of other animals C. Playing the calls so often that the
E. emphasize the usefulness of different monkeys become accustomed to them
responses in different situations and fail to react
28. The third paragraph (lines 38-54) D. Allowing the monkeys to detect the
contributes to the development of the presence of the human observers
passage primarily by E. Interfering with the hunting routines of
A. indicating an interpretation that is the usual predators
eventually ruled out 31. In line 82, “satisfied” most nearly means
A. convinced

6
B. dispelled D. receptive, because it is consistent with
C. fulfilled the data
D. appeased E. respectful, because it is shared by many
E. compensated experienced field researchers
32. The experiments described in the passage 34. The final paragraph primarily serves to
provide evidence that most directly A. show how an objection to a hypothesis
supports the conclusion that vervet actually confirms one of its central
monkeys elements
A. are highly adaptable to changing B. introduce a personal interpretation of
environmental conditions the findings
B. respond to the presence of predators C. suggest that responses to alarm calls
with calls particular to each danger are genetically determined
C. tolerate individuals who do not pose an D. cast doubt on the importance of a field
immediate threat of inquiry
D. protect themselves by mimicking the E. indicate the kinds of questions that are
calls of certain predators not susceptible to further study
E. illustrate the ability of most mammals 35. The author uses vervet monkeys to convey
to communicate information which point about animal communication?
33. The author’s reaction to an “alternative A. Animal vocalizations are modeled after
interpretation” (line 93) is best human sounds.
characterized as B. Some animals can impart vocally
A. offended, because it disregards the specific information about their
author’s own observations observations.
B. skeptical, because it perpetrates the C. Most animals respond differently to
falsehood that monkeys possess human different alarm calls.
traits D. Animals vocalize primarily to
C. supportive, because it provides proof communicate an internal state.
for a hypothesis E. Most animals exhibit an acute sense of
hearing when sensing predators.

STOP
If you finish before time is called, you may check your work on this section only.
Do not turn to any other section in the test.

7
SECTION 4
Time - 30 minutes
30 Questions

Directions: For each question in this section, select the best answer from among the choices given
and fill in the corresponding oval on the answer sheet.

Each sentence below has one or two blanks, an insult to the standards of the judicial
each blank indicating that something has system.
been omitted. Beneath the sentence are five 1. apathetic . . victory
words or sets of words labeled A through E. 2. exacting . . spectacle
Choose the word or set of words that, when 3. astute . . debacle
inserted in the sentence, best fits the meaning 4. negligent . . travesty
of the sentence as a whole. 5. surreptitious . . triumph
4. Constance was -------- by the speech,
1. Despite the wide-ranging curiosity about
regarding such criticisms of her company
her personal life, Eleanor Roosevelt
as extremely annoying.
enjoyed a degree of ------- that today’s
A. fascinated
highly scrutinized public figures can only --
B. galled
-----.
C. uplifted
A. privacy . . envy
D. soothed
B. popularity . . celebrate
E. disoriented
C. privilege . . imitate
5. Steven tried hard to give up sweets, but he
D. isolation . . regret
found it particularly difficult to -------
E. generosity . . refuse
chocolate.
2. Unable to decide between a career in
A. Digest
biology and one in philosophy, Gwen -------
B. extol
her two interests and became a medical
C. impugn
ethicist.
D. forgo
A. reclaimed
E. relish
B. merged
6. At first merely -------, his actions grew so
C. defined
bewildering and bizarre as to appear
D. abandoned
entirely ------- to us.
E. conveyed
A. dignified . . mystifying
3. The incompetent judge conducted the
B. perplexing . . inexplicable
hearing in so ------- a manner that the
C. eccentric . . stolid
entire proceeding was considered a -------,
D. intriguing . . reasonable

8
E. logical . . questionable Each question below consists of a related pair
7. Rather than focusing on the ------ sequence of words or phrases, followed by five pairs of
of events, the historian E.M.W. Tillyard ----- words or phrases labeled A through E. Select
a chronological approach and portrays, the pair that best expresses a relationship
instead, the dominant belief patterns of an similar to that expressed in the original pair.
age.
10. COOKBOOK : RECIPES ::
A. rational . . acknowledges
A. film : reviews
B. temporal . . avoids
B. manual : instructions
C. universal . . embraces
C. magazine : subscriptions
D. qualitative . . employs
D. paperback : editions
E. unseen . . forsakes
E. museum : tours
8. The fashion designer’s new line of spring
11. STONE : SCULPTURE ::
clothing was described in the style section
A. mural : painting
of the newspaper as -------, even -------; the
B. opera : stage
runway collection had dazzled the
C. canvas : easel
audience.
D. clay : pottery
A. unassuming . . audacious
E. plot : novel
B. capricious . . innocuous
12. LIBERTARIAN : CENSORSHIP ::
C. tawdry . . precocious
A. merchant : profit
D. vivacious . . insipid
B. legislator : order
E. resplendent . . incandescent
C. vegetarian : health
9. Robb Armstrong’s Jump Start fills a void in
D. anarchist : confusion
the cartoon industry, namely, a ------- of
E. pacifist : violence
comic strips representing African
13. BRINE : PRESERVATIVE ::
Americans.
A. grass : fertilizer
A. spate
B. foam : detergent
B. revision
C. fuse : explosive
C. dearth
D. grease : lubricant
D. dispersal
E. germ : antiseptic
E. consensus
14. OSTENTATION : SIMPLICITY ::
A. stereotype : prejudice
B. hypocrisy : sincerity
C. hypothesis : experiment
D. illusion : allure
E. fallacy : appearance

9
15. ARBOREAL : TREE :: C. humid : desert
A. edible : fruit D. granular : sand
B. aquatic : water E. carnivorous : animal

Each passage below is followed by questions based on its content. Answer the questions on the basis
of what is stated or implied in each passage and in any introductory material that may be provided.

Questions 16-24 are based on the following away. Or they may have secretly clung so
passage. 25 closely to treasured objects that they never
passed them on. Then these objects were lost.
In this passage, the narrator considers his
My uncle Lico ferreted out the past as a
family’s history and migration from Mexico to
passionate genealogist who used research,
Texas, which was once part of Mexico.
fantasy, and spells of breathless madness to
I never understood people’s fascination 30 craft his ancestral charts of the branches of our
with immortality. The idea of life without end family. Some are elaborate discs, in which each
gave me chills. Even as a kid, I wanted to be outward concentric ring represents a new
among my family and my ancestors, walking generation. In others, quickly dashed off as
Line
5 through our short time together. I wanted to notes to himself, ragged trees and jagged lines
bind Texas and Mexico together like a raft 35 are drawn between names like Evaristo,
strong enough to float out onto the ocean of Viviano, Blas, and Hermenegilda. In one, going
time, with our past trailing in the wake behind back to 1763, the capstone slot contains the
us like a comet tail of memories. cryptic entry “King of Spain,” from whom,

10 But the past can be difficult to conjure presumably, he believed we were descended.
again when so little has been left behind. Some 40 Subtle faculties and proclivities were passed,
families in Mexico have troves of their speechlessly, through the flesh of successive
ancestors’ belongings, from pottery of the generations. The ghosts of Spanish royalty
ancients and paintings of Mexico City in the mingled with Indians, Black people, and others
15 eighteenth century to helmets and shields of the from every part of the world in Uncle Lico’s
Spaniards. By comparison my family, the 45 secret genealogy. Yet, despite the ridicule of
Santos, are traveling light through time. many, he managed to recover numerous names
Virtually nothing has been handed down, not and stories. Lico knew I had some of the same
because there was nothing to give, but after magnetic attraction to the past that fueled his
20 leaving Mexico to come to Texas—so many manic genealogies, as if the molecules of our
loved ones left behind, cherished places and 50 bodies were polarized in a way that drew us
things abandoned—they ceased to regard both back in time, back, inexorably, toward the
anything as a keepsake. Everything was given ancestors.

10
In my dreams, the ancestors who have C. consideration of leaving Texas and
passed on visit with me in this world. They ask returning to Mexico
55 me questions they were once asked: Where did D. belief that Texas and Mexico are more
our forbears come from and what have we similar than not
amounted to in this world? Where have we E. awareness that he is neither a Texan
come to in the span of time, and where are we nor a Mexican
headed, like an arrow shot long ago into an 17. The objects mentioned in lines 13-16
60 infinite empty space? What messages and (“from pottery . . . Spaniards”) are
markings of the ancient past do we carry in examples of
these handed-down bodies we live in today? A. artifacts discovered by Uncle Lico
With these questions swirling inside me, B. possessions viewed as impediments to
I have rediscovered some stories of the family a simple life
65 past in the landscapes of Texas and Mexico, in C. gifts bestowed on departing loved ones
the timeless language of stone, river, wind, and D. necessities valued by earlier
trees. My great-uncle Abrán was a master of generations
making charcoal. He lived in the Texas hill E. items bearing both cultural and
country, where the cedars needed to make personal meaning
70 charcoal were planted a century ago. Today, 18. In line 17, “light” most nearly means
long after he worked there, walking in that A. unencumbered
central Texas landscape crowded with deep B. illuminated
cedar, I feel old Abrán’s presence, like the C. nimbly
whisper of a tale still waiting to be told, D. faintly
75 wondering whether my intuition and the E. gently
family’s history are implicitly intertwined. Even 19. The primary effect of lines 27-45 (“My
if everything else had been lost—photographs, uncle . . . genealogy”) is to depict the
stories, rumors, and suspicions—if nothing at A. collaboration between the narrator and
all from the past remained for us, the land his uncle
80 remains, as the original book of the family. It B. influence of the uncle on the narrator’s
was always meant to be handed down. generation
C. unorthodox nature of Uncle Lico’s
16. The image of the “raft” (line 6) most clearly
methodology
conveys the narrator’s childhood
D. family’s enthusiasm for Uncle Lico’s
A. wish to escape his circumstances
research
B. desire to merge his family’s Texan and
E. rigors of conducting genealogical
Mexican identities
investigations

11
20. The scientific language used in lines 47-52 B. a way to remain in touch with his
(“Lico . . . ancestors”) emphasizes the family’s past without keepsakes
A. forcefulness of a shared fascination C. an area in Texas that reminds him of
B. chaotic methods used by the narrator’s the home he had left
uncle D. stories that supply direct answers to
C. distillation of information about the the questions in his dreams
narrator’s past E. a method of using the land as a valuable
D. place of family systems in the natural source of income
world 24. The overall tone of the passage is best
E. intersection of two separate family described as
lines A. analytical
21. The narrator indicates that the questions B. whimsical
his ancestors pose (lines 54-62) are ones C. dramatic
that D. reflective
A. he cannot possibly answer truthfully E. speculative
B. are meant to forewarn as well as
confuse
Questions 25-30 are based on the following
C. are not really intended to elicit a
passage.
response
D. contain the answers hidden within This passage is excerpted from an essay about
themselves the novelist Jane Austen (1775-1817).
E. have been asked before and will be
When I read collections of letters by
asked again
eminent authors, I am now and then disposed
22. The characterization of the “bodies” in line
to suspect that the writers had at the back of
62 underscores the narrator’s
their minds the notion that one day the letters
preoccupation with
5 might find their way into print. When I learn
A. genealogical method
that they had kept copies of their letters, the
B. personal destiny
suspicion is changed into certainty. When
C. family harmony
André Gide wished to publish his
D. familial identity
correspondence with Claudel, and Claudel, who
E. genetic variability
10 perhaps didn’t wish it to be published, told him
23. The last paragraph suggests that the
that the letters had been destroyed, Gide
narrator has discovered
answered that it was no matter since he had
A. a collection of cedar mementos left by
kept copies of them. Whenever Charles Dickens
his great-uncle
went on a journey, he wrote long letters to his

12
15 friends in which he eloquently described the Since she says that her conversation was
sights he had seen, and which, as his first exactly like her letters, and her letters are full of
biographer justly observes, might well have witty, ironic, and malicious remarks, we can be
been printed without the alteration of a single pretty sure that her conversation was
word. People were more patient in those days. 55 delightful.
20 Still, one would have thought it a
25. The “suspicion” mentioned in line 7 refers
disappointment to receive a letter from a friend
to
only to find that it provided word pictures of
A. an uncertainty about how a letter will
mountains and monuments when you would
end
have been glad to know whether your friend
B. a doubt about the literary merit of
25 had run across anyone of interest, had been to
some authors
any interesting parties, and had been able to get
C. a skepticism about Austen’s letters to
you the books you wanted.
her sister
Most of the letters of Jane Austen that
D. a belief about the way a certain group
have survived were written to her sister
of people behave
30 Cassandra. Many of Austen’s warmest admirers
E. a feeling about how a particular event
have found the letters to be paltry. These
will turn out
people have said they showed that she was cold
26. The list in lines 24-27 (“whether . . .
and unfeeling and that her interests were
wanted”) provides examples of
trivial. I am surprised. The letters are very
A. individual insight
35 natural. Austen never imagined that anyone but
B. personal information
Cassandra would read them, and she told her
C. embarrassing revelations
sister just the sort of things she knew would
D. eloquent musings
interest her. She wrote about what people were
E. dramatic statements
wearing, how much she had paid for the
27. The author of the passage discusses Jane
40 flowered muslin she had bought, what
Austen (lines 28-55) primarily in order to
acquaintances she had made, what old friends
A. compare the novels of Austen to those
she had met, and what gossip she had heard.
of Dickens and Gide
In one of her letters, Austen said, “I have
B. contrast letters written by ordinary
now attained the true art of letter writing,
people with those written by
45 which we are always told is to express on paper
celebrities
exactly what one would say to the same person
C. explain why letters written by eminent
by word of mouth. I have been talking to you
authors are usually interesting to read
almost as fast as I could the whole of this letter.”
D. emphasize the distinctive style of
Of course, she was right. That is the art of letter
Austen’s novels
50 writing. She attained it with consummate ease.

13
E. champion a particular kind of letter D. Regret
writing E. Cynicism
28. The “people” mentioned in line 32 would 30. The author of the passage suggests that an
probably consider the subjects listed in important difference between the letters of
lines 38-42 (“what . . . heard”) to be Gide and Dickens and the letters of Austen
A. representative of Austen’s artistry is the
B. worthy of more detailed investigation A. ultimate intended audience of the
C. witty and ironic critiques letters
D. interesting only to academic specialists B. era during which the letters were
E. boring and mundane matters written
29. The author of the passage demonstrates C. gender and nationality of the letter
which attitude toward the “malicious writers
remarks” (line 53) ? D. number of surviving letters by each
A. Appreciation author
B. Curiosity E. influence of the letters on each author’s
C. Puzzlement novels

STOP
If you finish before time is called, you may check your work on this section only.
Do not turn to any other section in the test.

14
Questions 1-7: Each of the questions below B. talented . . extrovert
contains a sentence that has one or two blanks. C. busy . . retiree
The blanks stand for a part of the sentence that D. eminent . . recluse
has not been included. Following each sentence E. famous . . magnate
are five answer choices. Choose the answer 5. The book Silent Spring was published in
choice that best completes the sentence. 1962, and first exposed the ----- of using
toxic chemicals to ----- crop-eating insects.
1. ----- disposing of an old refrigerator is often
A. benefits...decimate
----- difficult. Some people abandon them at
B. hazards...exterminate
the landfill entrance.
C. convenience...massacre
A. Responsibly . . prohibitively
D. trepidation...eliminate
B. Reasonably . . curiously
E. vulnerability...eradicate
C. Diffidently . . eminently
6. When Shirley Chisholm ran for the
D. Satisfactorily . . completely
Democratic Party’s nomination for
E. EffIciently . . impossibly
president in 1972 her ----- was ----- for both
2. Women's baseball leagues became -----
African Americans and women.
during World War II, when many
A. attempt...unprecedented
professional baseball players were ----- in
B. bid...idiosyncratic
the military.
C. enthusiasm...exceptional
A. erratic . . interested
D. trepidation ...singular
B. possible . . enamored
E. struggle...unparalleled
C. prevalent . . engaged
7. It is doubtful that an untrained adult would
D. compatible . . involved
be able to master the ----- of playing the
E. sporadic . . integral
harp in our orchestra without ----- coaching
3. ----- writing, such as that found in The New
from Anna Delgardo.
Yorker magazine, tends to appeal to a
A. intricacies...intensive
particular group of ----- readers.
B. minutiae...exhaustive
A. interesting . . refined
C. workings...remedial
B. gauche . . stylish
D. information...demanding
C. urbane . . complicated
E. particulars...corrective
D. flowery . . educated
E. exceptional . . erudite
4. Mary Pickford was once one of the most ----
actors in Hollywood, so it is almost an
oxymoron that she died a ----- at her estate.
A. wealthy . . mogul

15
Questions 8-16: Listed beside each number B. chapters: book
below is a pair of words. Choose the pair of C. words: speech
words that are the most similar to the example. D. rhythms: song
E. dialects: language
8. SKULK : STRIDE :: 14. HACKNEYED: REMARKS::
A. bristle : fur A. novel: conjecture
B. cogitate : mull B. ludicrous: jokes
C. wheedle : coax C. bromidic: speech
D. whisper : talk D. tacky: guise
E. precursor : antecedent E. unique: perspective
9. PROSELYTIZE : lNDUCE :: 15. CONTRIVE: ALIBI::
A. imbibe : remove A. excuse: behavior
B. flummox : explain B. dispel: naysayer
C. displace : reposition C. initiate: legislation
D. repose : agitate D. plagiarize: create
E. dubious : decided E. create: explanation
10. RETICULATE : HIGHWAY :: 16. BEATIFIC: BRIDE::
A. funereal : sad A. plaintive: infant
B. canton : region B. petulant: puppy
C. itinerary : schedule C. dastardly: lion
D. ballot : election D. despondent: clergy
E. circuit : course E. delirious: fan
11. REDOUBTABLE : MOTHER ::
A. capricious : purpy
B. narcissistic : teenager
C. cherished : spouse
D. stately : house
E. congenial : child
12. JADED : IDEAS ::
A. invigorated : nap
B. befuddled : puzzle
C. bored : speech
D. optimistic : inauguration
E. anticipate : gift
13. IAMBIC : POETRY ::
A. syllables: word

16
Questions 17-23 refer to the following passage. After reading the passage, read and respond to the
questions that follow by selecting the best choice for each one.

Statins are a class of drugs that work to decrease the level of cholesterol in the blood. They are
able to perform this function by effectively blocking the cholesterol-producing enzyme in the liver. In
recent years, statins have increased in utility and popularity.
Although a certain amount of cholesterol is imperative for the human body to function, an
elevated level can cause a precarious situation in the body. Cholesterol affixes itself to arteries, lining
them and inhibiting blood flow. Since less blood can travel through those encumbered arteries, the
blood flow to the heart is adversely affected. Scientists have found that the walls of arteries in the body
can become inflamed from this plaque buildup. In many instances, the end result of this decreased
blood supply and inflammation is a heart attack sometimes a fatal one. In other cases the blood supply
to the brain is compromised by the plaque buildup, often causing a stroke, Decreased blood flow to the
legs can cause leg pains or cramps. In short, arterial plaque buildup is never a healthy situation.
Statins diminish the amount of cholesterol generated by the body. Although the drugs are not
always able to reduce the amount of plaque that may already be clogging arteries, they can slow the
production of new plaque. The drugs are also able to stabilize the plaques that are already present and
make them less likely to cause problems in the body.
Lowered cholesterol does not guarantee that a heart attack won't occur, but statin use will
lower the risk for most patients. Not everyone who has a heart attack has high cholesterol levels, but
most do have plaque formations on their arteries. It should be noted that the plaque is not always
formed by high levels of cholesterol in the blood.
Statins are generally prescribed by doctors for people with elevated cholesterol levels. As the
mean weight of the American population has risen, so has the number of individuals with high
cholesterol. Millions of men and women in this country are prescribed statins in an effort to decrease
the amount of cholesterol in their blood. Remarkably, these effects can be seen in as little as two weeks
after beginning a statin regime.

17. The author’s main point is that:


A. Cholesterol can cause heart attacks.
B. Statins can diminish the amount of cholesterol in the blood.
C. Most people have plaque buildup in their arteries.
D. Statins are not able to decrease the amount of plaque lining the arteries.
E. Heart attacks are not the only danger of high cholesterol.
18. Which of the following does the author imply about statins?
I. They can be lifesaving drugs.

18
II. They are overprescribed by doctors
III. They are expensive drugs to produce.
A. I only
B. II only
C. III only
D. I and II only
E. II and III only
19. Which of the following is suggested about the American population?
A. Their life expectancy has decreased in the past few decades.
B. They don't get enough exercise.
C. Their diet may be affecting their cholesterol levels.
D. They want an easy fix for a difficult problem.
E. There are a record number of heart disease deaths in the United States.
20. According to the article, which of the following is true of statins?
I. They can help rid the arteries of dangerous plaque.
II. They are able to affect the body relatively quickly.
III. They ere an inexpensive drug.?
A. I only
B. II only
C. III only
D. land II only
E. II and Ill only
21. According to the article, which is true of cholesterol?
I. A certain amount of cholesterol is needed by the body.
II. Cholesterol may be dangerous to the body.
III. A low cholesterol level means a person likely won't have a heart attack.
A. I only
B. II only
C. III only
D. I and II only
E. II and III only
22. The author concedes that statins’ job is primarily to:
A. slow the production of cholesterol in the body.
B. eliminate the possibility of a heart attack.
C. increase the amount of blood flowing to the heart.

19
D. attack lhe cholesterol in the human body.
E. improve the condition of a person’s arteries.
23. The author organizes the passage by:
A. enumerating reasons why statins are useful drugs.
B. debating the importance of statins.
C. criticizing the American public's diet and health.
D. outlining ways to reduce cholesterol in the blood.
E. discussing ways doctors treat high cholesterol.

Questions 24-27 refer to the following passage.

One way to deliver news is with a photograph. Well placed images can enhance and further
explain details of news stories while impacting the viewer emotionally. Indeed, the main goal of a news
photograph is not necessarily to be aesthetically pleasing, but to objectively deliver the news by way of
a graphic image.
Interpreting the news through photographs is not an effortless task, even for the experienced
photojournalist. What separates a good news photograph from a typical photo is that a good one will
not just be a picture. It will be a complete sentence, with a subject, a verb, and a verb object. It won't just
say “the dog” or “the boy,” but it will say, “The dog saved the boy and seemed to know what it was
doing." Good photographs won't be posed or set up, but will be a slice of the news story as it happens.
Most professional photo journalists know that the best photographs they can take are those that are
able to provoke emotion. A snapshot is not simply taken of a plane that has been involved in a runway
crash. A good photojournalist will try to find a way to show a Dart of the story: a distraught person, an
ice plagued runway, or even a spent firefighter. This can mean time invested waiting around for the
right moment and the right photograph. Professional photo journalists know and expect this, and know
that it is time well spent when they get the shot that shows the story.

24. The author's main point is that:


A. it's not easy to take a good photograph.
B. photojournalists are professionals.
C. photographs are an important part of the news.
D. news photographs are not just pictures.
E. it's wrong to pose a news photograph.
25. Which of the following does the author imply about photojournalism?
I. It's not as easy as it looks to take a news photograph.
II. Most photojournalists have strong relationships with those they photograph.
III. Most people can be taught to take a good photograph.

20
A. I only
B. II only
C. III only
D. I and II only
E. II and III only
26. Which statement is true about an objectively deliverey news photograph?
I. It shows the photographer's feelings about the subject.
II. It may show the photographer’s personal bias.
III. It will be devoid of the photographer’s feelings.
A. I only
B. II only
C. III only
D. I and II only
E. II and III only
27. As implied by the passage, which part of speech does a good photograph mainly capture?
A. adverb
B. noun
C. adjective
D. verb
E. preposition

21
Each sentence below has one or two blanks, 4. During the 1990’s, Shanghai benefited from
each blank indicating that something has an architectural -------, the result of a
been omitted. Beneath the sentence are five dramatic increase in innovative and artistic
words or sets of words labeled A through E. building.
Choose the word or set of words that, when A. intransigence
inserted in the sentence, best fits the meaning B. plenitude
of the sentence as a whole. C. desecration
D. stagnation
1. Much of our knowledge of dinosaurs comes
E. renaissance
from excavated bones, which, in -------
5. Many subatomic nuclear particles are -------
other clues such as fossilized tracks and
and nearly -------: they are hard to track as
eggs, help us to ------- the evolution of these
well as to detect.
creatures.
A. unstable . . explosive
A. convergence with . . supplant
B. elusive . . imperceptible
B. divergence from . . decode
C. minute . . immobile
C. dependence on . . belie
D. charged . . reactive
D. opposition to . . amplify
E. tenuous . . indivisible
E. conjunction with . . trace
6. Luisa worked with extreme precision, ------
2. Responding to criticism that the script was
that served her well in her law career.
rambling and -------, the new screenwriter
A. a meticulousness
revised the dialogue for greater
B. an effrontery
succinctness and -------.
C. an inhibition
A. engaging . . simplicity
D. a litigiousness
B. subjective . . ambiguity
E. an impetuousness
C. muddled . . clarity
7. The crafty child tricked his innocent
D. terse . . emptiness
brother, a particularly ------- and trusting
E. difficult . . abstraction
boy, into committing a mischievous prank.
3. Vernal pools are among the most ------- of
A. guileless
ponds: they form as a result of snowmelt
B. intrusive
and a high water table in winter, and then
C. astute
they ------- by late summer.
D. opportunistic
A. transitory . . expand
E. circumspect
B. anachronistic . . overflow
8. Ellen Ochoa’s ------- with the apparatus in
C. immutable . . drain
the space shuttle Discovery was apparent
D. itinerant . . teem
when she adroitly manipulated the
E. ephemeral . . evaporate
shuttle’s robot arm.

22
A. compromise D. embroidery : needlework
B. humility E. loom : weaving
C. machinations 13. DESPICABLE : SCORN ::
D. synergy A. resourceful : cleverness
E. deftness B. correctable : error
9. In 1916 Yellowstone National Park had C. eligible : qualification
only 25 bison, but the population has since D. laughable : respect
------- to more than 2,000. E. noteworthy : attention
A. dispersed 14. LANDSCAPE : PAINTING ::
B. mediated A. biography : life
C. attenuated B. cartoon : newspaper
D. burgeoned C. canvas : easel
E. reconciled D. ventriloquism : voice
E. anthem : song
Each question below consists of a related pair 15. STREAMLINE : COMPLEXITY ::
of words or phrases, followed by five pairs of A. improve : efficiency
words or phrases labeled A through E. Select B. deliver : destination
the pair that best expresses a relationship C. corrupt : impurity
similar to that expressed in the original pair. D. stretch : elasticity
E. depreciate : value
10. BROOM : DIRT ::
A. chimney : soot
B. mop : floor
C. rake : leaves
D. shovel : hole
E. hammer : fragments
11. BURNER : COOK ::
A. lampshade : brighten
B. furnace : fuel
C. boat : float
D. freezer : thaw
E. fan : ventilate
12. CALLIGRAPHY : HANDWRITING ::
A. photography : film
B. stenography : speech
C. pottery : clay

23
Each passage below is followed by questions based on its content. Answer the questions on the
basis of what is stated or implied in each passage and in any introductory material that may be
provided.

Questions 16-21 are based on the following This happiness, like the artist’s, must come from
passage. something within the animal, something
trainers call talent, and so cannot be imposed
The following passage is from a 1994 collection
on the animal. But at the same time it does not
of essays about animals, written by a poet,
30 arise in a vacuum; if it had not been a fairly
philosopher, and animal trainer.
ordinary thing in one part of the world at one
The question that comes first to my point to teach young children to play the
mind is this: What would it mean to say that an harpsichord, it is doubtful that Mozart’s music
animal has the right to the pursuit of would exist. There are animal versions, if not
happiness? How would that come about, and in 35 equivalents, of Mozart, and they cannot make

5 relationship to whom? their spontaneous passions into sustained


In speaking of “animal happiness,” we happiness without education, any more than
often tend to mean something like “creature Mozart could have.
comforts.” The emblems of this are the golden Aristotle identified happiness with
retriever rolling in the grass, the horse with his 40 ethics and with work, unlike Thomas Jefferson,

10 nose deep in the oats, kitty by the fire. Creature who defined happiness as “Indolence of Body;
comforts are important to animals: “Grub first, Tranquility of Mind,” and thus what I call
then ethics” is a motto that would describe creature comforts. Aristotle also excluded as
many a wise Labrador retriever, and I have a unethical anything that animals and artists do,
bull terrier named Annie whose continual quest 45 for reasons that look wholly benighted to me.
15 for the perfect pillow inspires her to awesome Nonetheless, his central insights are more
feats. But there is something more to animals, helpful than anything else I know in beginning
something more to my Annie, a capacity for to understand why some horses and dogs can
satisfactions that come from work in the full only be described as competent, good at what
sense something approximately like what leads 50 they do, and therefore happy. Not happy
20 some people to insist that they need a career because leading lives of pleasure, but rather
(though my own temperament is such that I happy because leading lives in which the
think of a good woodcarver or a dancer or a sensation of getting it right, the “click,” as of the
poet sooner than I think of a business executive pleasure that comes from solving a puzzle or
when I contemplate the kind of happiness 55 surmounting something, is a governing
25 enjoyed by an accomplished dressage¹ horse). principle.

24
¹Dressage is a complex series of movements signaled to a E. The author considers subjectivity and
horse by its rider.
selfknowledge to be critical to human
gratification.
16. The author presents examples in lines 8-10
19. The author’s discussion of Mozart in lines
in order to
30-34 primarily emphasizes the
A. illustrate the variety of activities in
A. role of social circumstances in the
which animals engage
emergence of a musical genius
B. suggest that appearances of happiness
B. fact that young children are sometimes
are deceptive
pushed to excel
C. evoke images of contentment
C. observation that genius was more
D. support an apparently implausible
common in the past than it is today
argument
D. belief that the harpsichord was the
E. arouse nostalgic longings
ideal musical instrument for Mozart’s
17. The motto in line 11 indicates that animals
early talent
A. are much more intelligent than many
E. pleasure that artists derive from
people believe
achievement
B. have been forced to develop keen
20. In line 36, “passions” most nearly means
survival skills
A. love affairs
C. desire consistency in their daily lives
B. violent outbursts
D. enjoy close relationships with human
C. enthusiasms
beings
D. prejudices
E. are concerned primarily with
E. sufferings
immediate physical gratification
21. Which situation most accurately illustrates
18. Which of the following statements is most
the author’s definition of a happy animal?
consistent with the author’s discussion of
A. A bird finding its one lifetime mate
“temperament” in lines 21-25?
B. A dog herding sheep into a pen
A. The author believes a poet can be
C. A horse being carefully groomed for a
successful in business.
show
B. The author considers artistic pursuits
D. A monkey escaping from a city zoo
to be the most personally fulfilling of all
E. A cat caring devotedly for her kittens
endeavors.
C. The author suspects that a busy life can
have its own rewards.
D. The author believes that few people are
ever satisfied with the jobs they have
chosen.

25
Questions 22-30 are based on the following assessment is for me the most stunning
passage. surprise dealt by the rock from Mars even more
mind-boggling than the suggestive traces of
This passage was written in 1996 after the
something that might once have lived and died
discovery of a meteorite that appeared to contain
35 in its microscopic fissures.
fossil evidence of microscopic life on Mars.
I cannot resist comparing this new
The rock that sprang to Martian “life” intimacy with our solar system to the shoebox
late last summer did not shock me by offering diorama of the planets I designed for my grade-
up apparent fossils of an extinct alien form of school science fair. I used marbles, jack balls,
life. I had long believed that the universe teems 40 and Ping-Pong balls, all hanging on strings and
5 with life elsewhere, and that our failure to find painted different colors, all inside a box
it simply results from a lack of exploration. representing our solar system. This crude
What did amaze me about the potato-size rock assortment of materials allowed a reasonable
that fell from Mars was that it had traveled representation of what was known 40 years ago
millions of miles across space to land here, 45 about the nine planets: Mars was red and had
10 blasted from world to world by a planetary two moons; Jupiter dwarfed the other planets (I
collision of the sort that purportedly killed off should have used a basketball but it wouldn’t fit
our dinosaurs, and had lain waiting for in the box); Saturn had rings. If my school-age
millennia upon an Antarctic ice field, until an daughter were to attempt such a construction
observant young woman traveling in an 50 today, she’d need handfuls of jelly beans and
15 expedition party picked it up, because she gum balls to model the newly discovered
figured that it had come from another world. satellites of the giant planets. She’d want rings
How could she know such a thing? around Jupiter, Uranus, Neptune, too, not to
The composition of ALH 84001, as the mention a moon for Pluto.
much scrutinized rock is designated, closely 55 Similarly, our solar system, once
20 matches the makeup of Martian matter that was considered unique, now stands as merely the
analyzed on site in 1976 by miniature first known example of a planetary system in
chemistry laboratories aboard two Viking Mars our galaxy. Since October of 1995, astronomers
landers. As a result of this positive at ground-based observatories in Europe and
identification, no astronomer seriously doubts 55 the United States have announced that they’ve
25 the meteorite’s Martian origin. Researchers found evidence of at least seven alien planets
think they have pinpointed its former resting orbiting other stars. As yet, not one of these
place to just two possible sites a region called large planets—some of which are many times
Sinus Sabaeus, fourteen degrees south of the the mass of Jupiter—has actually been seen
Martian equator, or a crater east of the Hesperia 60 through a telescope; we know about them
30 Planitia region. The bold precision of this indirectly through the gravitational effects they

26
exert on their parent stars. Yet, even though we C. Stipulated
have no picture of what they look like, enough D. Selected
information has been deduced abouttheir E. Allocated
65 atmospheric conditions to grant the nickname 24. The author considers the researchers’
Goldilocks to a planet attending the star 70 conclusion “bold” (line 30) primarily
Virginis, an appellation suggesting that the because it
cloud-top temperature is “just right,” as the A. daringly offers two conflicting answers
storybook Goldilocks would say, for the to a single question
70 presence of liquid water. Liquid water, not B. firmly supports a young geologist’s
known to exist anywhere in our solar system tentative theory
now except on Earth, is thought crucial to C. confidently uses a small piece of
biological life; thus, only a short leap of faith is evidence to build an exact hypothesis
needed to carry hopeful scientists from the D. courageously defies a conventional
75 presence of water to the existence of interpretation of Antarctic evidence
extraterrestrial life. To raise the specter of the E. defiantly espouses an unpopular theory
Mars rock once again, the primitive life-forms about comets in our solar system
that pressed their memory inside it likewise 25. The author uses the phrase “this new
suggest an era when dry-as-dust Mars was a intimacy” (line 36-37) to refer to the
80 wet world, where rivers flowed. A. hands-on quality of the learning
experience represented by the
22. In lines 7-16, the author suggests that the shoebox diorama
expeditionist’s discovery of the meteorite B. understanding that nonspecialists now
was surprising primarily because it have about meteorological phenomena
A. defied scientists’ doubts that such an C. general acceptance of the theory that
object could reach Earth biological life once existed on Mars
B. occurred after her party had given up D. increased knowledge that scientists
any hope of success have about our solar system
C. resulted from a seemingly unlikely E. way that events on one planet affect
sequence of events those on another
D. provided evidence to contradict a long- 26. In line 42, “crude” most nearly means
standing scientific theory A. natural and unaltered
E. led to an unprecedented degree of B. rough and inexpert
scrutiny C. obvious
23. In line 19, “designated” most nearly means D. vulgar
A. Drawn E. nonspecific
B. Called

27
27. In line 54, the author refers to Pluto’s moon B. The planet’s atmosphere was once
most likely in order to thought to be too cold to support
A. illustrate a feature of our solar system biological life.
discovered since the author’s childhood C. The simple methods astronomers used
B. cite an object too small in scale to have to discover the features of this planet
been included in the author’s diorama resemble the explorations of curious
C. draw a parallel between it and our own children.
moon D. Scientists’ wishful speculations about
D. contrast the scientific curiosity of the existence of this planet deserve
today’s children with that of children little more credence than a fairy tale.
years ago E. Only after much trial and error did
E. emphasize the need for a greater astronomers determine the precise
commitment to space exploration location of this planet.
28. The reasoning process presented in lines 30. Which of the following, if true, would the
57-62 (“As . . . stars”) is best described as “hopeful scientists” (line 74) most likely
A. inference based on an untested theory interpret as evidence of the potential for
B. extrapolation from similar situations life on Mars?
C. analysis of a single case by multiple A. Mars was affected by the same
observers planetary collision that caused the
D. hypothesis confirmed by direct extinction of dinosaurs.
observation B. Mars had a very mild atmospheric
E. comparison of theory with physical temperature millions of years ago.
evidence C. Mars had a wet environment at one
29. In lines 62-70 the author refers to the time in the past.
Goldilocks fairy tale (“Yet . . . water”) in D. The rock that fell from Mars resembled
order to make which point about a rocks found on the Antarctic ice field.
particular planet? E. The rock that fell from Mars had very
A. The planet’s environment may be few microscopic fissures.
conducive to a result some scientists
are eager to find.

STOP
If you finish before time is called, you may check your work on this section only.
Do not turn to any other section in the test.

28
SECTION 4
Time — 30 minutes
35 Questions

Directions: For each question in this section, select the best answer from among the choices given
and fill in the corresponding oval on the answer sheet.

Each sentence below has one or two blanks, each A. varied . . interposed
blank indicating that something has been B. diverse . . condensed
omitted. Beneath the sentence are five words or C. profound . . magnified
sets of words labeled A through E. Choose the D. transformative . . embellished
word or set of words that, when inserted in the E. impressive . . immersed
sentence, best fits the meaning of the sentence as 4. The representative was a traditionalist,
a whole. reluctant to support any legislation
inconsistent with the nation’s most -------
1. Originally ------- mainly by young, urban
principles.
audiences, rap music was ultimately -------
A. orthodox
by its appreciative listeners of all ages
B. impassioned
across the country.
C. precarious
A. admired . . embraced
D. impressionable
B. performed . . condemned
E. indeterminate
C. derided . . ignored
5. Historian Carlo Botta often contradicted
D. appropriated . . relinquished
himself, as when he first championed and
E. applauded . . instigated
then ------- the ideals of the French
2. It was out of ------- that Professor Green,
Revolution.
the author of several highly respected
A. invoked
books in his field, described himself to his
B. investigated
colleagues as -------.
C. conceived
A. embarrassment . . a paragon
D. coveted
B. magnanimity . . an avenger
E. denounced
C. insolence . . a pedant
6. The author constructed a scenario in which
D. egotism . . an apprentice
playful, creative children are rewarded for
E. modesty . . a dilettante
their ------- and strict, dour adults are
3. The ------- experiences of Madonna Swan,
punished for their -------.
the 1983 North American Indian Woman of
A. spontaneity . . rigidity
the Year, cannot be fully appreciated if they
B. digressions . . mirth
are ------- in a tidy summary.

29
C. solemnity . . malice E. vilified
D. inflexibility . . rigor
E. improvisations . . buoyancy Each question below consists of a related pair of
7. Although usually warm and ------- in words or phrases, followed by five pairs of words
greeting friends, Lauren was too reserved or phrases labeled A through E. Select the pair
ever to be truly -------. that best expresses a relationship similar to that
A. joyous . . conventional expressed in the original pair.
B. cordial . . effusive
11. BOTTLE : CONTAINER ::
C. restrained . . gracious
A. door : room
D. dismissive . . ebullient
B. well : bottom
E. genial . . antisocial
C. knife : tool
8. Legal scholars argue that when “justice” is
D. beaker : fluid
interpreted too broadly, the concept
E. medicine : dose
becomes -------, easily changed and
12. KELP : OCEAN ::
controlled by outside forces.
A. cactus : desert
A. Malleable
B. alga : plant
B. influential
C. spider : web
C. coherent
D. surf : beach
D. felicitous
E. avalanche : snow
E. prosaic
13. BEES : HONEY ::
9. The instructor’s voice was so ------- that
A. moth : flame
most students preferred taking a test to
B. fish : bait
listening to its grating sound.
C. birds : feathers
A. receptive
D. sheep : herd
B. cajoling
E. spiders : webs
C. melodious
14. COAT : BODY ::
D. muted
A. bracelet : wrist
E. strident
B. uniform : team
10. Though surgeon and researcher Charles
C. mitten : hand
Drew never enjoyed celebrity, he truly
D. shirt : sleeve
deserves to be ------- for his life’s
E. bandage : cut
achievements.
15. LION : PRIDE ::
A. mollified
A. hawk : prey
B. lionized
B. whale : mammal
C. accosted
C. parrot : jungle
D. galvanized

30
D. sheepdog : flock D. subject : biography
E. wolf : pack E. narrator : anecdote
16. ERRATIC : CONSISTENCY :: 22. LABYRINTH : PERPLEXITY ::
A. lethargic : energy A. prison : liberty
B. eccentric : oddity B. harbor : navigation
C. spellbound : charm C. refuge : relief
D. scintillating : conversation D. lair : danger
E. feverish : delirium E. mirage : image
17. DEFOLIATION : LEAVES :: 23. FOIBLE : VICE ::
A. contamination : germs A. frugality : generosity
B. evaporation : dryness B. obsession : persistence
C. delimitation : boundaries C. venom : malevolence
D. acceleration : speed D. oath : loyalty
E. dehydration : moisture E. fib : lie
18. DEBACLE : FAILURE ::
A. catastrophe : assistance
B. impediment : progress
C. chaos : confusion
D. indecisiveness : action
E. anarchy : order
19. GRIMACE : PAIN ::
A. applause : praise
B. inquiry : curiosity
C. sneer : contempt
D. gesture : offense
E. hug : affection
20. EPIC : HEROIC ::
A. euphony : discordant
B. prose : theatrical
C. melodrama : exaggerated
D. instrument : harmonious
E. satire : hypocritical
21. PROTAGONIST : NOVEL ::
A. contributor : anthology
B. editor : newspaper
C. reviewer : movie

31
The two passages below are followed by questions based on their content and on the relationship
between the two passages. Answer the questions on the basis of what is stated or implied in the
passages and in any introductory material that may be provided.

Questions 24-35 are based on the following sense of being at fault for chronic
passages. disagreements. Many women and men feel
dissatisfied with their close relationships—with
Is a person’s gender an important influence on
25 spouses, siblings, parents—and become even
how he or she behaves with others?
more frustrated when they try to talk things
Contemporary sociologists and other scholars
out. Taking a sociolinguistic approach to such
have argued this question fiercely. The following
troubling encounters makes it possible to
pair of passages presents two contrasting voices
explain these dissatisfactions without accusing
from that debate.
30 anyone of being wrong and without blaming—
Passage 1 or discarding—the relationship.
The desire to affirm that women and The sociolinguistic approach I take in
men are completely equal has made some my work is based on my belief that many
scholars reluctant to show ways in which they frictions arise because, here in the United
are different, because differences between two 35 States, boys and girls grow up in what are

5 groups of people have so often been used to essentially different cultures, so that talk
“justify” unequal treatment and opportunity. between women and men is actually cross-
Much as I understand and am in sympathy with cultural communication. For little boys, talk is
those who wish there were no differences primarily a means of making statements of
between women and men—only reparable 40 achievement through games like bragging
10 social injustice—my research on styles of contests. This may also be done by exhibiting
conversation tells me that, at least in this area, knowledge or skill and by holding center stage
it simply isn’t so. I believe that there are gender through such verbal performance as
differences in ways of speaking, and we need to storytelling, joking, or imparting information.
identify and understand them. Without such 45 Little girls appear to be eager to share and

15 understanding, we are doomed to blame others compare interests and ideas. Emphasis is placed
or ourselves—or our own relationships—for on displaying similarities and matching
the otherwise mystifying and damaging effects experiences. For them, the language of
of our contrasting conversational styles. conversation is primarily a language of rapport:
It is clear to me that recognizing gender 50 a way of establishing connection and
20 differences in conversational styles would free negotiating relationships. So this view of
individuals from the burden of an inappropriate children’s behavior predicts that more women

32
than men will be comfortable speaking one-on- contradictory behaviors as they encounter
one, to individuals. And even when addressing different social norms and pressures.
55 an audience, women may be more concerned Some researchers view male-female
than men with establishing rapport. conversations as cross-cultural communication.
90 The two-cultures approach postulates that
Passage 2 difficulties in communication between men and
Gender stereotypes should concern us women arise because of a clash of
for several reasons. First, they may dictate what conversational styles. But this approach has a
we notice and bias our perceptions in the number of limitations. First, the coherence of
60 direction of expectation. Some researchers 95 male and female subcultures in childhood has
attempt to elucidate gender differences in order been exaggerated. We arrive at a contrasting
to help women and men understand and picture of the cultures of boys and girls only by
respond to one another better. In the process, singling out those children who fit common
however, their work encourages people to gender stereotypes and margin alizing others.
65 notice and attend to differences rather than 100 We fail to notice the children who do not fit
similarities, to perceive men and women in those stereotypes—for example, boys who
accordance with stereotypes that may not excel at caring for younger siblings or girls who
accurately depict their behavior or intentions. enjoy building things in shop class. Second,
Second, gender stereotypes may not only although children may choose samesex
70 describe behavior but also prescribe it, 105 playmates as preferred partners, they interact
dictating how men and women “should” behave. daily inside and outside school with the
People begin to act in ways that support other opposite sex. Children have countless
people’s gender-role expectations of them. experiences communicating with people of both
It is time to rethink our understanding sexes: they do not learn to communicate in
75 of gender, to move away from the notion that 110 gendersegregated worlds. They learn to display
men and women have two contrasting styles of different styles of interaction in different
interaction that were acquired in childhood. We contexts: they do not learn a single gender-
need to move from a conceptualization of related style. The same child may display
gender as an attribute or style of behavior to an dominance and give orders to a younger

80 under standing of gender as something people 115 playmate but show deference and follow orders
do in social interaction. As a noted scholar from an older friend.
proposes, “None of us is feminine or is
masculine or fails to be either of those. In
particular contexts people do feminine, in
85 others, they do masculine.” People display

33
24. The primary purpose of Passage 1 is to 27. In lines 45-51 (“Little girls . . .
A. present a historical overview of a relationships”), the author of Passage 1
controversy assumes that for girls, a primary function
B. acknowledge previous errors in of communication is to
thinking A. foster a sense of intimacy between
C. urge changes in organized activities speaker and listener
provided for children B. establish a set of conversational rules
D. assert the value of a particular shared by speaker and listener
approach to an issue C. convey information previously
E. downplay the significance of a recent unknown by the listener
discovery D. promote nostalgic feelings about past
25. Passage 1 makes which suggestion about friendships
the work of “some scholars” (line 2-3) ? E. create an objective atmosphere for
A. It will become widely accepted in the personal discussions
scientific community. 28. The sentence in lines 58-60 in Passage 2
B. It is well intentioned but misguided. (“First . . . expectation”) primarily
C. It attempts to be objective but does not emphasizes which damaging effect of
succeed. gender stereotypes?
D. It puts forth a convincing theory. A. They may offend the person being
E. It could be used to excuse injustice in a stereotyped.
society. B. They may distort our observations of
26. Passage 1 argues that “recognizing gender people we meet.
differences” (line 19-20) would most likely C. They have been used to justify gender
A. cause people to exaggerate their inequality.
similarities when communicating with D. They commonly cause
one another miscommunication between men and
B. lead to further dissatisfaction in women.
conversations among friends and E. They reflect negatively on those who
relatives believe in them.
C. promote the equal treatment of distinct 29. The quotation in lines 82-85 (“None . . .
social groups masculine”) primarily serves to
D. relieve individuals of much of the A. introduce a personal experience
blame for problems in relationships B. provide a typical example
E. affect the way that future research on C. elaborate on an idea
gender is conducted D. signal a change in topic
E. offer recent research data

34
30. Passage 2 suggests that some scholars D. there is less conflict between men and
construct “a contrasting picture of the women than sociologists assume
cultures of boys and girls” (lines 96-97) by E. children’s behaviors have changed
studying children whose dramatically in recent years
A. readiness to interact with strangers is 33. The two passages differ most on which
apparent topic?
B. demand for approval from adults is A. Whether boys and girls communicate
particularly strong in gender specific patterns
C. rebellion against authority results in B. Whether important social behavior is
creative behavior learned in childhood
D. personalities are highly idiosyncratic C. Whether adult conversational styles
E. actions correspond to a narrow can be studied systematically
preconception of behavior D. Whether gender plays a role in
31. The author of Passage 2 implies that the determining a child’s playtime
“boys” mentioned in line 101 and the activities
“child” mentioned in line 103 resemble one E. Whether society concerns itself with
another in that they the concept of gender roles
A. have not learned to imitate adult 34. The assumptions underlying the research
behaviors work described in lines 60-63 of Passage 2
B. refuse to get along with their peers are most similar to the assumptions held
C. do not conform to traditional gender by the
stereotypes A. “scholars” in line 3
D. openly mock adult expectations about B. “women and men” in line 23
their behavior C. “noted scholar” in line 81
E. communicate primarily with children D. author of Passage 1
of their own gender E. author of Passage 2
32. The author of Passage 2 would most likely 35. Which of the following best characterizes
challenge the claim made in lines 32-38 of the ideas about gender communication
Passage 1 (“The sociolinguistic . . . styles as they are presented in the two
communication”) by arguing that passages?
A. children do not grow up in single- A. Passage 1 argues that styles are based
gender cultures on competition, while Passage 2
B. children may become skilled at suggests that they are a form of
deceiving adults cooperation.
C. gender differences are impossible to B. Passage 1 argues that styles are a
assess scientifically burden, while Passage 2 implies that

35
they can help facilitate relationships D. Passage 1 suggests that styles are
between men and women. constant, while Passage 2 argues that
C. Passage 1 claims that styles are they are fluid.
semantic, while Passage 2 suggests that E. Passage 1 states that styles are random,
they are whimsical. while Passage 2 indicates that their
patterns become obvious upon closer
scrutiny.

STOP
If you finish before time is called, you may check your work on this section only.
Do not turn to any other section in the test.

36
ACEPT D. :
E. None
Latihan Grammar and Structure 5. Everyone was shocked when Max
CAPITALIZATION AND PUNCTUATION Smithfield a studious, extremely bright
high school senior decided that college was
Choose the punctuation mark that is needed in
not for him.
each of the following sentences. If no additional
A. ;
punctuation is needed, choose answer E.
B. ,

1. “It isn't fair!” shouted Martin. Coach Lewis C. –


never lets me start the game!” D. :

A. . E. None

B. , 6. Kims assistant, usually so reliable, has been


C. ! late for work three times this week,
D. “ without any excuse.
E. None A. ‘
2. Maurccn’s three sisters, Molly, Shannon, B. ,

and Patricia are all spending the summer at C. ;


their grandmother’s beach house. D. .
A. ; E. None
B. – 7. Before sending out invitations, Margo

C. ! cheched the party date with her mother-in-

D. , law.
E. None A. ,
3. For the centerpieces, the florist B. ;
recommended the following flowers C. –

daisies, tulips, daffodils, and hyacinths. D. .

A. : E. None
B. , 8. “I remember” Luis recollected, “the first
C. . time I was allowed to walk home from
D. ; svhool by myself.”

E. None A. ?

4. Lily is an accomplished gymnast she won B. ,


three medals in her last competition. C. :

A. ; D. ;
B. , E. None

C. ?

37
9. Madeline Larkin out office manager, is the Choose the answer that shows the best
most organized person I’ve ever known. punctuation for the underlined part of the
A. : sentence. If the sentence is correct as is, choose
B. ; E.
C. –
D. , 11. Simone bought three new pairs of shoes
E. None even though she had put herself on a tight
10. I spend most of my time at the gym on the budget just last week.
treadmill walking is my favorite from of A. shoes, even thought, she
exercise. B. shoes, even though she
A. , C. shoes. Even though she
B. ? D. shoes; even though she
C. ; E. correct as is
D. ! 12. Most residents of the building have air
E. None conditioners however I've always found
that a ceiling fan is sufficient.
A. air conditioners however: I've
B. air conditioners, however, I’ve
C. air conditioners however, I've
D. air conditioners; however, I've
E. correct as is
13. “Are you OK," asked Timothy, "Are you
sure you don't want to sit down and rest
for a while?”
A. OK?” asked Timothy. "Are
B. OK?” asked Timothy, "Are
C. OK,” asked 1imothy? "Are
D. OK?” asked Timothy? "Are
E. correct as is
14. The owners of the restaurant maintain that
only organic ingredients are used in their
kitchen.
A. maintain, that only
B. maintain that, only
C. maintain: that only
D. maintain-that only

38
E. correct as is A. about isn't it?
15. Before the student could be hired by the B. about, is'nt it?
company, the students adviser had to C. about, isn’t it.
provide a letter of recommendation. D. about isn't it.
A. company the students E. correct as is
B. company, the student's 19. Turnips a root vegetable can be mashed,
C. company the students’ roasted, or used in casseroles.
D. company the students' A. Turnips, a root vegetable,
E. correct as is B. Turnips, a root vegetable
16. The volunteers who would like to work the C. Turnips, a root vegetable-
morning shift should sign their name on D. Turnips a root vegetable,
this sheet. E. correct as is
A. volunteers, who would like to work the 20. They met for the first time on August 27,
morning shift 1972 in Seattle Washington.
B. volunteers who would like to work the A. August 27 1972 in Seattle, Washington.
morning shift, B. August 27 1972, in Seattle Washington.
C. volunteers, who would like to work the C. August 27, 1972 in Seattle, Washington.
morning shift, D. August 27, 1972, in Seattle, Washington.
D. volunteers who, would like to work the E. correct as is
morning shift,
E. correct as is
17. The employees asked whether the
company would be offering tuition
reimbursement within the next three
years?
A. reimbursement within the next three
years!
B. reimbursement, within the next three
years.
C. reimbursement within the next three
years.
D. remursement, within the next three
years?
E. correct as is
18. This is the new restaurant you've been
talking about, isn’t it?

39
For each queston, find the sentence that has a C. We were staying at my sister's cape
mistake in capitalization or punctuation. If you Cod vacation home.
find no mistakes, mark choice D. D. No mistakes.
26. A. The instructor asked us if we needed
21. A. My least favorite season is winter. more time?
B. Next Friday, Uncle Jake is coming to B. Carla's mother is a pediatric dentist.
visit. C. Every item in the store costs less than a
C. Maureen served as treasurer for the dollar.
women's organization. D. No mistakes.
D. No mistakes. 27. A. Jane's family owned three Persian cats.
22. A. “Can you attend next week's meeting?” B. My Uncle always takes the subway to
she asked. Yankee Stadium.
B. His new car was damaged in the C. Everyone knows that Marisa's favorite
accident. book is Pride and Prejudice.
C. The girls’ giggled through the whole D. No mistakes
movie. 28. A. "I'll do the grocery shopping for you,
D. No mistakes. grandma,” Lucy said.
23. A. Leo told her, to call the customer service B. “Where can I find the best pizza in
department in the morning town?” he asked.
B. She put up signs all over town, but she C. “Be sure to arrive two hours early,” she
didn't get any response. warned.
C. Occasionally, her neighbors ask her to D. No mistakes.
feed their cat. 29. A. I always have a hard time getting up in
D. No mistakes. the morning.
24. A. Did you see the movie Shrek? B. We took: a tent, a cooler, and a sleeping
B. She was given an award by mayor bag.
Chambers. C. The fog was as thick as potato soup.
C. Math and science are my two best D. No mistakes.
subjects. 30. A. This is someone eles coat.
D. No mistakes B. Which of these songs was recorded by
25. A. A major highway is being built on the Bruce Springsteen?
outskirts of town. C. That book must be yours.
B. When you reach the traffic light on D. No mistakes.
Berkshire Road, turn right onto 31. A. Don't stand in my way.
Springfiled B. Cecilia an I fough out way through the
crowd.

40
C. The vegetables were old rubbery and D. countries including
tasteless. E. correct as it is
D. No mistakes. 35. A. fake, throughout
32. A. Remember walk the dog. B. fake. Throughout
B. “Don’t run"! Mr. Ellington shouted. C. fake: throughout
C. It's supposed to snow today and D. fake; throughout
tomorrow. E. correct as it is
D. No mistake. 36. A. death. Calling
B. death, calling
Questions 33-36 are based on the following C. death: calling
passage. First, read the passage, and then choose D. death; calling
the answer that shows the best capitalization E. correct as it is
and punctuation for each underIined part.
Questions 37-40 are based on the following
Madam Helena P. (33) Blavatsky born in Russia
passage. First, read the passage, and then
on May 8, 1831. claimed to have psychic powers
choose the answer that shows the best
and to be capable of performing feats of
capitalization and punctuation for each
clairvoyance and telepathy. During her sixty
underlined part.
years, she traveled to many (34) countries—
including the United States, England, India, and June 2, 2006
Egypt, in order to study the occult. Although
many considered her a (35) fake throughout Melanie Jeffords
her lifetime she was surrounded by faithful 312 Maple Avenue
believers, including such influential persons as Chicago, Illinois 60632
British statesman Allen O. Hume and Swedish
countess Constance Wachtmeister. To this day Mark (37) Franklin, yeneral manager
followers commemorate the date of her (36) Wholesome Food Market
death calling May 8, “white Lotus Day.” 1245 Main Street
Chicago, Illinois 60627
33. A. Blavatsky: born
B. Blavatsky—born
(38) dear Mr. Franklin
C. Blavatsky, born
D. Blavatsky. Born
I am writing to complain about the behavior of
E. correct as it is
one of your sales clerks. On (39) Monday May
34. A. countries, including
22nd I visited your store to return a package of
B. countries: including
ground turkey that had purchased the day
C. countries. Including

41
before. when I explained to your sales clerk that Fill in the blank with the word that creates the
the expiration date on the package was May 1st, most logical sentence. (Hint: Use a dictionary to
she was (40) extremely rude and she refused to determine which words best complete the
refund my money. This is not the kind of sentence's meaning.)
treatment I expect from your fine
41. ________Sarah drives to the cabin several
establishment. I hope you will make restitution
times a year, she is often nervous about
and have a discussion with your staff about
finding her way.
customer service. My receipt is enclosed.
A. Besides
B. Unless
Sincerely yours,
C. Nevertheless
D. Although
Melaine Jeffords
42. Lila wasn’t feeling well. __________, she
decided to stay home from work.
37. A. Franklin, general Manager
A. Therefore
B. franklin, General Manager
B. Meanwhile
C. Franklin, General Manager
C. However
D. Franklin, General manager
D. Anyway
E. correct as it is
43. __________ he waited for the doctor to call
38. A. Dear Mr. Franklin.
B. Dear, Mr. Franklin, him in, Sam sat in the waiting room and

C. dear Mr. Franklin: read the newspaper.


D. Dear Mr. Franklin: A. So that
E. Correct as it is B. Whilet
39. A. Monday, May 22nd I C. Even if
B. Monday May 22nd; I D. Besides when
C. Monday. May 22nd I 44. Ruby loves bluberry pie ______ it is made
D. Monday, May 22nd, I with freshly picked blueberries.
E. correct as it is A. whether
40. A. extremely rude, and she B. because
B. extremely rude: and she C. when
C. extremely rude? And she D. as if
D. extremely rude and, she 45. Mitchell loves listening to jazz and rhythm
E. correct as it is and blues. Greg, ________ will only listen to
country.
A. however
B. then

42
C. too B. and
D. therefore C. but
46. __________ our low annual fee, you will D. was
receive a 20% discount if you sign up this 51. I ______ the speech you gave last Thursday
week. night, but I was in bed with the flu.
A. Because A. will habe heard
B. While B. would hear
C. In spite of C. might hear
D. In addition to D. would have heard
47. The ticket said the show would start at 52. ________ the Beatles’ most popular songs
8:00, but the curtains didn’t go up _________ most of which were written by Lennon and
8:30. McCartney—are “I want to Hold Your
A. less than Hand” and “Hey, Jude.”
B. until A. With
C. about
B. Considering
D. since
C. Among
48. My neighbor is deathly afraid of dogs;
D. To
___________ never let my Golden Retriever,
Sandy, outside without a leash.
A. moreover
B. yet
C. mainly
D. consequently
49. The wedding quilt was designed as a
sentimental way to make use of fabric
taken ________ blankets and bedding that
belonged to older couples in her family.
A. from
B. with
C. in
D. at
50. Sandra Day O’Connor, the first woman to
serve on the U.S Supreme Court, ________
appointed by President Roland Reagan in
1981.
A. she

43
Choose the sentence that best cimbines the D. Plato believed that boys and girls
underline sentences. should be given an equal education,
53. The airport is called the Glynco Jetport. The whereupon this idea is rarely
airline reservations and travel systems mentioned in textbooks.
refer to its location as Brunswick Georgia. 55. Recently there have been government
A. Where the airport is called the Glynco cutbacks in funds. Experts foresee steady
Jetport, the airline reservations and hiring in the government’s future.
travel systems refer to the location as A. Despite recent government cutbacks in
Brunswick, Georgia. funds, experts foresee steady hiring in
B. But the airport is called the Glynco the government's future.
Jetport, the airline reservations and B. whereupon recent government
travel systems refer to the location as cutbacks in funds, experts foresee
Brunswick, Georgia. steady hiring in the government's
C. Even though the airline reservations future.
and travel systems refer to the location C. So that there have been recent
as Brunswick, Georgia, the airport as government cutbacks in funds, experts
called the Glynco Jetport. foresee steady hiring in the
D. when the airport is called the Glynco government's future.
Jetport, the airline reservation to the D. Nonetheless, there have been recent
location as Brunswick. Georgia, and the government cutbacks in funds, experts
travel systems. foresee steady hiring in the
54. Plato bclieved that boys and girls should be government's future.
given an equal education. This idea is 56. The federal government has diversity of
rarely mentioned in textbooks. jobs and geographic locations. The federal
A. Plato believed that boys and girls government offers flexibility in job
should be given an equal an education, opportunities that is unmatched in the
where this idea is rarely mentioned in private sector.
textbooks. A. In spite of its diversity of jobs and
B. Plato believed that boys and girls geographic locations, the federal
should be given an equal education, an government offers flexibility in job
idea that is rarely mentioned in opportunities that is unmatched in the
textbooks. private sector.
C. Believing that boys and girls should be B. No matter its diversity of jobs and
given an equal education, Plato's idea is geographic locations, the federal
rarely mentioned in textbooks. government offers flexibility in job

44
opportunities that is unmatched in the C. The old brain is called the reptilian
private sector. brain, whereupon it does not know
C. Because of its diversity of jobs and passion, but only stolid obedience to its
geograpic locations, the federal own genetic dictates.
government offers flexibility in job D. Unless the old brain, called the reptilian
opportunities that is unmatched in the brain, does not know passion, only
private sector. stolid obedience to its own genetic
D. The federal government has diversity dictates.
of jobs and geograpich locations, so it
offers flexibility in job opportunities
that is unmatched in the private sector.
57. The Greeks thought that the halcyon, I or
kingfisher, nested on the sea. All birds nest
on land.
A. whereupon all birds nest on land, the
Greeks thought that the halcyon or
kingfisher, nested on the sea.
B. The Greeks thought that the halcyon, or
kingfisher, nested on the sea, whereas
all birds nest on land.
C. Whenever all birds nest on land, the
Greeks thought that the halcyon, or
kingfisher, nested on the sea.
D. The Greeks thought that the halcyon, or
kingfisher, nested on the sea, as all
birds nest on land.
58. The old brain is called the reptilian brain. It
does not know passion, but only stolid
obedience to its own genetic dictates.
A. After the old brain is called the
reptilian brain, it does not know
passion, but only stolid obedience to its
own genetic dictates.
B. The old brain, called the reptilian brain,
does not know passion, but only stolid
obedience to its own genetic dictates.

45
Replace the underlined portion with the phrase B. Of the five speeches the mayor gave
that best completes the sentence. If the sentence during May, this was the fifth one.
is correct as is, choose A. C. Thus far during the month of May, the
mayor gave five speeches and this was
59. When making a chocolate torte, only the
the fifth.
best ingredients should be used.
D. This fifth speech of the mayor's given
A. only the best ingredients should be
during the month of May was one of
used.
five speeches.
B. you should use only the best
E. This was the fifth speech the mayor has
ingredients.
given during the month of May.
C. the best ingredients only should be
62. An American poet of the nineteenth
used.
century, Walt Whitman's collection of
D. one should have used only the best
poems, Leaves of Grass, celebrates nature
ingredients.
and individualism.
E. using only the best ingredients is
A. Walt Whitman’s collection of poems,
essential.
Leaves of Grass,
60. With her book Coming of Age in Samoa,
B. Leave’s of Grass, a collection of poems
anthropologist Margaret Mead emphasized
by Walt Whitman,
the role of culture, rather than biology, in
C. a collection of poems, Leaves of Grass,
shaping human behavior.
by Walt Whitman,
A. rather than biology, in shaping human
D. Walt Whitman published poems,
behavior.
collected as Leaves of Grass, that
B. rather than biology with shaping
E. Walt Whitman published a collection of
human behavior.
poems entitled Leaves of Grass, that
C. somewhat better than biology to shape
63. We loved our trip to the desert where you
human behavior.
could see the tall cactus, the blooming
D. in shaping human behavior, and not
flowers, and the little desert animals.
biology.
A. desert where you could see
E. in shaping human behavior over
B. desert; you could see
biology.
C. desert; where we saw
61. This was the fifth of the five speeches the
D. Desert; we saw
mayor gave during this the month of May.
E. desert in that you saw
A. This was the fifth of the five speeches
64. Opposite in what many financial analysts
the mayor gave during this the month
had predicted, the stock market rose by 22
of May.
points this month.

46
A. Opposite in what many financial Choose the sentence that is NOT correctly written
analysts had predicted, or that is unclear. If all sentences are correct,
B. Contrary to the predictions of many choose answer D.
financial analysts, 65. A. We asked him to pick us up in the
C. As against the predictions of many morning.
financial analysts, B. Mrs. Jacobs needed a ride to the
D. Contrasting of many financial analysts’ airport.
predictions, C. The car racing up the street.
E. Contrary with what many financial D. No mistakes.
analysts predicted, 66. A. Our neighbors went on vacation, going
to the Grand Canyon.
B. There are yellow and red tulips in my
garden.
C. We invited Molly to our house for
dinner.
D. No mistakes.
67. A. We are planning to build a new fence in
our backyard.
B. where is the new diner that everyone is
talking about?
C. There's nothing I can do to help.
D. No mistakes.
68. A. Make sure the door is locked.
B. I love pumpkin pie Pearl does too.
C. Yes, I will bring the dessert.
D. No mistakes.
69. A. After he left, I went straight to bed.
B. For the first time, I understood what
she was talking about.
C. We visited the town where my father
grew up last summer.
D. No mistakes.
70. A. Kate was allergic to all dairy products.
B. Which of the Beatles’ songs is your
favorite?

47
C. The company newsletter explained the D. No mistakes.
new vacation policy. 74. A. The free passes were given to Lena and
D. No mistakes. me.
71. A. They went to the park and flew a kite. B. Where's my purple umbrella?
B. “Don't tell me what to do,” she shouted. C. After midnight, the light on the front
C. Liam loves the warm weather, unless porch goes off.
he knows it won't last much longer. D. No mistakes.
D. No mistakes. 75. A. Katya and I were in the same pottery
72. A. Bring your umbrella tomorrow it's class.
supposed to rain. B. The weather was nicer today than it
B. The dancers' costumes were being was yesterday.
delivered on Saturday. C. The grapes cost more lhan the melon
C. Would you consider bringing me as does.
your guest? D. No mistakes.
D. No mistakes. 76. A. His jacket is just like mine.
73. A. Marlene likes my apple crisp better than B. Talia went to yoga class, and that she
Aunt Kate’s. forgot her mat.
B. The people in the auditorium, whether C. Indira visits her relatives frequently.
they were seated or standing. D. No mistakes.
C. I registered for a class in West Indian
literature.

48
Cloze Test Exercise

Directions (1-10): In the following passage 1. A. help B. aiding


there are blanks, each of which has been C. prescribe D. feature
numbered. These numbers are printed below the E. present
passage and against each, five words are 2. A. plus B. lonely
suggested, one of which fits the blank C. ably D. many
appropriately. Find out the appropriate word in E. deeply
each case. 3. A. incorporating B. pressing
B. following D. parting
Visual experiences can (1) children,
E. leaving
teenagers and even adults learn and absorb
4. A. Make B. demand
more due to its highly stimulating and (2) C. impart D. vision
engaging impact. It is for this reason that we are E. need
seeing an increase in schools across the globe 5. A. dissolved B. enhanced
(3) content provider programmes into their C. measured D. failed
class curriculum to (4) lessons through video. E. blasted
Visual excursions and school collaborations are (5) 6. A. deed B. total
by advances in high definition video, high fidelity C. parent D. person
audio and content sharing allowing students to E. lieu
experience a richer and more stimulating learning 7. A. involving B. saving
experience. C. away D. off
Schools that have previously transported E. vacating
students to excursions in (6), now face increased 8. A. let B. enable
transportation costs, higher insurance premiums,
C. present D. pressure
attendance costs for the families and strict duty of
E. collect
care policies for students while (7) school
9. A. clauses B. dictionaries
property/ Virtual excursions (8) students to
C. books D. experts
improve their presentation, research, learning and
E. partners
speaking skills while they engage in a live learning
10. A. vacancy B. availability
session. Students also now have the ability to meet
C. safety D. comfortable
peers from many cultures, speak to subject-matter
E. gap
(9) like scientists or authors practise a foreign
language with students from another country, and
learn about global issues from the (10) of their
own classrooms.

49
Directions (11-20): In the given passage there turn a profit after six years of losses, it is not
are blanks, each of which has been numbered. because they are any better run. In an effort to
Against each five words are suggested, one of (20) a stagnant rural economy, the central bank
which fits the blank appropriately. Find out the has pumped more than $9 billion into them
appropriate word in each case. hoping that they will lend more to farmers. But
the root causes of their problems remain and
If China’s state owned commercial
the real solution may have to involve a mix of
banks seem burdened by bad debts, the
approaches from commercial banking to real
Country’s rural financial sector is even worse.
cooperatives.
In the villages, the only formal banking
institutions are what are known as rural credit 11. A. awarded B. enjoy
co-operatives. These (11) the distinction in C. worry D. making
China of having been officially declared E. trouble
insolvent. The rural credit co-operatives are ill 12. A. sanctions B. apply
named. They are often reluctant to (12) and C. part D. provide
they are not run as co-operatives as they do not E. giving
(13) any profits and their customers have no 13. A. function B. eligible
say in their operations. Until 1996,they were C. claims D. declared
offshoots of the Agricultural Bank of China. E. share
Since then they have been (14) by the Central 14. A. own B. govern
Bank, though they are in reality run by country C. regulations D. ran
government. Even the word ‘rural’ is E. supervised
misleading. (15) of their deposits are sucked up 15. A. Such B. Partly
and put in the urban banking system. Farmers C. Whole D. Most
usually find it easier to (16) from friends or E. Entire
relatives or black market moneylenders. Yet the 16. A. visit B. help
cooperatives remain a big part of China’s C. borrow D. loan
financial system. Last year, they (17) for 12 E. advice
percent of deposits and 11 percent of loans. In 17. A. include B. accounted
recent years, commercial banks (including the C. fulfilled D. achieved
Agricultural bank) have closed down (18) in E. taking
the countryside. Yet some 40,000 credit co- 18. A. branches B. all
operatives remain in place with one in almost C. operating D. staff
every township as the larger villages or smaller E. factory
rural towns are (19). If as the government 19. A. Thinking B. known
claims, the credit cooperatives are beginning to C. creating D. cross

50
E. develop C. met D. forwarded
20. A. make B. release E. dissolved
C. boosting D. stall 25. A. Main B. forced
E. revitalise C. force D. compulsion
E. awareness
Directions (21-25): In the following passage
there are blanks, each of which has been Directions (25-30): In the following passage
numbered. Against each, five words are there are blanks, each of which has been
suggested, one of which fits the blank numbered. Against each, five words are
appropriately. Find out the appropriate word in suggested one of which fits the blank
each case. appropriately. Find out the appropriate word in
each case.
Primary school enrolment in India has been a
success story, (21) due to various programmes Education has been a problem in our
and drives to increase enrolment even in country for (26). The lack of it has been blamed
remote areas. With enrolment reaching at least for all (27) of evil for hundreds of years. Even
96 percent since 2009, and girls (22) up 56 scholars have written lengthy articles about
percent of new students between 2007 and how the Indian education system needs to
2013, it is clear that many (23) of access to change. The funny thing is that fro colonial
schooling have been (24). Improvement in times, things have (28) changed. We have
infrastructure has been the (25) behind established reputed business schools, law
achieving this and now in India 98 percent schools and other institutions of excellence.
habitations have a primary school within one Students, now, so routinely score 90% marks
kilometre and 92 percent have an upper that even with this percentage they find it (29)
primary school within a three kilometre to get into the colleges of their choice. The
walking distance. problem thus lies with us doing more of the
same old staff. This needs to change by bringing
21. A. most B. properly
about (30) in education.
C. totally D. optionally
E. largely 26. A. time B. take
22. A. coming B. reaching C. ever D. long
C. counting D. making E. decade
E. touching 27. A. possession B. abundance
23. A. issue B. opportunities C. typical D. much
C. problems D. efforts E. sorts
E. exertions 28. A. bare B. hardly
24. A. accustomed B. addressed C. little D. much

51
E. highly employers may have to hire even more (37)
29. A. simple B. easy and raise pay if they want to expand their
C. irregular D. noble businesses .The shortage of laid –off workers
E. difficult searching for jobs means that more companies
30. A. innovation B. dreams may need to pay more to (38) talent. Other data
C. creating D. foreign confirm that across the economy, job cuts have
E. choice reached unusually low levels. Total layoffs in
May dropped below prerecession levels. Still,
Directions (31-40): In the following passage while layoffs have fallen 7.5 percent this year,
there are blanks, each of which has been actual hiring has increased just 3 percent,
numbered. These numbers are printed below the That’s a big reasons the job market might not
passage and against each, five words are seem as healthy as the series of strong monthly
suggested, one of which fits the blank net job gains might suggest. Even so, more
appropriately. Find out the appropriate word in people with jobs means more people with
each case. paychecks. Which tends to (39) consumer
spending and growth. After a sharp (40) in the
The (31) of losing your job is getting
economy in the first three months of the year,
smaller and smaller. As economy has improved
most economists expect growth to exceed a 3
and employers have (32) confidence,
percent annual pace in the second half of 2014.
companies have been steadily shedding fewer
workers. The trend means greater job security 31. A. risk B. dangerous
and suggests a critical turning point in the C. risky D. prosperity
economic recovery. It (33) the hope that E. aspect
workers’ pay will finally accelerate after 32. A. regain B. regained
grinding through a sluggish recovery for the C. been regained D. been lost
past half-decade. When the economy (34) into E. lost
recession at the end of 2007, employers cut 33. A. raised B. rose
deeply into their staffs. And then during the C. raises D. diminishes
recovery, they hired only (35). Instead, they E. decreased
sought to maximize the productivity of their 34. A. sink B. sank
existing employees. But in recent months, the C. float D. swam
picture has (36). Employers have added E. floated
200,000-plus jobs for five straight months, and 35. A. hesitating B. peacefully
the unemployment rate has reached 6.1 C. confidently D. rare
percent, the lowest since 2008. Now, the E. hesitantly
steadily declining level of layoffs suggests that 36. A. brightened B. shining

52
C. dooming D. brightening discussions over the extent to which the current
E. doomed growth can be maintained and various means
37. A. aggressively B. aggression by (44) it might be increased.
C. faithful D. hesitant Second, analysts have examined the
E. confidentially behaviour of particular output sectors. A
38. A. subtract B. attract number of authors have studied productivity in
C. attracting D. detract manufacturing – reaching a wide range of
E. demean conflicting conclusions. However, as explained
39. A. strong B. weak indetail by Goldar and Mitra (2002), differences
C. weakening D. boosting in the findings can be (45) to a variety of
E. boost measurement issues, such as the use of singe
40. A. contraction B. contract versus double deflation to construct estimates
C. contracting D. expand of real growth in manufacturing value added.
E. expanding Goldar (2004) provides a careful recent update
showing that TFP growth in manufacturing
Directions (41-50): In the following passage (46) to have slowed in the post reform period –
there are blanks, each of which has been raising additional puzzles discussed below.
numbered. These numbers are printed below the However, (47) difficulties in measuring
passage and against each, five words are employment within individual industries, our
suggested, one of which fits the blank analysis focuses (48) on the broader industrial
appropriately. Find out the appropriate word in sector. The 3 studies that focus on India’s
each case. services sector (many of which discuss the issue
There is already an extensive empirical of sustainability), and those that discuss
literature – often using growth accounts – that agriculture, are discussed in the body of the
(41) these and other aspects of India’s paper.
economic growth. Many of the studies (42) one Given the large body of prior research,
or more of the following topics. First, a number many of the results to be discussed below (49)
of analysts (43) focused on characterizing already wellknown to those in the field. None-
India’s economic performance at the most the-less, this paper seeks to make a
aggregate level. While there is agreement that contribution to that literature in a variety of
growth did indeed improve during the past ways. In particular, the growth accounting
quarter century,researchers have reached framework, combined with our emphasis on
varying conclusions on some issues such as the data issues, pulls together concerns that have
timing and precise magnitude of this typically been treated separately, and in some
acceleration, and the relative importance of cases, raise implications that do not appear to
changes in domestic policy. There are on-going have been (50) recognized. Our updated

53
growth accounts incorporate recent data E. forecast
revisions, some of which are quite large. They 47. A. due to B. because
also provide new estimates for the C. for want of D. than
contributions to overall growth oflabour E. that
productivity growth within major economic 48. A. primary B. primarily
sectors versus the gains from reallocation of C. chief D. prime
labour and capital among the factors. E. elementary
Furthermore, we have examined a 49. A. is B. was
variety of additional data in our analysis of the C. shall D. will
role of capital accumulation - providing E. are
estimates of the returns to schooling for human 50. A. consistent B. consistently
capital, and reporting on trends in sectoral C. hazardly D. irregular
saving and investment, for physical capital. E. turbulently
Thus, this paper is comprised of four remaining
sections. The next section details the Directions (51-60): In the following passage
construction of growth accounts for India, with there are blanks, each of which has been
considerable attention paid to the quality of the numbered. These numbers are printed below the
underlying data. passage and against each, five words are
suggested, one of which fits the blank
41. A. examine B. examines
appropriately. Find out the appropriate word in
C. forecast D. forecasts
each case.
E. augur
42. A. address B. denote Economic development of country is (51)
C. addresses D. facilitate to their industrial growth. In a developing
E. evolve country like India, Small Scale Industries play a
43. A. has B. will significant role in economic development of the
C. should have D. have country. They are a (52) segment of Indian
E. had economy in terms of their contribution towards
44. A. which B. that country’s industrial production, exports
C. if D. whether employment and creation of an entrepreneurial
E. whose base.
45. A. devote B. attributes These industries by and large represent a
C. attributed D. decided stage in economic (53) from traditional to
E. developed modern technology. Small industry plays a very
46. A. appeared B. appears important role in widening the base of
C. looked D. seemed entrepreneurship. The development of small

54
industries (54) an easy and effective means of C. Heighten D. demeaning
achieving broad based ownership of industry, E. demeaned
the diffusion of enterprise and initiative in the 56. A. strategy B. less
industrial field. C. complex D. strategic
Given their importance, the Government E. meagre
policy framework right from the First plan has 57. A. conducive B. congruence
(55) the need for the development of SSI sector C. unsuitable D. unfit
keeping in view its (56) importance in the E. unfair
overall economic development of India. 58. A. high B. higher
Accordingly, the policy support from the C. highest D. measurable
Government towards Small Scale Industries has E. lowest
tended to be (57) and favourable to the 59. A. unsuitable B. suitable
development of small entrepreneurial class. C. suitable D. strategy
Government accords the (58) preference to E. unmatched
development of SSI by framing and 60. A. incentives B. imperatives
implementing (59) policies and promotional C. needs D. improvement
schemes. E. help
The most important promotional policy of
the Government for the SSI’s is fiscal (60) in the Directions (61-70): In the following passage
form of tax concessions and exemptions of there are blanks, each of which has been
direct or indirect taxes leviable on production numbered. Against each, five words are
or profits. suggested, one of which fits the blank
appropriately. Find out the appropriate word in
51. A. related B. relating
each case.
C. concentrated D. resembled
E. assimilated Agriculture is a (61) sector of our
52. A. mean B. vital economy and credit plays an important role in
C. insignificance D. visual increasing agriculture production. Availability
E. insignificant and access to adequate, timely and low cost
53. A. translation B. transferring credit from institutional sources is of great (62)
C. transition D. transitional especially to small and marginal farmers. Along
E. growth with other inputs, credit is essential for
54. A. offer B. offers establishing sustainable and (63) farming
C. differ D. differs systems. Most farmers are small producers
E. encourage engaged in agricultural activities in areas of
55. A. highlighting B. highlighted widely (64) potential. Experience has shown

55
that easy access to financial services at E. branch
affordable cost (65) affects productivity, asset 67. A. truth B. aim
formation and income and food security of the C. goals D. founder
rural poor. The major concern of the E. course
Governmenttherefore ,is to bring all the farmer 68. A. make B. supply
households within the banking (66) and C. enable D. reach
promote complete financial inclusion, The E. focus
Government has initiated several policy 69. A. gives B. keeps
measures in improve the accessibility of C. set D. always
farmers to institutional sources of credit. The E. lays
(67) of these policies has been progressive 70. A. augmenting B. sending
institutionalisation for providing timely and C. submitting D. receiving
adequate credit support to all farmers in order E. limiting
to (68) them to adopt modern technology and
improved agricultural practices for increasing
agricultural production and productivity. The
policy (69) emphasis on (70) credit flow at the
ground level.

61. A. integral B. centre


C. dominant D. proven
E. highest
62. A. view B. importance
C. urgency D. source
E. choice
63. A. isolated B. apportioned
C. abject D. continuous
E. profitable
64. A. justified B. true
C. most D. varying
E. catering
65. A. not B. seriously
C. must D. positively
E. patiently
66. A. loans B. fold
C. premises D. area

56
Error Indentification

For the following questions, choose the underlined part of the sentence that contains a grammatical error.
If there are no errors, choose answer E.

1. We knew Lawrence must of missed the appointment because train service was disrupted for
A B C
three hours this morning. No error.
D E
2. Every year, a few committed citizens exceeds our expectations and work tirelessly to improve
A B C D
our community programs in significant ways. No error.
E
3. Each of the employees have had a half-hour evaluation meeting with his or her supervisor.
A B C D
No error.
E
4. Here are one of the three keys you will need to unlock the office door tomorrow. No error.
A B C D E
5. Soon after Donovan left to walk to work, he realized that he would forget his umbrella. No error.
A B C D E
6. Someone from the garage phoned to say, that the car had been fixed and asking if we would pick
A B C D
it up by 5:00. No error.
E
7. In 1963, Betty Friedan’s exposé of domesticity, The Feminine Mystique became an immediate
A B C
bestseller and creating a national sensation. No error.
D E
8. The staff at the university library deserve recognition for helping to locate the
A B C
many sources needed for the successful completion of my doctoral dissertation. No error.
D E
9. Homesteaders on the Great Plains had to build homes, find water in a semiarid land,
A B C

57
and to learn to understand the blessings of the environment. No error.
D E
10. During the winter season, homeowners should change their disposable furnace filters at least
A B
once a month; a dirty filter reduce furnace efficiency. No error.
C D E
11. The chief executive officer and the chairman of the board agrees that the new benefit package
A B
should include a dental health plan as well as eye care. No error.
C D E
12. Watching the film. I begun to ask myself why I cared about these characters when I felt such an
A B C D
intense unease. No error.
E

58
Replace the underlined words with the phrase B. flood control, generating electric
the best completes the sentence. If the sentence power, and for soil conservation.
is correct as is, choose A. C. controlling floods, generating electric
power, and soil conservation.
13. The words Equal Justice Under Law is D. flood control, the generation of
carved above the main entrance to the electric power, and soil conservation.
Supreme Court. E. flood control, for the generation of
A. is carved electric
B. carved power, and conserving the soil.
C. has been carved
D. are carved 16. According to traditional
E. been carved Chinesevmedicine, people with healthy
livers are said to be calm and that they
14. In classical economic theory, the possess unerring judgment.
relationship between supply and A. are said to be calm and that they
demand determines the price of a possess
commodity. B. are said to be calm and to possess
A. between supply and demand C. said to be calm and possessing
determines D. have said to be calm and to possess
B. among supply and demand E. are said to be calm and possessive of
determines
C. among supply and demand 17. When the phone is ringing, Jacoby had
determine been writing in his journal.
D. between supply and demand A. is ringing, Jacoby had been writing
determine B. rings, Jacoby was writing
E. with supply and demand determine C. rang, Jacoby was writing
D. had rung, Jacoby was writing
15. A corporation created by the federal E. rang, Jacoby will be writing
government during the Great
Depression, the Tennessee Valley 18. To determine the speed of automobiles,
Authority (TVA) is responsible for flood radar is often used by the state police.
control, must generate electric power, A. To determine the speed of
and soil consevation. automobiles, radar is often used by
A. flood control, must generate electric the state police.
power, and soil conservation.

59
B. To determine the speed of C. you submit it
automobiles, it is often necessary for D. we will submit it
the state police to use radar. E. we submitted it
C. In determining the speed of
automobiles, the use of radar by state 20. I have a cross-training exercise
police is often employed. program:I swim laps, play tennis, the
D. To dctcrmincthcspccd of weight machines, and bicyclc riding.
automobiles, thc state police often A. I swim laps, play tennis, the weight
use radar. machines, and bicycle riding.
E. Radar by state police in determining B. I swim laps, play tennis, lift weights,
the speed of automobiles is often and ride a bicycle.
used. C. I swim laps, play tennis, I lift weights,
and bicycle riding is a change.
19. Everyone signed the petition before D. swimming laps, tcnnis, lifting wcights,
submitting to the city council. and the bicycle.
A. submitting E. swim laps, play tennis, lifting weights,
B. one submits it and riding a bicycle.

60
Structure Section offered at the university, just likemany
Time: 18 Minutes
23 Questions others classes that have lowattendance in
B C
Directions: This section measures your ability to spite of their importance,at least several
recognize language appropriate for D
standardwritten English. One type of question classes are alwaysavailable.
consists of incomplete sentences, with a blank 4. E Coli has proven to be __________most
showingwhere information is to be filled in. dangerous bacteria that can beacquired from
Choose the word or phrase that most correctly food and water, even indeveloped countries.
completesthe sentence. A second type of A. one of the
question consists of sentences with four B. one of
underlined words orphrases. For each sentence, C. one
choose the one underlined word or phrase that D. of one
is incorrect in standardwritten English. Mark
the answer in your book or on a separate piece 5. The death toll would __________ muchhigher if
of paper. immediate action had not beentaken.
A. probably being
1. Although a number of voters has casttheir B. probably be
A C. probably been
ballots in temporarilyended the election D. be probable
B
because ofathe city election,the 6. For years, this varsity athleteshave been
C A B
supervisorof elections malfunction in the known throughout thecountry for their
D C
voting mechanism tremendousabilities.
2. Neither Professor Johnson nor anyother D
faculty member __________ toapply for the 7. A fire in the __________ building couldbe a
dean’s position problem for firefighters.
A. Intend A. ninety-story-tall
B. Intends B. ninety-tall-story
C. are intending C. ninety-stories-tall
D. has intend D. ninety stories
3. While this is not the most popularcourse
A

61
8. The company had been operate in anold 13. __________ better, the team wouldhave been
A able to defeat the opponent.
warehouse since its inception, whenit built A. If it prepares
B B. If prepares
a huge, efficient, and modernoffice building C. Preparing
C D D. Had it prepared
9. Their office consisted of three
rooms,__________ was used as a 14. The news of the decision to invadewith
conferenceroom. A B
A. larger of which armed forces were not well received bythe
B. the largest of which C D
C. the largest of them citizens.
D. largest
15. Nobody knows why __________postponed until
10. Before administering the exam, theproctor next week.
A A. the meeting
required that the students taketheir seats B. was the meeting
B C. did the meeting
and removing all items fromtheir D. the meeting was
C D
workplaces. 16. Air traffic controllers must usea formof
11. In the past six months, the company A B
hasalready received twice __________ ingross communication that is universal
revenues as it earned in the entirepreceding C
year. understood because a pilot’s
A. as much
B. more understanding of instructions is critical.
C. as many D
D. as more 17. The curriculum at the public school isas
good __________ of any privateschool.
12. Some people enjoy preparingtheir own A. or better than
A B B. as or better that
meals whileanother would rather eatout C. as or better than that
C D D. as or better than those
regularly. 18. Hurricanes hardly neverreach the east
A B

62
coast of Florida, but some that havewere D. developing
C 21. With so many choices of wireless
extremelyhazardous. A B
D technology available, it is oftendifficulty
19. Children raisedin foster homes C
A to determine which offers thebest value
requirement special attention toovercome D
B C and quality.
the feelings ofabandonment and 22. Entering the country in car may cause
D A B C
isolation differenttreatment by customs officials
D
20. Being a private university, __________a well- than entering by way of masstransportation.
organized charitable givingprogram in order 23. The greater the number of bacteria attacking
to offer a sufficientnumber of quality courses the system, __________.
andactivities. A. the sooner treatment must be begun
A. development of B. sooner must begin treatment
B. it developed C. begin treatment as soon as possible
C. develop D. must begin treatment sooner

63
Structure Section
Time: 20 Minutes
25 Questions

Directions: This section measures your ability to recognize language appropriate for standardwritten
English. One type of question consists of incomplete sentences, with a blank showingwhere information
is to be filled in. Choose the word or phrase that most correctly completesthe sentence. A second type of
question consists of sentences with four underlined words or phrases. For each sentence, choose the
one underlined word or phrase that is incorrect in standard written English. Mark the answer in your
book or on a separate piece of paper.
1. A congressional committee has C. with hopes of
beenappointed to study a new D. hoping to
procedure__________ to eliminate some 4. Swimming is a beneficial exercise,__________
costlyexpenditures. aerobic activity and uses anumber of
A. that is expected muscle groups.
B. what is expected A. not only because it provides
C. which expects B. because it both provides
D. that expected C. for provision
2. Some professors enjoy writing articles D. as result of providing
A 5. Tests have been performed to determine
and performing research, while anothers A
B C whetherstudying TOEFL questions will
would be more content to devote alltheir B C
D help students rise their test scores.
time to teaching. D
3. Some people send job applications 6. The professor instructed the
evenwhen they are reasonably happy in students__________ the essay
theirjobs, __________ improving withoutpreparing an outline first.
theirposition. A. to not write
A. with hoping to B. not to write
B. hoping that C. do not write
D. to no write
7. It is not clear when __________,although there
are many differenttheories.
A. dinosaurs becoming extinct

64
B. dinosaurs extinction B. had let the photographers to enterthe
C. dinosaurs became extinct building
D. did dinosaurs become extinct C. permitting the photographers enterthe
8. The professor decided to allow the building
A D. the photographers let into thebuilding
students to take the examination asecond 13. The committee members resented__________
B C of the meeting.
time because the low scores. A. the president that he did not tellthem
D B. the president not to inform them
9. If the driver’s own car __________ damaged, the C. the president’s not informing them
favorite probably wouldhave won the race. D. that the president had failedinforming
A. had not been themselves
B. not 14. __________ did Arthur realize thatthere was
C. no had been danger.
D. has no be A. Upon entering the store
10. Having withdrawn from the race, B. When he entered the store
A C. After he had entered the store
thecandidate decided supporting D. Only after entering the store
B 15. The congressman, accompanied by
hisopponent despite the A
C secret service agents and aides, are
opponent’srepresenting the other B
D preparingto enter the convention hall
political party. C
within the nextfew minutes.
11. The soldiers were unable to determinewhere D
__________. 16. Because the torrential rains that
A. the jeep had been left A B
B. had been leave the jeep haddevastated
C. had the jeep been left C
D. had the jeep left the area, the governor sentthe National
12. The manager was angry becausesomebody Guard to assist in theclean-up operation.
_________. D
A. had allowed the photographers toenter 17. Lack of sanitation in restaurants are a
the building A B
major cause of disease in some areas of

65
C D A B
the country. proposed as a solution to the famine
18. Had the committee members considered C D
A inmany underdeveloped countries.
the alternatives more carefully, they 22. Because the residents had worked so
B A B
would have realized that the second was diligentto renovate the old building,
C C D
better as the first. themanager had a party.
D 23. John’s wisdom teeth were troubling him,
19. Malnutrition is a major cause ofdeath A
A so he went to a dental surgeonto see
in those countries where thecultivation B
B abouthaving them pull.
of rice have been impededby recurrent C D
C D 24. Hardly __________ the office when herealized
drought. that he had forgotten his wallet.
20. The decision to withdrawall support A. he had entered
A B B. had entered
from the activities of the athletesare C. entered
C D. had he entered
causing an uproar among theathletes’ 25. Suzy had better to change her study
D A B
fans. habits if she hopes to be admittedto a
21. Underutilized species of fish has been C D
good university

66
ACEPT
Latihan Reading Comprehension

SECTION 7
Time — 15 minutes
13 Questions

Directions: For each question in this section, select the best answer from among the choices given and
fill in the correspondingoval on the answer sheet.
The two passages below are followed by questions based on their content and on the relationship
between the two passages. Answer the questions on the basis of what is stated or implied in the
passages and in any introductory material that may be provided.

Questions 1-13 are based on the following years of life lies in the pace at which the child is
passages. growing and learning. In no other period do
Passage 1 is adapted from a 1994 report by a such profoundchangesoccur so rapidly: the
major U.S.educational task force. Passage 2 is newborn grows from a completely dependent
from a 1999 bookwritten by a consultant on human being into one who walks, talks,
educational issues. plays,and explores. Babies raised by caring,
15 attentive adults insafe, predictable
Passage 1 environments are better learners than
The first three years of life appear to be a those raised with less attention in less secure
crucial starting point—a period particularly settings.Unfortunately, in contrast to all the
sensitive to the protective mechanisms of other leadingindustrialized nations, the United
parental and family support. For States fails to give parents time to be with their
millennia,parents have recognized the 20 newborns, it fails to ensure pre- and postnatal
5 newborn’s basic need forsafety, nourishment, health care for mothers and infants, and it fails
warmth, and nurturing. Now sciencehas added to provide adequate child care. The result is
stunning revelations about human significant losses in the quality of its future
developmentfrom birth to age three, confirming workforce,
that parents and otheradult caregivers play a citizenry, and parents. There are, of course,
critical role in influencing thechild’s reasons otherthan economic ones for protecting
development. The importance of the first three 2555young children andtheir families. Children need
to be treasured for theirown sake, not merely
for what they do for the labormarket when they
are grown. But the issues of “humancapital”—

67
the combined skills, knowledge, and ideasof a areview of child development and neuroscience
nation’s people—are real. issues. Charles Nelson and Floyd Bloom discuss
America’sbusiness and political leaders some genuinelynew findings in neuroscience—
are understandably worried about the nation’s what happens in the brainwhen adults learn

30 children and itseducational system. Their new motor skills and the rapidity with which
concern is well founded,but school reform alone the adult brain can recorganize after loss of
is not the answer. Any effortto strengthen the sensoryinput from an amputated limb. The new
workforce must begin with the family, 60findings thatNelson and Bloom allude to
a key factor in the development of human suggest that the brain retainsits ability to
35 capital. Bysupporting families during the child’s reorganize itself in response to experience
earliest years,society ensures that children will orinjury throughout life. They conclude, “It may
enter school readyto learn and will be ready, in be usefulto question the simplistic view that the
time, to enter the workforce and be good 65brain becomes unbendable and increasingly
parents. It is time to sound—and difficult to modify beyond the first few years of
40 answer—the alarm about the neglect of our life. Although clearly much of
nation’syoung children and their families. All braindevelopment occurs through the first
Americans musttake responsibility for years of life, the brainis far from set in its
reversing this trend. As the risksto our children trajectory, even at the completion of
intensify, so must our determination toenact adolescence.” If so, we should be wary of claims
family-centered programs and policies to 70that parents have only a single, biologically
45 ensureall of our youngest children the decent delimited, oncein-a-lifetime opportunity to help
start that they their children build betterbrains.
deserve. Some might ask why we should care about the
75scientificaccuracy of a view put forth by those
Passage 2 who want to helpchildren: isn’t any argument
Much early childhood literature suggests that leading to improved opportunities and
the firstthree years of life is the critical period outcomes for children a good argument?
85 for brain development. It’s a time when the Manywell-intentioned early childhood
50 young brain’s learning powerisalmost limitless. advocates do take thisposition. It’s the hard-
After this period, as child psychiatristFelton nosed but often realistic view that everyone
Earls remarks, “A kind of irreversibility sets knows that policy arguments are merely
in.There is this shaping process that goes on 80exercisesin political rhetoric. Helping society’s
early, and thenat the end of this process, you children is a worthyaim. But if we want to take
have essentially designeda brain that probably the science seriously, then wehave to care if we
is not going to change very muchmore.” are acting on a science-based agendaor a myth.
Neuroscientists see it a little differently. In What a science-based policy argument should

68

90
dois add some evidence, beyond our own B. United States Congress is unlikely to
prejudices andideological tastes, for what the legislate amandate for parental leave
preferable policy might be.What the science can from work
add to the policy debate are insightsabout C. health care issues affecting the United
leverage points that we could most effectively States havenot yet been addressed by
exploitto reach our goal. If the science is wrong, other nations
then we are trying to achieve our policy goals D. long-term economic effects of deficient
by pushing the wrongbuttons. child careare insignificant when
compared to each child’sindividual
1. The first sentence of Passage 1 (lines 1-3) comfort
functionsprimarily as a. . . . E. political agenda of policymakers in the
A. statement that its author later UnitedStates exaggerates the need to
undermines plan for a futureworkforce
B. foundation for a particular argument 4. The use of quotation marks in lines 27-28
C. facetious paraphrase of a well-known primarilyserves to. . . .
claim A. illustrate an inappropriate phrase
D. disclosure of a personal bias B. introduce a note of irony
E. celebration of early childhood C. refer to an archaic notion
2. Lines 3-9 (“For millennia . . . development”) D. highlight an unusual term
draw aparallel between. . . E. challenge an established concept
A. traditional practices and contemporary 5. The author of Passage 1 implies that
critiques attemptingto strengthen the workforce by
B. basic human needs and intellectual reinforcing theeducational system is. .
endeavors A. necessary but not sufficient
C. widespread beliefs and scientific B. pragmatic but not idealistic
findings C. feasible but not probable
D. parental anxieties and developmental D. possible but not important
advances E. overwhelming but not impossible
E. experimental hypotheses and proven 6. The quotation from Felton Earls in lines
theories 51-55 servesprimarily to. . . .
3. In the second paragraph of Passage 1 (lines A. voice an incontrovertible fact
17-29), theauthor implies that the. . . . B. challenge an atypical claim
A. future workforce of other major C. rectify an unjust misconception
industrializedcountries will be better D. express a widely held point of view
prepared than that of theUnited States E. support the argument made by the
author ofPassage 2

69
7. The author of Passage 2 refers to 11. The author of Passage 1 and Nelson and
“Neuroscientists”(line 55) specifically in Bloom in Passage 2 all agree that. . . .
order to. . . . A. adults are able to learn new behaviors
A. criticize the notion of scientific rapidlyand successfully
infallibility B. the human brain is able to withstand a
B. cite further evidence in support of greatdeal of trauma
Earls’remarks C. children are able to acquire motor
C. provide a historical overview of an skills moreeasily than language skills
intriguingnew field D. much brain development takes place
D. argue against a particular public policy duringthe early years of childhood
E. pose a challenge to a common belief E. the brain becomes increasingly
8. In line 69, “set” most nearly means. . . . difficult tomodify after the first three
A. ready to go years of life
B. agreed upon 12. The authors of both passages agree on the
C. prepared merits of. . . .
D. arranged A. adults continuing to acquire new skills
E. fixed B. society supporting children’s
9. Lines 74-77 (“Some . . . argument?”) development
primarily serve to. . . . C. parents learning from as well as
A. illustrate an unlikely misunderstanding teaching their
B. discourage a possible investigation children
C. anticipate a potential objection D. scientists setting realistic and fiscally
D. reveal a conflict of interest responsiblegoals
E. reject a scientific claim E. the United States developing a strong
10. The author of Passage 2 suggests that the and skilledworkforce
need for“family-centered programs and 13. The author of Passage 2 would most likely
policies” (line 44)argued for in Passage 1 characterizethe author of Passage 1 as. . . .
is. . . . A. logical but arrogant
A. acute but difficult to satisfy fully B. well meaning but inaccurate
B. pressing but politically sensitive C. persuasive but patronizing
C. possibly real but scientifically D. precise but impersonal
unproven E. well intentioned but abstruse
D. widely recognized but contrary to
economicinterests
E. often cited but rarely meant sincerel

70
Reading Test
65 MINUTES, 52 QUESTIONS

Turn to Section 1 of your answer sheet to answer the questions in this section.

DIRECTIONS
Each passage or pair of passages below is followed by a number of questions. After reading each
passage or pair, choose the best answer to each question based on what is stated or implied in the
passage or passages and in any accompanying graphics (such as a table or graph).

Questions 1-10 are based on the following


passage.
This passage is from Lydia Minatoya, The charcoal brazier. A thick quilt spread over the
Strangeness of Beauty. ©1999 by Lydia 15 sides of the table so their legs were tucked
Minatoya. The setting is Japan in1920. Chie and inside with the heat.
her daughter Naomi are members of theHouse “Who is it at this hour, in this weather?”
of Fuji, a noble family. Chie questioned as she picked the name card off
the maid’s lacquer tray.
Akira came directly, breaking all “Shinoda, Akira. Kobe Dental College,”
tradition. Was that it? Had he followed form— 20 she read. Naomi recognized the name. Chie

had he asked his heard a soft intake of air.


mother to speak to his father to approach a go- “I think you should go,” said Naomi.
between—would Chie have been more Akira was waiting in the entry. He was
receptive? in his early twenties, slim and serious, wearing
5 He came on a winter’s eve. He pounded 25 the blackmilitary-style uniform of a student. As
on the door while a cold rain beat on the he bowed—his hands hanging straight down, a
shuttered veranda, so at first Chie thought him black cap in one, a yellow oil-paper umbrella in
only the wind. The maid knew better. Chie the other—Chie glanced beyond him. In the
heard her soft scuttling footsteps, the creak of glistening surface of the courtyard’s rain-
10 the door. Then the maid brought a calling card 30 drenched paving stones, she saw his reflection

to the drawing room, for Chie. like a dark double.


Chie was reluctant to go to her guest; 70 “Madame,” said Akira, “forgive my
35 perhaps she was feeling too cozy. She and disruption,but I come with a matter of urgency.”
Naomi were reading at a low table set atop a His voice was soft, refined. He straightened and
stole a deferential peek at her face. In the dim

71
light his eyes shone with sincerity. Chie felt Eager to make his point, he’d been looking her
herself starting to like him. fullin the face. Abruptly, his voice turned gentle.
“Come inside, get out of this nasty night. Surely “I seeI’ve startled you. My humble apologies. I’ll
your business can wait for a moment or two.” “I take nomore of your evening. My address is on
don’t want to trouble you. Normally I would my card. Ifyou don’t wish to contact me, I’ll
40 approach you more properly but I’ve received 75reapproach you intwo weeks’ time. Until then,
word of a position. I’ve an opportunity to go to good night.”He bowed and left. Taking her ease,
America, as dentist for Seattle’s Japanese with effortlessgrace, like a cat making off with a
community.” “Congratulations,” Chie said with fish.“Mother?” Chie heard Naomi’s low voice
amusement. “That is an opportunity, I’m sure. andturned from the door. “He has asked
45 But how am I involved?” 80 you?”The sight of Naomi’s clear eyes, her dark
Even noting Naomi’s breathless reaction to the browsgave Chie strength. Maybe his hopes
name card, Chie had no idea. Akira’s message, werepreposterous.
delivered like a formal speech, filled her with “Where did you meet such a fellow? Imagine!
maternal amusement. You know how children 85 He thinks he can marry the Fuji heir and take
50 speak so earnestly, so hurriedly, so endearingly her to America all in the snap of his fingers!”
about things that have no importance in an Chie waited for Naomi’s ripe laughter. Naomi
adult’s mind? That’s how she viewed him, as a was silent. She stood a full half minute looking
child. It was how she viewed Naomi. Even straight into Chie’s eyes. Finally, she spoke. “I
though Naomi was eighteen and training 90met him at my literary meeting.”
endlessly in the arts needed to make a good Naomi turned to go back into the house, then
55 marriage, Chie had made no effort to find her a stopped.
husband. Akira blushed. “Mother.”
“Depending on your response, I may “Yes?”
stay inJapan. I’ve come to ask for Naomi’s 95“I mean to have him.”
60 hand.” Suddenly Chie felt the dampness of the * a man who marries a woman of higher status
night. “Does Naomi know anything of your . . . and takes her family’s name
ambitions?”“We have an understanding. Please
don’t judgemy candidacy by the unseemliness 1. Which choice best describes what happens
65 of this proposal. Iask directly because the use of in the passage?
a go-between takesmuch time. Either method A. One character argues with another
comes down to the samething: a matter of character who intrudes on her home.
parental approval. If you give yourconsent, I B. One character receives a surprising
become Naomi’s yoshi.* We’ll live in theHouse request from another character.
of Fuji. Without your consent, I must go C. One character reminisces about choices
toAmerica, to secure a new home for my bride.” she has made over the years.

72
D. One character criticizes another B. objectivity but not complete
character for pursuing an unexpected impartiality.
course of action. C. amusement but not mocking
2. Which choice best describes the disparagement.
developmental pattern of the passage? D. respect but not utter deference.
A. A careful analysis of a traditional 7. The main purpose of the first paragraph is
practice to
B. A detailed depiction of a meaningful A. describe a culture.
encounter B. criticize a tradition.
C. A definitive response to a series of C. question a suggestion.
questions D. analyze a reaction.
D. A cheerful recounting of an amusing 8. As used in line 2, “form” most nearly means
anecdote A. appearance.
3. As used in line 1 and line 65, “directly” B. custom.
most nearly means C. structure.
A. frankly. D. nature.
B. confidently. 9. Why does Akira say his meeting with Chie
C. without mediation. is “a matter of urgency” (line 32)?
D. with precision. A. He fears that his own parents will
4. Which reaction does Akira most fear from disapprove of Naomi.
Chie? B. He worries that Naomi will reject him
A. She will consider and marry someone else.
B. She will mistake his earnestness for C. He has been offered an attractive job in
immaturity. another country.
C. She will consider his unscheduled visit D. He knows that Chie is unaware of his
an imposition. feelings for Naomi.
D. She will underestimate the sincerity of 10. Which choice provides the best evidence
his emotions. for the answer to the previous question?
5. Which choice provides the best evidence A. Line 39 (“I don’t . . . you”)
for the answer to the previous question? B. Lines 39-42 (“Normally . . .
A. Line 33 (“His voice . . . refined”) community”)
B. Lines 49-51 (“You . . . mind”) C. Lines 58-59 (“Depending . . . Japan”)
C. Lines 63-64 (“Please . . . proposal”) D. Lines 72-73 (“I see . . . you”)
D. Lines 71-72 (“Eager . . . face”)
6. In the passage, Akira addresses Chie with
A. affection but not genuine love.

73
Questions 11-21 are based on the following 25 ‘‘the deadweight loss of Christmas”). To wit,
passage and supplementary material. givers
This passage is adapted from Francis J. Flynn are likely to spend $100 to purchase a gift that
and Gabrielle S. Adams, "Money Can't Buy Love: receivers would spend only $80 to buy
Asymmetric Beliefs about Gift Price and themselves.This ‘‘deadweight loss” suggests
Feelings of Appreciation." ©2008 by Elsevier that gift-givers arenot very good at predicting
Inc. 30 what gifts others will appreciate. That in itself is
not surprising to social psychologists. Research
Every day, millions of shoppers hit the has found that people oftenstruggle to take
stores in full force—both online and on foot account of others’ perspective their insights are
searching frantically for the perfect gift. Last subject to egocentrism, social projection, and
year, Americansspent over $30 billion at retail multiple attribution errors.
5 stores in the month ofDecember alone. Aside 35 What is surprising is that gift-givers
from purchasing holiday gifts, most people have considerable experience acting as both
regularly buy presents for other occasions gift-givers and gift-recipients, but nevertheless
throughout the year, including weddings, tend to overspend each time they set out to
birthdays, anniversaries, graduations, and baby purchase a meaningful gift. In the present
showers. This frequent experience of gift-giving research, we propose a unique psychological
10 can engender ambivalent feelings in gift-givers. 40 explanation for this overspendingproblem—i.e.,
Many relish the opportunity to buy presents that gift-givers equate how much they spend
because gift-giving offers a powerful means to with how much recipients will appreciate
build stronger bonds with one’s closest peers. thegift (the more expensive the gift, the
At the same time, many dread the thought of stronger a gift-recipient’s feelings of
buying gifts; they worry that their purchases appreciation). Although a link between gift
15 will disappoint rather than delight the intended 45price and feelings of appreciationmight seem
recipients. intuitive to gift-givers, such an assumption may
Anthropologists describe gift-giving as a be unfounded. Indeed, we propose that gift-
positive social process, serving various political, recipients will be less inclined to base their
religious, and psychological functions. feelings of appreciation on the magnitude of a
20 Economists, however, offer a less favorable giftthan givers assume.
view. According to Waldfogel (1993), gift-giving 50 Why do gift-givers assume that gift price
represents an objective waste of resources. is closely linked to gift-recipients’ feelings of
People buy gifts that recipients would not appreciation? Perhaps givers believe that bigger
choose to buy on their own, or at least not (i.e., more expensive) gifts convey stronger
spend as much money to purchase (a 55 signals ofthoughtfulness and consideration.

phenomenon referred to as According to Camerer (1988) and others, gift-

74
giving represents asymbolic ritual, whereby
gift-givers attempt to signaltheir positive
attitudes toward the intended recipientand
their willingness to invest resources in a
futurerelationship. In this sense, gift-givers may
60 be motivated to spend more money on a gift in
order tosend a “stronger signal” to their
intended recipient.As for gift-recipients, they
may not construe smallerand larger gifts as
representing smaller and largersignals of
thoughtfulness and consideration. The notion of
65 gift-givers and gift-recipients being unable to
5 11. The authors most likely use the examples in
account for the other party’s perspective seems
lines 1-9 of the passage (“Every... showers”)
puzzling because people slip in and out of these
to highlight the
roles every day, and, in some cases, multiple
A. regularity with which people shop
times in the course of the same day. Yet, despite
for gifts.
the extensive experience that people have as
70 B. recent increase in the amount of
both givers and receivers, they often struggle to
money spent on gifts.
transfer information gained from one role (e.g.,
C. anxiety gift shopping causes for
as a giver) and apply it in another,
consumers.
complementary role (e.g., as a receiver). In
D. number of special occasions
theoretical terms, people fail to utilize
involving gift-giving.
75 information about their own preferences and
12. In line 10, the word “ambivalent” most
experiences in order to produce more efficient
nearly means
outcomes in their exchange relations. In
A. unrealistic.
practical terms, people spend hundreds of
B. conflicted.
dollars each year on gifts, but somehow never
C. apprehensive.
learn to calibrate their gift expenditures
D. supportive.
according to personal insight.
13. The authors indicate that people value gift-
80
giving because they feel it
A. functions as a form of self-
expression.
B. is an inexpensive way to show
appreciation.
C. requires the gift-recipient to
reciprocate.

75
D. can serve to strengthen a C. exchange.
relationship. D. communicate.
14. Which choice provides the best evidence for 19. The authors refer to work by Camerer and
the answer to the previous question? others (line 56) in order to
A. Lines 10-13 (“Many... peers”) A. offer an explanation.
B. Lines 22-23 (“People... own”) B. introduce an argument.
C. Lines 31-32 (“Research... C. question a motive.
perspectives”) D. support a conclusion.
D. Lines 44-47 (“Although... 20. The graph following the passage offers
unfounded”) evidence that gift-givers base their
15. The “social psychologists” mentioned in predictions of how much a gift will be
paragraph 2 (lines 17-34) would likely appreciated on
describe the “deadweight loss” A. the appreciation level of the gift-
phenomenon as recipients.
A. predictable. B. the monetary value of the gift.
B. questionable. C. their own desires for the gifts they
C. disturbing. purchase.
D. unprecedented. D. their relationship with the gift-
16. The passage indicates that the assumption recipients.
made by gift-givers in lines 41-44 may be 21. The authors would likely attribute the
A. insincere. differences in gift-giver and recipient mean
B. unreasonable. appreciation as represented in the graph to
C. incorrect. A. an inability to shift perspective.
D. Substantiated B. an increasingly materialistic culture.
17. Which choice provides the best evidence for C. a growing opposition to gift-giving.
the answer to the previous question? D. a misunderstanding of intentions
A. Lines 53-55 (“Perhaps...
consideration”)
B. Lines 55-60 (“According...
relationship”)
C. Lines 63-65 (“As... consideration”)
D. Lines 75-78 (“In... relations”)
18. As it is used in line 54, “convey” most nearly
means
A. transport.
B. counteract.

76
Questions 22-31 are based on the following 20 The other biologically important feature is
passage and supplementary material. themanner in which the two chains are held
together. This is done by hydrogen bonds
This passage is adapted from J. D. Watson and F. between the bases.The bases are joined
H. C. Crick, “Genetical Implications of the together in pairs, a single basefrom one chain
Structure of Deoxyribonucleic Acid.” ©1953 by 25 being hydrogen-bonded to a singlebase from
Nature Publishing Group. Watson and Crick the other. The important point is that
deduced the structure of DNA using evidence onlycertain pairs of bases will fit into the
from Rosalind Franklin and R. G. Gosling’s X-ray structure.
crystallography diagrams of DNA and from One member of a pair must be a purine and the
Erwin Chargaff’s data on the base composition othera pyrimidine in order to bridge between
of DNA. the twochains. If a pair consisted of two
30 purines, forexample, there would not be room
The chemical formula of deoxyribonucleic acid for it.We believe that the bases will be present
(DNA) is now well established. The molecule is almostentirely in their most probable forms. If
a very long chain, the backbone of which this is true,the conditions for forming hydrogen
consists of a regular alternation of sugar and 35 bonds are morerestrictive, and the only pairs of
phosphate groups. bases possible are:adenine with thymine, and
5 To each sugar is attached a nitrogenous base, guanine with cytosine.Adenine, for example, can
which can be of four different types. Two of the occur on either chain; butwhen it does, its
possible bases—adenine and guanine—are partner on the other chain mustalways be
purines, and the other two—thymine and 40 thymine.The phosphate-sugar backbone of our
cytosine—are pyrimidines. So far as is known, model iscompletely regular, but any sequence
10 the sequence of bases along the chain is of the pairs ofbases can fit into the structure. It
irregular. The monomer unit, consisting follows that in along molecule many different
ofphosphate, sugar and base, is known as a permutations are possible, and it therefore
nucleotide. The first feature of our structure seems likely that the precise sequence of bases
which is of biological interest is that it consists 45 is the code which carries the genetical
not of one chain, but of two. These two chains information. If the actual order of the bases on
15 are both coiled around a common fiber axis. It one of the pair of chains were given, one could
has often been assumed that since there was write down the exact order of the bases on the
only one chain in the chemical formula there other one, because of the specific pairing. Thus
would only be one in the structural unit. one chain is, as it were, the complement of the
However, the density, taken with the X- 50other, and it is this feature which suggests how
rayevidence, suggests very strongly that there the deoxyribonucleic acid molecule might
are two. duplicate itself/

77
The table shows, for various organisms, the B. Lines 9-10 (“So far... irregular”)
percentage of each of the four types of C. Lines 23-25 (“The bases... other”)
nitrogenous bases in that organism’s DNA. D. Lines 27-29 (“One member... chains”)
24. In the second paragraph (lines 12-19), what do
theauthors claim to be a feature of biological
interest?
A. The chemical formula of DNA
B. The common fiber axis
C. The X-ray evidence
D. DNA consisting of two chains
25. The authors’ main purpose of including the
information about X-ray evidence and density is
to
A. establish that DNA is the molecule that
carries the genetic information.
B. present an alternate hypothesis about the

Adapted from Manju Bansal, “DNA Structure: composition of a nucleotide.

Revisiting theWatson-Crick Double Helix.” C. provide support for the authors’ claim

©2003 by Current Science about the number of chains in a molecule of


DNA.
Association,Bangalore.
D. confirm the relationship between the

22. The authors use the word “backbone” in lines density ofDNA and the known chemical

3and 39 to indicate that formula of DNA.

A. only very long chains of DNA can be taken 26. Based on the passage, the authors’ statement “If
apair consisted of two purines, for example,
froman organism with a spinal column.
B. the main structure of a chain in a DNA therewould not be room for it” (lines 29-30)

moleculeis composed of repeating units. implies that apair

C. a chain in a DNA molecule consists entirely A. of purines would be larger than the space

ofphosphate groups or of sugars. between a sugar and a phosphate group.


B. of purines would be larger than a pair
D. nitrogenous bases form the main structural
unitof DNA. consistingof a purine and a pyrimidine.

23. A student claims that nitrogenous bases C. of pyrimidines would be larger than a pair

pairrandomly with one another. Which of the ofpurines.

followingstatements in the passage contradicts D. consisting of a purine and a pyrimidine


would belarger than a pair of pyrimidines.
the student’s claim?
A. Lines 5-6 (“To each... types”)

78
27. The authors’ use of the words “exact,” “specific,” of guanine is closest to the percentage of
and“complement” in lines 47-49 in the final cytosine.
paragraphfunctions mainly to D. No, because for each given organism, the
A. confirm that the nucleotide sequences are percentage of adenine is closest to the
knownfor most molecules of DNA. percentage of guanine, and the percentage
B. counter the claim that the sequences of of cytosine is closest to the percentage of
bases thymine.
along a chain can occur in any order. 30. According to the table, which of the
C. support the claim that the phosphate- following pairs of base percentages in sea
sugarbackbone of the authors’ model is urchin DNA provides evidence in support of
completelyregular. the answer to the previous question?
D. emphasize how one chain of DNA may serve A. 17.3% and 17.7%
as atemplate to be copied during DNA B. 17.3% and 32.1%
replication. C. 17.3% and 32.8%
28. Based on the table and passage, which choice D. 17.7% and 32.8%
givesthe correct percentages of the purines in 31. Based on the table, is the percentage of
yeast DNA? adenine in each organism’s DNA the same or
A. 17.1% and 18.7% does it vary, and which statement made by
B. 7.1% and 32.9% the authors is most consistent with that
C. 18.7% and 31.3% data?
D. 31.3% and 32.9% A. The same; “Two of... pyrimidines” (lines
29. Do the data in the table support the authors’ 6-8)
proposed pairing of bases in DNA? B. The same; “The important... structure”
A. Yes, because for each given organism, the (lines 25-26)
percentage of adenine is closest to the C. It varies; “Adenine... thymine” (lines 36-
percentage of thymine, and the percentage 38)
of guanine is closest to the percentage of D. It varies; “It follows... information” (lines
cytosine. 41-45)
B. Yes, because for each given organism, the
percentage of adenine is closest to the
percentage of guanine, and the percentage
of cytosine is closest to the percentage of
thymine.
C. No, because for each given organism, the
percentage of adenine is closest to the
percentage of thymine, and the percentage

79
Questions 32-41 are based on the following curtained window watched educated men
passage. leaving the house at about nine-thirty to go to
an office, returning to the house at about six-
This passage is adapted from Virginia Woolf, thirty from an office, need look passively no
Three Guineas. ©1938 by Harcourt, Inc. Here, 30 longer. We too can leave the house, can mount
Woolf considers the situation of women in those steps, pass in and out of those doors,...
English society. make money, administer justice. . . . We who
now agitate these humble pens may in another
Close at hand is a bridge over the River century or two speak from a pulpit. Nobody will
Thames, an admirable vantage ground for us to dare contradict us then; we shall be the
make a survey. The river flows beneath; barges 35mouthpieces of the divine spirit—a solemn
pass, laden with timber, bursting with corn; thought, is it not? Who can say whether, as time
5 there on one side are the domes and spires of goes on, we may not dress in military uniform,
the city; on the other, Westminster and the with gold lace on our breasts, swords at our
Houses of Parliament. It is a place to stand on sides, and something like the old family coal-
by the hour, dreaming. But not now. Now we scuttle on our heads, save that that venerable
are pressed for time. Now we are here to 40object was never decorated with plumes of
consider facts; now we must fix our eyes upon white horsehair. You laugh—indeed the shadow
10 the procession—the procession of the sons of of the private house still makes those dresses
educated men. look a little queer. We have worn private
There they go, our brothers who have clothes so long. . . . But we have not come here
been educated at public schools and 45 to laugh, or totalk of fashions—men’s and
60 universities, mounting those steps, passing in women’s. We are here, on the bridge, to ask
15 and out of those doors, ascending those pulpits, ourselves certain questions. And they are very
preaching, teaching, administering justice, important questions; and we have very little
practising medicine, transacting business, time in which to answer them. The questions
making money. It is a solemn sight always—a that we have to ask and to answer about that
procession, like a caravanserai crossing a 50 procession during this moment of transition are

desert. . . . But now, for the past twenty years or so important that they may well change the
20 so, it is no longer a sight merely, a photograph, lives of all men and women for ever. For we
or fresco scrawled upon the walls of time, at have to ask ourselves, here and now, do we
which we can look with merely an esthetic wish to join that procession, or don’t we? On
appreciation. For there, trapesing along at the what terms shall we join that procession?
tail end of the procession, we go ourselves. And 55 Above all, where is it leading us, the procession
25 that makes a difference. We who have looked so of educated men? The moment is short; it may
long at the pageant in books, or from a last five years; ten years, or perhaps only a

80
matter of a few months longer.... But, you will B. women can have positions of influence in
object, you have no time to think; you have your English society only if they give up some of
battles to fight, your rent to pay, your bazaars to their traditional roles.
organize. That excuse shall not serve you, C. the male monopoly on power in English
Madam. As you know from your own society has had grave and continuing
experience, and there are facts that prove it, the effects.
daughters of educated men have always done D. the entry of educated women into positions
their thinking from hand to mouth; not under of power traditionally held by men will

65 green lamps at study tables in the cloisters of transform those positions.


secluded colleges. They have thought while they 34. Woolf uses the word “we” throughout the
stirred the pot, while they rocked the cradle. It passage mainly to
was thus that they won us the right to our A. reflect the growing friendliness among a
brand-new sixpence. It falls to us now to go on group of people.
70 thinking; how are we to spend that sixpence? B. advance the need for candor among a
Think we must. Let us think in offices; in group of people.
omnibuses; while we are standing in the crowd C. establish a sense of solidarity among a
watching Coronations and Lord Mayor’s Shows; group of people.
let us think . . . in the gallery of the House of D. reinforce the need for respect among a
Commons; in the Law Courts; let us think at group of people.
75 baptisms and marriages and funerals. Let us 35. According to the passage, Woolf chooses the
never cease from thinking—what is this setting of the bridge because it
“civilization” in which we find ourselves? What A. is conducive to a mood of fanciful
are these ceremonies and why should we take reflection.
part in them? What are these professions and B. provides a good view of the procession
why should we make money out of them? of the sons of educated men.
Where in short is it leading us, the procession of C. is within sight of historic episodes to
the sons of educated men? which she alludes.
80 D. is symbolic of the legacy of past and
32. The main purpose of the passage is to present sons of educated men.
A. emphasize the value of a tradition. 36. Woolf indicates that the procession she
B. stress the urgency of an issue. describes in the passage
C. highlight the severity of social divisions. A. has come to have more practical
D. question the feasibility of an undertaking influence in recent years.
33. The central claim of the passage is that B. has become a celebrated feature of
A. educated women face a decision about how English public life.
to engage with existing institutions.

81
C. includes all of the richest and most C. complex the political and social issues of
powerful men in England. the day are.
D. has become less exclusionary in its D. enjoyable the career possibilities for
membership in recent years. women are.
37. Which choice provides the best evidence for
the answer to the previous question?
A. Lines 12-17 (“There... money”)
B. Lines 17-19 (“It... desert”)
C. Lines 23-24 (“For... ourselves”)
D. Lines 30-34 (“We... pulpit”)
38. Woolf characterizes the questions in lines
53-57 (“For we... men”) as both
A. controversial and threatening.
B. weighty and unanswerable.
C. momentous and pressing.
D. provocative and mysterious.
39. Which choice provides the best evidence for the
answer to the previous question?
A. Lines 46-47 (“We... questions”)
B. B) Lines 48-49 (“And... them”)
C. C) Line 57 (“The moment... short”)
D. D) Line 62 (“That... Madam”)
40. Which choice most closely captures the
meaning of the figurative “sixpence” referred to
in lines 70 and 71?
A. Tolerance
B. Knowledge
C. Opportunity
D. Perspective
41. The range of places and occasions listed in lines
72-76 (“Let us... funerals”) mainly serves to
emphasize how
A. novel the challenge faced by women is.
B. pervasive the need for critical reflection
is.

82
Questions 42-52 are based on the following build an off-planet economy free of any bonds
passages. with Earth, in which the materials extracted and
processed from the moon and asteroids are
Passage 1 is adapted from Michael Slezak, delivered for space-based projects. In this
“Space Mining: the Next Gold Rush?” ©2013 by scenario, water mined from other worlds could
New Scientist. Passage 2 is from the editors of 30 become the most desired commodity. “In the
New Scientist, “Taming the Final Frontier.” desert, what’s worth more: a kilogram of gold
©2013 by New Scientist. or a kilogram of water?” asks Kris Zacny of
HoneyBee Robotics in New York. “Gold is
Passage 1 useless. Water will let you live.”
Follow the money and you will end up Water ice from the moon’s poles could
in space. That’s the message from a first-of-its- 35 be sent to astronauts on the International Space
kind forum on mining beyond Earth. Station for drinking or as a radiation shield.
Convened in Sydney by the Australian Splitting water into oxygen and hydrogen
5 Centre for Space Engineering Research, the makes spacecraft fuel, so ice-rich asteroids
event brought together mining companies, could become interplanetary refuelling
robotics experts, lunar scientists, and stations.Companies are eyeing the iron, silicon,
government agencies that are all working to 40 and aluminium in lunar soil and asteroids,

make space mining a reality. The forum comes which could be used in 3D printers to make
10 hot on the heels of the 2012 unveiling of two spare parts or machinery. Others want to turn
private asteroid-mining firms. Planetary space dirt into concrete for landing pads,
Resources of Washington says it will launch its 45 shelters, and roads.
first prospecting telescopes in two years, while
Deep Space Industries of Virginia hopes to be Passage 2
harvesting metals from asteroids by 2020. The motivation for deep-space travel is shifting
15 Another commercial venture that sprung up in from discovery to economics. The past year has
2012, Golden Spike of Colorado, will be offering seen a flurry of proposals aimed at bringing
trips to the moon, including to potential lunar celestial riches down to Earth. No doubt this
miners. Within a few decades, these firms may 50 will make a few billionaires even wealthier, but
be meeting earthly demands for precious we all stand to gain: the mineral bounty and
20 metals, such as platinum and gold, and the rare spin-off technologies could enrich us all. But
earth elements vital for personal electronics, before the miners start firing up their rockets,
such as yttrium and lanthanum. But like the we should pause for thought. At first glance,
gold rush pioneers who transformed the 55 space mining seems to sidestep most
western United States, the first space miners environmental concerns: there is (probably!)
won’t just enrich themselves. They also hope to no life on asteroids, and thus no habitats to

83

25
trash. But its consequences —both here on
Earth and in space—merit careful 42. In lines 9-17, the author of Passage 1 mentions
consideration. several companies primarily to
Part of this is about principles. Some A. note the technological advances that make
will argue that space’s “magnificent desolation” space mining possible.
60 is not ours to despoil, just as they argue that our B. provide evidence of the growing interest in
own planet’s poles should remain pristine. space mining.
Others will suggest that glutting ourselves on C. emphasize the large profits to be made from
space’s riches is not an acceptable alternative to space mining.
65 developing more sustainable ways of earthly D. highlight the diverse ways to carry out space
life. mining operations.
History suggests that those will be hard 43. The author of Passage 1 indicates that space
lines to hold, and it may be difficult to persuade mining could have which positive effect?
the public that such barren environments are A. It could yield materials important to Earth’s
70 worth preserving. After all, they exist in vast economy.
abundance, and even fewer people will B. It could raise the value of some precious
experience them than have walked through metal on Earth.
Antarctica’s icy landscapes. There’s also the C. It could create unanticipated technological
emerging off-world economy to consider. The innovations.
resources that are valuable in orbit and beyond D. It could change scientists’ understanding of
75 may be very different to those we prize on space resources.
Earth. Questions of their stewardship have 44. Which choice provides the best evidence for the
barely been broached—and the relevant legal answer to the previous question?
and regulatory framework is fragmentary, to A. Lines 18-22 (“Within... lanthanum”)
put it mildly. B. Lines 24-28 (“They... projects”)
Space miners, like their earthly C. Lines 29-30 (“In this... commodity”)

80 counterparts, are often reluctant to engage with D. Lines 41-44 (“Companies... machinery”)
such questions. One speaker at last week’s 45. As used in line 19, “demands” most nearly
space-mining forum in Sydney, Australia, means
concluded with a plea that regulation should be A. offers.
avoided. But miners have much to gain from a B. claims.
broad agreement on the for-profit exploitation C. inquiries.
85 of space. Without consensus, claims will be D. desires.
disputed, investments risky, and the gains made 46. What function does the discussion of water in
insecure. It is in all of our long-term interests to lines 35-40 serve in Passage 1?
seek one out.

84
A. It continues an extended comparison that 50. The author of Passage 2 would most likely
begins in the previous paragraph. respond to the discussion of the future of
B. It provides an unexpected answer to a space mining in lines 18-28, Passage 1, by
question raised in the previous paragraph. claiming that such a future
C. It offers hypothetical examples supporting a A. is inconsistent with the sustainable use
claim made in the previous paragraph. of space
D. It examines possible outcomes of a proposal resources.
put forth in the previous paragraph. B. will be difficult to bring about in the
47. The central claim of Passage 2 is that space absence of
mining has positive potential but regulations.
A. it will end up encouraging humanity’s reckless C. cannot be attained without technologies
treatment of the environment. that do
B. its effects should be thoughtfully considered not yet exist.
before it becomes a reality. D. seems certain to affect Earth’s economy
C. such potential may not include replenishing in a
key resources that are disappearing on Earth. negative way
D. experts disagree about the commercial 51. Which choice provides the best evidence
viability of the discoveries it could yield. for the answer to the previous question?
48. As used in line 68, “hold” most nearly means A. Lines 60-63 (“Some... pristine”)
A. maintain. B. Lines 74-76 (“The resources... Earth”)
B. grip. C. Lines 81-83 (“One... avoided”)
C. restrain. D. Lines 85-87 (“Without... insecure”)
D. Withstand 52. Which point about the resources that will
49. Which statement best describes the relationship be highly valued in space is implicit in
between the passages? Passage 1 and explicit in Passage 2?
A. Passage 2 refutes the central claim advanced A. They may be different resources from
in those that are valuable on Earth.
Passage 1. B. They will be valuable only if they can be
B. Passage 2 illustrates the phenomenon harvested cheaply.
described C. They are likely to be primarily precious
in more general terms in Passage 1. metals and rare earth elements.
C. Passage 2 argues against the practicality of D. They may increase in value as those
the same resources become rare on Earth.
proposals put forth in Passage 1.
D. Passage 2 expresses reservations about
developments discussed in Passage 1.

85
ACEPT (1) Caribbean cuisine is a fusion of Spanish,
French, African, Amerindian, and Indian cuisine.
Latihan Composing Skills (2) Many people travel to the Caribbean to
enjoy the beautiful beaches and warm weather.

For each of the following paragraphs, choose (3) A typical dish and one increasingly common

the sentence that does NOT belong. outside of the arca is "jerk" seasoned meats,
commonly chicken. (4) Other popular dishes

(1) The cassowary, a solitary, meat-cating include curried goat and a soup-like dish called

creature who makes its home deep in the callaloo.

jungles of New Guinea, hardly seems like a bird 3. A. Sentence 1

at all. (2) It is enormous, weighing up to 190 B. Sentence 2

pounds. (3) Its plumage is more like hair than C. Sentence 3

feathers; its song is a deep, men- acing rumble; D. Sentence 4

and it has lost the capability of flight. (4)


Human beings have long been fascinated by (1) Ratatouille is a dish that has grown in

birds, particularly by their ability to fly. popularity over the last few years. (2) It fea-

1. A. Sentence 1 tures eggplant, zucchini, tomato, peppers, and

B. Sentence 2 garlic, chopped, mixed together, and cooked

C. Sentence 3 slowly over low heat. (3) Zucchini is a summer

D. Sentence 4 squash and has a smooth, dark green skin. (4)


As the vegetables cook slowly, they make their

(1) Story-telling should speak first to the heart own broth, which may be extended with a little

and only second to the intellect. (2) It should, in tomato paste.

Isaac Bashevis Singer's words, "be both clear 4. A. Sentence 1

and profound," and it should also entertain. (3) B. Sentence 2

Many fine writing programs have sprung up C. Sentence 3

across the United States. (4) The new writer D. Sentence 4

should avoid creating pieces that are


deliberately obscure and impossible to (1) An odd behavior associated with sleep and

understand except by a small, clite group of dreaming is somnambulism, commonly known

other writers. as sleepwalking. (2) Sleepwalkers suffer from a

2. A. Sentence 1 malfunction in a brain mech- anism that

B. Sentence 2 monitors the transition from REM to non-REM

C. Sentence 3 sleep. (3) REM sleep is vitally important to

D. Sentence 4 psychological well- being. (4) Sleepwalking


episodes diminish with age and usually cause

86
no serious harm-the worst thing that could make use of the solstices in their rites. (3) The
happen would be a fall down the stairs. first day of summer is called summer solstice
5. A. Sentence 1 and is also the longest day of the year. (4)
B. Sentence 2 However, June 21 marks the beginning of
C. Sentence 3 winter in the southern hemisphere, when that
D. Sentence 4 hemisphere is tilted away from the sun.
8. A. Sentence 1
(1) Lyme discase is sometimes called the "great B. Sentence 2
imitator" because its many symp- toms mimic C. Sentence 3
those of other illnesses. (2) When treated, this D. Sentence 4
disease usually pres- ents few or no lingering
effects. (3) Left untreated, it can be extremely (1) People are quick to blame the weather- man
debilitating and sometimes fatal. (4) One if it rains on their parade! (2) The American
should be very careful when returning from a Meteorological Society defines a meteorologist
trek in the woods to check for deer ticks. as a person "who uses scien- tific principles to
6. A. Sentence 1 explain, understand, observe, or forecast the
B. Sentence 2 earth's atmospheric phenomena and/or how
C. Sentence 3 the atmosphere affects the carth and life on the
D. Sentence 4 planet." (3) Many meteorologists have degrees
in physics, chemistry, and other fields. (4) Their
(1) The harp is a musical instrument that has an work often involves teaching, weather
upright triangular frame. (2) Its strings are forecasting, atmospheric research, and other
positioned perpendicular to the sound- ing kinds of applied meteorology.
board. (3) Harps are found in Africa, Europe, 9. A. Sentence 1
North and South America, and a few parts of B. Sentence 2
Asia. (4) Its beautiful sound, which is capable of C. Sentence 3
stirring great emotion, might bring tears to your D. Sentence 4
eyes.
7. A. Sentence 1 (1) The park was empty, except for a child who
B. Sentence 2 stood just on the other side of the fence, a little
C. Sentence 3 girl about seven years old, thin and pale, with
D. Sentence 4 dark eyes and dark hair-cut short and ragged.
(2) The statistics on neg- lected children in our
(1) In the summer, the northern hemi- sphere is country probably fall short of the actual
slanted toward the sun, making the days longer numbers. (3) The child wore no coat, only a
and warmer than in winter. (2) Many religions brown, cotton skirt that was too big for her-

87
pinned at the waist with a safety pin-and a C. 2, 3, 1
soiled, long- sleeved yellow blouse with D. 3, 2, 1
rhinestone but- tons. (4) Her fingernails were
dirty and broken, the tips of her fingers bluish (1) There is no harm in putting a special treat in
with cold. your child's lunchbox from time to time. (2)
10. A. Sentence 1 Usually, healthy snacks are defined as foods
B. Sentence 2 with low sugar and fat content. (3) Some
C. Sentence 3 examples include carrot and celery sticks,
D. Sentence 4 granola bars, yogurt drinks, and string cheese.
(4) However, in general, it is a much better idea
to provide healthy snacks.
For each of the following groups of four 13. A. 2, 4, 1, 3
numbered sentences, choose the sentence order B. 1, 4, 2, 3
that would result in the best paragraph. C. 1, 2, 3, 4
D. 3, 1, 2, 4
(1) Figures have the power to mislead peo- ple.
(2) Mathematics tells us about eco- nomic (1) Additionally, once a year, the associa- tion
trends, patterns of disease, and the growth of hosts a block party with food, music, and
populations. (3) Math is good at exposing the games. (2) The association organizes
truth, but it can also perpet- uate neighborhood watch teams and liaises with
misunderstandings and untruths. the police department on issues of crime and
11. A. 1,2,3 safety. (3) The main goal of the neigh-
B. 2, 3, 1 borhood association is to help make the
C. 3, 1,2 community a safer place.
D. 3, 2, 1 14. A. 1, 2, 3
B. 3, 2, 1
(1) The reason for so many injuries and C. 2, 3, 1
fatalities is that a vehicle can generate heat of D. 3, 1, 2
up to 1,500° F. (2) Firefighters know that the
dangers of motor-vehicle fires are too often (1) Leaving us behind in a bitter cloud of
overlooked. (3) In the United States, 1 out of 5 exhaust, the bus would cough and jolt down
fires involves motor vehicles, resulting each the narrow main street of Crossland. (2) Then,
year in 600 deaths, 2,600 civilian injuries, and even before the bus got moving, she'd look
1,200 injuries to fire- fighters. away, ahead toward her real life. (3) But I
12. A. 1, 2, 3 could always imagine the way it would be
B. 1, 3, 2 once it got out on the open high- way,

88
gathered speed, and took Grandma back to a who is out sick for more than three days must
life as exotic to me as the deserts of Egypt. (4) provide a doctor's note.
When Grandma's visit was over, we'd take her 17. A. 2, 3, 1
down to the Greyhound station, watch her B. 3, 1, 2
hand her ticket to the uniformed driver, C. 3, 2, 1
disappear inside, and reappear to wave D. 1, 3, 2
goodbye-her expression obscured by the bus's
grimy window.
15. A. 4, 2, 1, 3 (1) Every spring the softball field became his
B. 4, 1, 3, 2 favorite destination, and he had taken his son,
C. 1, 3, 4, 2 Arnie, there when he was small to teach him
D. 1, 2, 3, 4 how to pitch. (2) He walked home, as usual,
through the park and, as usual, passed by the
(1) The Fifth Amendment of the U.S. softball field. (3) This memory made him feel
Constitution guarantees citizens freedom from sad and guilty. (4) Arnic hadn't been in the
double jeopardy in criminal pro- ccedings. (2) least interested in softball, and so after two or
It also means a person can- not be tried for a three lessons, he had given up the idea.
crime for which he has alrcady been 18. A. 2, 1, 4, 3
convicted; that is to say, a person convicted by B. 3, 2, 1, 4
a state court cannot be tried for the same C. 4, 3, 1, 2
offense in, for exam- ple, federal court. (3) D. 2, 3, 4, 1
Finally, a person cannot be punished more
than once for the same crime. (4) This means (1) If there are expenses incurred, com- plete
that a per- son cannot be tricd for a crime for report form 103; if there was damage to
which he has already been acquitted. equipment, complete form 107. (2) If form
16. A. 1,4, 2, 3 107 and form 103 are required, com- plete
B. 1, 2, 4, 3 form 122 also. (3) Log on to the com- puter
C. 3, 2, 1, 4 and go to the directory that contains the
D. 3, 4, 2, 1 report forms. (4) As an employee, you must
complete all paperwork.
(1) If these new policies are any indication, 19. A. 3, 2, 1, 4
employees will have much less freedom than B. 1, 3, 4, 2
they did before. (2) The handbook also states C. 2, 1, 4, 3
that employces must give at least three weeks D. 4, 3, 1, 2
notice before taking a personal day. (3) The
new employce handbook states that anyone

89
(1) In some areas, the salt is combined with It weighs less than three pounds and is hardly
calcium chloride, which is more effective in more interesting to look at than an overly ripe
below-zero temperatures and which melts ice cauliflower.
better. (2) After a snow or icefall, city streets 21. A. 1,3, 4, 2
are treated with ordinary rock salt. (3) This B. 2, 1, 4, 3
combination of salt and calcium chloride is C. 3, 1, 2, 4
also less damaging to foliage along the D. 4, 1, 2, 3
roadways. (1) Before you begin to compose a business
20. A. 2, 1, 3 letter, sit down and think about your pur- pose
B. 1,3, 2 in writing the letter. (2) Do you want to request
C. 3, 2, 1 information, order a product, regis- ter a
D. 2, 3, 1 complaint, or apply for something? (3) Always
keep your objective in mind. (4) Do some
(1) Yet the human brain is the most myste- brainstorming and gather infor- mation before
rious and complex object on Earth. (2) It has you begin writing.
created poetry and music, planned and 22. A. 4, 3, 2, 1
executed horrific wars, devised intricate B. 2, 4, 3, 1
scientific theories. (3) It thinks and dreams, C. 1, 2, 4, 3
plots and schemes, and easily holds more D. 3, 2, 1, 4
information than all the libraries on Earth. (4)

90
Wrtiting and Language Test
35 MINUTES, 44 QUESTIONS

Turn to Section 2 of your answer sheet to answer the questions in this section.

DIRECTIONS

Each passage below is accompanied by a number of questions. For some questions, you will consider
how the passage might be revised to improve the expression of ideas. For other questions, you will
consider how the passage might be edited to correct errors in sentence structure, usage, or punctuation.
A passage or a question may be accompanied by one or more graphics (such as a table or graph) that
you will consider as you make revising and editing decisions.

Some questions will direct you to an underlined portion of a passage. Other questions will direct you to
a location in a passage or ask you to think about the passage as a whole.

After reading each passage, choose the answer to each question that most effectively improves the
quality of writing in the passage or that makes the passage conform to the conventions of standard
written English. Many questions include a “NO CHANGE” option. Choose that option if you think the best
choice is to leave the relevant portion of the passage as it is.

Questions 1-11 are based on the following many methods of controlling and eliminating
passage. most environmental threats. Given these
solutions as well as the many health benefits of
Whey to Go
the food, the advantages of Greek yogurt [1]out
Greek yogurt—a strained form of do the potential drawbacks of its production.
cultured yogurt—has grown enormously in
[1] The main environmental problem
popularity in the United States since it was first
caused by the production of Greek yogurt is the
introduced in the country in the late 1980s.
creation of acid whey as a by-product. [2]
From 2011 to 2012 alone, sales of Because it requires up to four times more milk
Greek yogurt in the US increased by 50 percent. to make than conventional yogurt does, Greek
The resulting increase in Greek yogurt yogurt produces larger amounts of acid whey,
production has forced those involved in the
which is difficult to dispose of. [3] To address
business to address the detrimental effects that
the problem of disposal, farmers have found a
the yogurt-making process may be having on
number of uses for acid whey. [4] They can add
the environment. Fortunately, farmers and
it to livestock feed as a protein [2] supplement,
others in the Greek yogurt business have found
and people can make their own Greek-style

91
yogurt at home by straining regular yogurt. 1. A. NO CHANGE
[5] If it is improperly introduced into the B. defeat
environment, acid-whey runoff [3] can pollute C. outperform
waterways, depleting the oxygen content of D. outweigh
streams and rivers as it decomposes. [6] Yogurt 2. Which choice provides the most relevant
manufacturers, food [4] scientists;and detail?
government officials are also working together A. NO CHANGE
to develop additional solutions for reusing B. supplement and convert it into gas to
whey. [5] use as fuel in electricity production.
C. supplement, while sweet whey is more
desirable as a food additive for humans.
[6] Though these conservation methods can D. supplement, which provides an
be costly and time-consuming, they are well important element of their diet.
worth the effort. Nutritionists consider Greek 3. A. NO CHANGE
yogurt to be a healthy food: it is an excellent B. can pollute waterway s,
source of calcium and protein, serves [7] to be C. could have polluted waterways,
a digestive aid, and [8] it contains few calories D. has polluted waterway’s,
in its unsweetened low- and non-fat forms. 4. A. NO CHANGE
Greek yogurt is slightly lower in sugar and B. scientists: and
carbohydrates than conventional yogurt is. [9] C. scientists, and
Also, because it is more concentrated, Greek D. scientists, and,
yogurt contains slightly more protein per 5. To make this paragraph most logical,
serving, thereby helping people stay sentence 5 should be placed
A. where it is now.
B. after sentence 1.
[10] satiated for longer periods of time. These C. after sentence 2.
health benefits have prompted Greek yogurt’s D. after sentence 3.
recent surge in popularity. In fact, Greek yogurt 6. The writer is considering deleting the
can be found in an increasing number of underlined sentence. Should the writer do
products such as snack food and frozen this?
desserts. Because consumers reap the A. Yes, because it does not provide a
nutritional benefits of Greek yogurt and support transitionfrom the previous
those who make and sell [11] it, therefore paragraph.
farmersand businesses should continue finding B. Yes, because it fails to support the
safe and effective methods of producing the mainargument of the passage as
food. introduced in thefirst paragraph.

92
C. No, because it continues the Questions 12-22 are based on the following
explanation of howacid whey can be passage and supplementary material.
disposed of safely.
Dark Snow
D. No, because it sets up the argument in
theparagraph for the benefits of Greek Most of Greenland’s interior is covered
yogurt. by a thick layer of ice and compressed snow
7. A. NO CHANGE known as the Greenland Ice Sheet. The size of
B. as the ice sheet fluctuates seasonally: in summer,
C. like average daily high temperatures in Greenland
D. for can rise to slightly above 50 degrees
8. A. NO CHANGE Fahrenheit, partially melting the ice; in the
B. Containing winter, the sheet thickens as additional snow
C. Contains falls, and average daily low temperatures can
D. will contain drop[12] to as low as 20 degrees.
9. A. NO CHANGE
B. In other words,
C. Therefore, Average Daily High and Low
D. For instance, Temperatures Recorded at Nuuk Weather
10. A. NO CHANGE Station, Greenland (1961—1990)
B. Fulfilled
C. Complacent
D. Sufficient
11. A. NO CHANGE
B. it, farmers
C. it, so farmers
D. it: farmers

Adapted from WMO. ©2014 by World


Meteorological Organization.

Typically, the ice sheet begins to show evidence


of thawing in late [13] summer. This follows
several weeks of higher temperatures. [14] For
example, in the summer of 2012, virtually the
entire Greenland Ice Sheet underwent thawing

93
at or near its surface by mid-July, the earliest information. [5] The next step for Box and his
date on record. Most scientists looking for the team is to travel to Greenland to perform direct
causes of the Great Melt of 2012 have focused sampling of the ice in order to determine just
exclusively on rising temperatures. The how much the soot is contributing to the
summer of 2012 was the warmest in 170 years, melting of the ice sheet. [6] Members of the
records show. But Jason [15]Box, an associate public will be able to track his team’s
professor of geology at Ohio State believes progress—and even help fund the expedition—
that another factor added to the early through a website Box has created. [22]
[16]thaw; the “dark snow” problem.
12. Which choice most accurately and effectively
According to Box, a leading Greenland represents the information in the graph?
expert, tundra fires in 2012 from as far away as A. NO CHANGE
North America produced great amounts of soot, B. to 12 degrees Fahrenheit.
some [17]of it drifted over Greenland in giant C. to their lowest point on December 13.
plumes of smoke and then [18] fell as particles D. to 10 degrees Fahrenheit and stay there for
onto the ice sheet. Scientists have long known months.
that soot particles facilitate melting by 13. Which choice most effectively combines the two
darkening snow and ice, limiting [19]it’s ability sentences at the underlined portion?
to reflect the Sun’s rays. As Box explains, “Soot A. summer, following
is an extremely powerful light absorber. It B. summer, and this thawing follows
settles over the ice and captures the Sun’s heat.” C. summer, and such thawing follows
The result is a self-reinforcing cycle. As the ice D. summer and this evidence follows
melts, the land and water under the ice become 14. A. NO CHANGE
exposed, and since land and water are darker B. However,
than snow, the surface absorbs even more heat, C. As such,
which [20]is related to the rising D. Moreover,
temperatures. 15. A. NO CHANGE
B. Box an associate professor of geology at
[1] Box’s research is important because
Ohio State,
the fires of 2012 may not be a one-time
C. Box, an associate professor of geology
phenomenon. [2] According to scientists, rising
atOhio State,
Arctic temperatures are making northern
D. Box, an associate professor of geology,
latitudes greener and thus more fire prone. [3]
atOhio State
The pattern Box observed in 2012 may repeat
16. A. NO CHANGE
[21]itself again, with harmful effects on the
B. thaw; and it was
Arctic ecosystem. [4] Box is currently
C. thaw:
organizing an expedition to gather this crucial

94
D. thaw: being Questions 23-33 are based on the following
17. A. NO CHANGE passage.
B. soot
Coworking: A Creative Solution
C. of which
D. DELETE the underlined portion. When I left my office job as a website
18. A. NO CHANGE developer at a small company for a position that
B. falls allowed me to work full-time from home, I
C.will fall thought I had it made: I gleefully traded in my
D. had fallen suits and dress shoes for sweatpants and
19. A.NO CHANGE slippers, my frantic early-morning bagged lunch
B. its packing for a leisurely midday trip to my
C. there refrigerator. The novelty of this comfortable
D. their work-from-home life, however, [23]soon got
20. Which choice best completes the worn off quickly. Within a month, I found
description of aself-reinforcing cycle? myself feeling isolated despite having frequent
A. NO CHANGE email and instant messaging contact with my
B. raises the surface temperature. colleagues. Having become frustrated trying to
C. begins to cool at a certain point. solve difficult problems, [24] no colleagues
D. leads to additional melting were nearby to share ideas. It was during this
21. A. NO CHANGE time that I read an article [25]into coworking
B. itself, spaces.
C. itself, with damage and
The article, published by Forbes magazine,
D.itself possibly,
explained that coworking spaces are designated
22. To make this paragraph most logical,
locations that, for a fee, individuals can use to
sentence 4 should be placed
conduct their work. The spaces are usually
A. where it is now.
stocked with standard office [26]equipment,
B. after sentence 1.
such as photocopiers, printers, and fax
C. after sentence 2.
machines. [27]In these locations, however,
D. after sentence 5.
the spaces often include small meeting areas
and larger rooms for hosting presentations.
[28]The cost of launching a new coworking
business in the United States is estimated to
be approximately $58,000.

What most caught my interest, though, was a


quotation from someone who described

95
coworking spaces as “melting pots of C. promptly wore
creativity.” The article refers to a 2012 survey D. wore
in which [29]64 percent of respondents
24. A. NO CHANGE
noted that coworking spaces prevented
B. colleagues were important for
them from completing tasks in a given time.
sharingideas.
The article goes on to suggest that the most
valuable resources provided by coworking C. ideas couldn’t be shared with colleagues.
spaces are actually the people [30]whom use D. I missed having colleagues nearby to consult.
them.
25. A. NO CHANGE
[1] Thus, even though I already had all the B. about
equipment I needed in my home office, I C. upon
decided to try using a coworking space in my D. for
city. [2] Because I was specifically interested in
26. A. NO CHANGE
coworking’s reported benefits related to
B. equipment, such as:
creativity, I chose a facility that offered a bright,
C. equipment such as:
open work area where I wouldn’t be isolated.
D. equipment, such as,
[3] Throughout the morning, more people
appeared. [4] Periods of quiet, during which 27. A. NO CHANGE
everyone worked independently, were broken B. In addition to equipment,
up occasionally with lively conversation. [30] C. For these reasons,
D. Likewise,
I liked the experience so much that I now go to
the coworking space a few times a week. Over 28. The writer is considering deleting the
time, I’ve gotten to know several of my underlinedsentence. Should the sentence
coworking [32]colleagues: another website be kept or deleted?
developer, a graphic designer, a freelance A. Kept, because it provides a detail that
writer, and several mobile app coders. Even supportsthe main topic of the paragraph.
those of us who work in disparate fields are B. Kept, because it sets up the main topic of
able to [33]share advice and help each other theparagraph that follows.
brainstorm. In fact, it’s the diversity of their C. Deleted, because it blurs the paragraph’s
talents and experiences that makes my mainfocus with a loosely related detail.
coworking colleagues so valuable. D. Deleted, because it repeats information that
hasbeen provided in an earlier paragraph.
23. A).NO CHANGE
29. At this point, the writer wants to add
B. was promptly worn
specific information that supports the main
topic of the paragraph.

96
B. after sentence 1.
C. after sentence 2.
D. after sentence 3.
32. A. NO CHANGE
B. colleagues;
C. colleagues,
D. colleagues
33. A. NO CHANGE
B. give some wisdom
C. proclaim our opinions
D. opine

Which choice most effectively completes the


sentence with relevant and accurate information
based on the graph above?
A. NO CHANGE
B. 71 percent of respondents indicated that
using a coworking space increased their
creativity.
C. respondents credited coworking spaces with
giving them 74 percent of their ideas relating
to business.
D. respondents revealed that their ability to
focus on their work improved by 12 percent
in a coworking space.
30. A. NO CHANGE
B. whom uses
C. who uses
D. who use
Questions 34-44 are based on the following
30. The writer wants to add the following sentence
passage
to the paragraph. After filling out a simple
registration form and taking a quick tour of the The Consolations of Philosophy Long viewed by
facility, I took a seat at a table and got right to many as the stereotypical useless major,
work on my laptop. The best placement for the philosophy is now being seen by many students
sentence is immediately and prospective employers as in fact a very
A. before sentence 1. useful and practical major, offering students a

97
host of transferable skills with relevance to the majoring in philosophy have no intention of
modern workplace. [34]In broad terms, becoming philosophers; instead they plan to
philosophy is the study of meaning and the apply those skills to other disciplines. Law and
values underlying thought and behavior. But business specifically benefit from the
[35]more pragmatically, the discipline complicated theoretical issues raised in the
encourages students to analyze complex study of philosophy, but philosophy can be just
material, question conventional beliefs, and as useful in engineering or any field requiring
express thoughts in a concise manner. Because complex analytic skills. [42] That these skills
philosophy [36]teaching students not what to are transferable across professions.
think but how to think, the age-old discipline
[43]which makes them especially beneficial to
offers consistently useful tools for academic and
twenty-first-century students. Because today’s
professional achievement. [37] A 1994 survey
students can expect to hold multiple jobs—
concluded that only 18 percent of American
some of which may not even exist yet—during
colleges required at least one philosophy
[44]our lifetime, studying philosophy allows
course. [38]Therefore, between 1992 and 1996,
them to be flexible and adaptable. High demand,
more than 400 independent philosophy
advanced exam scores, and varied professional
departments were eliminated from institutions
skills all argue for maintaining and enhancing
More recently, colleges have recognized the philosophy courses and majors within academic
practicality and increasing popularity of institutions
studying philosophy and have markedly
34. A. NO CHANGE
increased the number of philosophy programs
B. For example,
offered. By 2008 there were 817 programs, up
C. In contrast,
from 765 a decade before. In addition, the
D. Nevertheless,
number of four-year graduates in philosophy
35. A. NO CHANGE
has grown 46 percent in a decade. Also, studies
B. speaking in a more pragmatic way,
have found that those students who major in
C. speaking in a way more pragmatically,
philosophy often do better than students from
D. in a more pragmatic-speaking way,
other majors in both verbal reasoning and
36. A. NO CHANGE
analytical [39]writing. These results can be
B. Teaches
measured by standardized test scores. On the
C. to teach
Graduate Record Examination (GRE), for
D. and teaching
example, students intending to study
37. Which choice most effectively sets up the
philosophy in graduate school [40]has scored
information that follows?
higher than students in all but four other
majors. These days, many [41] student’s

98
A. Consequently, philosophy students have A. Yes, because it reinforces the passage’s
been receiving an increasing number of job mainpoint about the employability of
offers. philosophymajors.
B. Therefore, because of the evidence, B. Yes, because it acknowledges a
colleges increased their offerings in commoncounterargument to the passage’s
philosophy. central claim.
C. Notwithstanding the attractiveness of this C. No, because it blurs the paragraph’s focus
course of study, students have resisted byintroducing a new idea that goes
majoring in philosophy. unexplained.
D. However, despite its many utilitarian D. No, because it undermines the passage’s
benefits, colleges have not always claimabout the employability of philosophy
supported the study of philosophy. majors.
38. A. NO CHANGE 43. A. NO CHANGE
B. Thus, B. That
C. Moreover, C. And
D. However, D. DELETE the underlined portion.
39. Which choice most effectively combines the 44. A. NO CHANGE
sentences at the underlined portion? B. one’s
A. writing as C. his or her
B. writing, and these results can be D. their
C. writing, which can also be
D. writing when the results are
40. A. NO CHANGE
B. have scored
C. scores
D. scoring
41. A.NO CHANGE
B. students majoring
C. students major
D. student’s majors
42. At this point, the writer is considering
adding the following sentence. The ancient
Greek philosopher Plato, for example,wrote
many of his works in the form of dialogues.
Should the writer make this addition here?

99
GMST
Latihan Reasoning - Verbal & Non Verbal

i) Verbal Reasoning ii) Non-verbal reasoning (figural)


a. Completing a series a. Odd one out
 Number series b. Completion of series
 Letter series c. Analog test
 Analog test d. Completing an Incomplete pattern
 Coding and decoding e. Spotting hidden pattern

 Arithmetical reasoning f. Identical and similar pattern


g. Lateral inversion

COMPLETING A SERIES. g. 1, 4, 9, 16, 25, ?


NUMBER SERIES h. 1, 2, 8, 9, 15, 16, ?

Hints for answering the questions ANSWERS:


In these questions the series consists a. 78,each term is obtained after addingtwo
ofnumbers(digits). Number series are formed to the previous number
inmany ways b. 768, multiply each term by 4 to get thenext
1. Addition of figures given in row number
2. Subtraction of figures from one another c. 54, divide by 2 and multiply by
3. Division of figures by one another 3alternatively.
4. Multiplication of figures d. 22, constant difference of 3 in
5. Logical transportation of figures eachsubsequent number.
6. Increase/decrease of numbers in aspecific e. 25, decreasing difference of
pattern 5,4,3,2,1between subsequent numbers
f. 82, interval between each pair ofnumbers
EXAMPLE:
becomes the succeeding termin series
Write down the number which will complete
g. each term is square of natural number
the sequence and replace the?
h. 22, interval is alternatively 1 and 6
a. 6, 12, 18, 30, 48, ?
b. 3, 12, 48, 192, ?
c. 48, 24, 72, 36, 108, ?
d. 1, 4, 7, 10, 13, 16, 19, ?
e. 10, 15, 19, 22, 24, ?
f. 905, 576, 329, 247, ?

100
Multiple Choice Questions 9. 1, 2, 3, 5, 7,______,13
A. 8 B. 9
Find the missing number in the following
C. 10 D. 11
series out of four options.

1. 1, 2, 5, 7, 11,_____,17 10. 3, 8, 22, 63, 185,_____


A. 12 B. 13 A. 285 B. 295
C. 14 D. 15 C. 310 D. 500

2. 1, 8, 27,_____,125 11. 4, 5, 7,_____, 19


A. 37 B. 47 A. 56 B. 66
C. 54 D. 64 C. 11 D. 17

3. 6, 12, 21, 33,_____ LETTER SERIES


A. 38 B. 40 Letter series contain only letters of thealphabet
C. 45 D. 48 placed in a specific pattern . They mayconsist of
terms with one , two , several letters.In such
4. 2, 20,_____,110, 182 series the successive elements areobtained
A. 56 B. 72 from the previous element according toa
C. 90 D. 96 definite rule.

Example: U,B,I,P,________
5. 81, 9, 64,_____,49, 7
A. 8 B. 9 Answer : If you study the pattern carefullyyou
C. 10 D. 11 will find that every seventh letter is
takenalphabetically. After U the letters VWXYZA
6. 15, 20, 30,_____,65 areomitted and B is taken. After B , CDEFGH
A. 35 B. 40 areomitted and I is taken. After I , JKLMNO
C. 45 D. 50 areomitted and P is taken to follow the same
patternnaturally. After P, QRSTUV should be
7. 18, 30, 48, 72, 96,______ omittedand W will be the next letter in the
A. 106 B. 115 series.
C. 120 D. 96

8. 12, 32, 72, 152,_______,632


A. 515 B. 613
C. 815 D. 312

101
Multiple Choice Questions 8. A, C, F, J, O,____________
A. U B. C
Each of the term in the questions belong toletter
C. D D. F
series. One of the term in questions belongto
letter series. One of the terms of the series
9. C, J, P, U, Y,____________
ismissing. Find the missing term from amongst
A. C B. D
thefour alternatives.
C. B D. A
1. J,L,N,P,R,T,________
A. V B. X 10. AZ, BY, CX, DW, ______
C. Y D. Z A. EX B. EU
C. EY D. EV
2. C,F,I,L,O,R,_________
A. V B. U 11. aa HI cc, cc JK ee, ee LM gg, _____
C. W D. X A. ff NO ii B. gg NO ii
C. gg ON ee D. ee ON ii
3. B,F,J,N,R,V,_________
A. X B. Y 12. aBEf, bCFg, cDGh, _______
C. Z D. C A. dEGi B. dHIg
C. dEHi D. dHEi
4. E,J,O,T,Y,D,_________
A. J B. K 13. A, C, B, E, C, G, D,______
C. I D. F A. M,N B. L,M
C. I,E D. G,H
5. AD, EH, IL, ______, QT, UX
A. MP B. MN 14. CD, GH, KL, OP, ST, ________
C. MO D. MQ A. RS B. QR
C. GH D. WX
6. B, I, P, W, D, K,_________
A. Q B. R 15. WZI, EHJ, XAK, _____, YBM, GJN
C. S D. T A. ZCO B. IFL
C. FFL D. FIL
7. AZ, BY, CX, D________
A. E B. W
C. R D. V

102
ANALOGY TEST b) Cause and effect relationship
c) Part and whole relationship
Analogies Test your ability to see
d) Part and part relationship
therelationship between types of relationships
e) Action and object relationship
andrecognize which relationship are similar.
f) Synonym and antonym relationship
Hints for Answering Questions: g) Place relationship
h) Degree relationship
1. Establish the relationship
i) Characteristic relationship
Before you see the available answers try
j) Sequence relationship
andestablish what is the relationship
k) Numerical relationship
between givenpair.
l) Association relationship
Example:
Author : Book
a. Poem : poet
Multiple Choice Questions
b. Ring : goldsmith
c. Carpenter : hammer In each of the questions below, a related pair of
d. Composer : song words(in capital letters) is given. Select the
lettered pair out of four pair of words (in capital
The relationship is that author creates a
letters) is given. Select the lettered pair out of
book.Take the keyword ‘create’ and try to
four pair of words that expresses the relationship
fit this inall given choices
that is most similar to that of the capitalized
Poem ‘create’ poet wrong pair.
Ring ‘create’ Goldsmith wrong
1. SCISSORS:CLOTH ::
Carpenter ‘create’ hammer wrong
A. Axe:Wood
Composer ‘create’ song right
B. Stone : Grinder
So ‘d’ is the correct answer. C. Knife: Stone
D. Gun: Hunt
2. Sequence of items is important
2. FOOD:HUNGER ::
Like in above example person ie
A. Axe:Wood
‘Author’comes first and the item ie ‘Book’
B. Stone : Grinder
comes second.So in the answer also the
C. Knife: Stone
person comes second.The only answer that
D. Gun: Hunt
satisfy this condition is (d).
3. BIRD : NEST ::
Following types of relationship are
A. dog : doghouse
possible.
B. squirrel : tree
a) Purpose relationship
C. beaver : dam

103
D. cat : litter box A. Exercise:Strength
E. book : library B. Heat:Coil
4. DALMATIAN : DOG :: C. Power:Battery
A. Nightingale : bird D. Automobile:Engine
B. horse : pony 11. VOLCANO:LAVA ::
C. shark : great white A. Death:Sorrow
D. ant : insect B. Rock:Sand
E. stock : savings C. Delta:River
5. DOCTOR : HOSPITAL :: D. Fault:Earthquake
A. sports fan : stadium 12. SCALES : FISH ::
B. cow : far A. Lady : Dress
C. professor : college B. Skin : Man
D. criminal : jail C. Tree: Leaves
E. food : grocery store D. Bird: Feather
6. CUB : BEAR :: 13. PETAL : FLOWER ::
A. piano : orchestra A. salt : pepper
B. puppy : dog B. tire : bicycle
C. cat : kitten C. base : ball
D. eagle : predator D. sandals : shoes
E. fork : utensil E. puppy : dog
7. NOODLES : FLOUR :: 14. BRISTLE : BRUSH
A. snacks : breakfast A. arm : leg
B. Tomato : gravy B. stage : curtain
C. beans : coffee C. recline : chair
D. thread : cotton D. key : piano
8. INK:PAPER :: E. art : sculpture
A. Pen: pencil 15. WOOL : WARMTH
B. Paint: painting A. spring : elasticity
C. Chalk: blackboard B. marketing : advertising
D. Carbon paper: ballpoint C. person : success
9. ROCKGARDEN:CHANDIGARH :: D. radio : broadcast
A. Delhi: RedFort 16. ODOMETER : DISTANCE
B. Film Studio:Pune A. scale : weight
C. Chicago: USA B. length : width
D. India: Asia C. inch : foot
10. LIGHT : CANDLE :: D. mileage : speed

104
E. area : size C. artist : carpenter
17. WRITER : PEN D. nail : hammer
A. pencil : eraser E. frame : picture
B. ball pen : notebook 24. EXPLORE : DISCOVER
C. typewriter : paper A. read : skim
D. blacksmith : hammer B. research : learn
E. gun : hunter C. write : print
18. PEDAL : BICYCLE D. think : relate
A. inch : yardstick E. sleep : wake
B. walk : skip 25. COTTON : BALE
C. tire : automobile A. butter : churn
D. buckle : belt B. wine : ferment
E. oar : boat C. paper : ream
19. RIBS : LUNGS D. curd : cheese
A. ball : sphere E. beef : steak
B. hand : finger 26. STOOL : BENCH
C. shell : nut A. chair : table
D. coat : tie B. carpenter : chair
20. BIRD : CAGE C. foot rule : yardstick
A. animal : zoo D. glass : cup
B. thief : prison 27. LAWN : GRASS
C. antique : museum A. wool : sheep
D. crime : punishment B. skin : goat
21. WAR : DESTRUCTION C. fur : animal skin
A. fire : burn D. rice : farm
B. court : justice 28. WAITRESS : RESTAURANT
C. water : drown A. doctor : diagnosis
D. food : hunger B. actor : role
22. YEN : CURRENCY C. driver : truck
A. brass : metal D. teacher : school
B. flower : fragrance E. author : book
C. paper : book 29. FINCH : BIRD
D. karnataka : state A. frog : toad
23. BINDING : BOOK B. elephant : reptile
A. criminal : gang C. Dalmatian : dog
B. display : museum D. collie : marsupial

105
E. ant : ladybug 2. Balwant had 17 sheep. A storm in the
30. RAIN : DRIZZLE villagekilled all but 7 sheep .How many was
A. swim :dive he leftwith?
B. hop : shuffle A. 10 B. 6
C. juggle : bounce C. 7 D. 9
D. walk : run 3. What is the smallest number of ducks
E. run : jog thatcan swim in this formation- two ducks
31. EGG : OMELETTE infront of a duck two ducks behind a duck
A. dam : river anda duck between two ducks?
B. student : classroom A. 3 B. 5
C. clay : pottery C. 7 D. 9
D. onion : salad 4. What is the product of all the numbers in
32. TAILOR : SUIT thedial of a telephone?
A. scheme : agent A. 1,58,480 B. 1,59,450
B. edit : manuscript C. 1,59,480 D. None of those
C. revise : writer 5. A group of 1200 persons consisting
D. mention : opinion ofcaptains and soldiers is travelling in a
E. architect : building train.For every 15 soldiers there is one
captain.The number of captains in the
ARITHMETICAL REASONING group is
A. 85 B. 80
Hints for Answering Questions:
C. 75 D. 70
The section tests the basic calculation skills such
6. Aruna cut a cake into two halves and
as addition, subtraction, multiplication, division,
cutsone half into smaller pieces of equal
calculation of percentages, average etc. The
size.Each of the small pieces is twenty
questions of ‘age’ and direction are also included
grams inweight. If she has seven pieces of
in this test. The candidate must know the basics
the cakein all with her, how heavy was the
of geometry to solve the direction problems.
originalcake?
1. Rajesh walks 7 km to the north then A. 120 grams B. 140 grams
turnsleft and walks 3 km. Then takes C. 240 grams D. 280 grams
another leftand continue walking this time 7. First bunch of bananas has ¼ again as
another 7 kmforward. How much distance manybananas as a second bunch. If the
in km is heaway from the starting point. secondbunch has 3 bananas less than the
A. 10km B. 6km first bunch,then the number of bananas in
C. 3km D. 2km the first bunchis
A. 9 B. 10

106
C. 12 D. 15 To determine which product is the better
8. At the end of a business conference the buy, calculate each one’s unit price. Which
tenpeople present all shake hands with of these five boxes of Klean-O Detergent is
eachother once. How many handshakes the best buy?
will therebe altogether? A. Travel-size: Rs. 100 for 5 Kg.
A. 20 B. 45 B. Small: Rs. 200 for 11 KG.
C. 55 D. 90 C. Regular: Rs. 400 for 22 Kg.
9. A student got twice as many sums wrong D. Large: Rs. 700 for 40 Kg.
ashe got right. If he attempted 48 sums in 14. Rimmy pulse rate is 19 beats every 15
all,how many did he solve correctly. seconds. What is her rate in beats per
A. 12 B. 16 minute?
C. 18 D. 24 A. 76
10. There were 28 people at last week’s B. 60
boardmeeting. If the ratio of men to women C. 57
was4:3, how many women were at the D. 45
meeting? 15. If you write down all the numbers from 1
A. 16 B. 12 to 100, then how many times do you write
C. 7 D. 4 3?
11. At a certain corporation, the ratio of A. 11
clericalworkers to executives is 7 to 2. If a B. 18
combinedtotal of 81 clerical workers and C. 20
executiveworkers for that corporation, D. 21
how manyclerical workers are there? 16. The total number of digits used in
A. 9 B. 14 numbering the pages of a book having 366
C. 18 D. 63 pages is
12. Last Year, there were 720 crimes A. 732
committed in the ninth precinct. These B. 990
crimes involved theft, rape, and drugs in C. 1098
the ration of 4:2:3, How many crimes D. 1305
involved drugs? 17. In a garden, there are 10 rows and 12
A. 80 coloumns of mango trees. The distance
B. 160 between two trees is 2 meters and a
C. 240 distance of one meter is left from all sides
D. 320 of the boundary of the garden. The length
13. A unit price is a ratio that compares the of the garden is
price of an item to its unit of measurement. A. 20 m

107
B. 22m C. 12/31
C. 24m D. 5/8
D. 26m 22. Find the number which when added to
18. A farmer built a fence around his square itself 13 times, gives 112.
plot. He used 27 fence poles on each side of A. 7
the square. How many poles did he need B. 8
altogether? C. 9
A. 100 D. 11
B. 104 23. A monkey climbs 30 feet at the beginning
C. 108 of each hour and rests for a while when he
D. None of these slips back 20 feet before he again starts
19. An enterprising businessman earns an climbing in the beginning of the next hour.
income of Re.1 on the first day of his If he begins his ascent at 8.00 a.m., at what
business. On every subsequent day, he time will he first touch a flag at 120 feet
earns on income which is just double of from the ground?
that made on the previous day. One the A. 4 p.m.
10th day of business, his income is B. 5 p.m.
A. Rs. 29 C. 6 p.m.
B. Rs. 210 D. None of these
C. Rs. 10 24. In a city, 40% of the adults are illiterate
D. Rs. 102 while 85% of the children are literate. If the
20. In a family, the father took ¼ of the cake ratio of the adults to that of the children is
and he had 3 times as much as each of 2:3, then what percent of the population is
theother members had. The total number literate?
of family members are A. 20%
A. 3 B. 25%
B. 7 C. 50%
C. 10 D. 75%
D. 12 25. If every 2 out of 3 readymade shirts need
21. Three friends had dinner at a restaurant. alterations in the sleeves, and every 4 out
When the bill was received, Amita Paid 2/3 of 5 need in the body, how many
as much as Veena paid and Veena paid 1/3 alterations will be required for 60 shirts?
as much as Tanya paid. What fraction of the A. 24
bill did Veena pay? B. 123
A. 1/3 C. 133
B. 3/11 D. 143

108
26. If a clock takes seven seconds to strikes she has driven in the wrong directions. She
seven, how long will it take to strike ten? then turns to her right and travels 2 km
A. 7 seconds and then she again turns to the right and
B. 9 seconds drives straight another 25 km. How much
C. 10 seconds distance has she now to cover to go back to
D. None of these the starting point?
A. 25 km
B. 2km
Directions: Mr and Mrs. Sharma have two
C. 4km
children Asha and Shahsi.Shashi married Radha,
D. 40 km
daughter of Mrs. Mahajan. Suresh, son of Mrs.
32. Rana travels 10 km to the north, turns left
Mahajan marries Rita. Sonu and Rocky are born
and travels 4 km and then again turns right
to Suresh and Rita. Uma and Sudha are the
and covers another 5 km and then turns
daughters of Shashi and Radha.
right and travels another 4 km. How far is
27. What is the surname of Sonu? he from the right starting point?
A. Mahajan A. 15 km
B. Sharma B. 4 km
C. Shashi C. 5 km
D. None of these D. 10 km
28. How is Suresh related to Sudha? 33. A taxi driver commenced his journey from
A. brother a point and drove 10 km towards north and
B. maternal uncle turned to his left and drove another 5 km.
C. uncle After waiting to meet a friend here, he
D. cousin turned to his right and continued to drive
29. What is Sudha’s relation to Asha? another 10 km. He has covered a distance
A. sister of 25 km so far, but in which direction
B. niece would he be now?
C. aunt A. North
D. daughter B. East
30. How is Sonu related to Mr. Mahajan? C. South
A. son-in-law D. West
B. grandson 34. A tourist drives 10 km towards east and
C. son turns to the right hand and drives 3 km.
D. none of these Then he drives towards west (turning to
31. Rita drives to the north of her place of stay his right) 3 km. He then turns to his left and
at A and after traveling 25 km finds that drives 2 km. Finally he turns to his right

109
and travels 7 km. How far is he from his LOGICAL VENN DIAGRAMS
starting point and in which direction would
This Section deals with questions which
he be?
aim at analyzing a candidate’s ability to relate a
A. 10 km, East
certain given group of items and illustrate it
B. 9 km, North
diagrammatically. In these tests a relationship
C. 8 km, West
is to be established between two or more items
D. 5 km, West
represented by diagrams. The items
35. Of the 30 Officers on traffic duty, 20%
represented by the diagrams may be individual,
didn’t work on Friday. How many officers
a particular group/class of people etc.
worked on Friday?
A. 6 Example:
B. 10
You are required to choose from the five
C. 12
diagrams the one that best illustrates the
D. 24
relationship among the three given classes in
the questions that follow

Questions

1. Criminals, Pickpocketers, Terrorists

2. Dogs, Friendly Animals, Cats

3. Food, Curd, Spoons

Answers

1. E

2. D

3. C

110
Explanation: 1. Complete Exclusion

1. Both Pickpocketers and Terrosist fall under 2. Complete Inclusion


the class of criminals. There is a possibility
3. Partial Inclusion
that some of pickpocketers can be terrorists
and some of terrorists can be pickpocketers
2. Both dogs and cats can be wild as well as
friendly animals, so we can infer that
although both fall under separate classes
some cats and some dogs may be friendly
3. Curd is covered under the class Food.
However spoon is a separate class and has These three cases represent possible

no relation with them. relationships between two classes if more


classes are added it would result in more
possible relationships. A class could be
Hints for Answering Questions:
represented by any shape (circle, square,
To tackle questions on logical diagrams triangle etc ).
following hints will be helpful
Directions: Select the diagram that
1. These tests are based on the concept of class. bestrepresents the relation of terms given in
A class is a group of items, all having question 1-10:
something in common. Hence a class of
MAMMALS will be all those members of
animal kingdom who are MAMMALS and
REPTILES will be not included in this class.
2. Classes can contain classes. A class may
contain or be contained with another.
Example: FATHERS will be contained in the
larger group of MALES. But not all MALES
will be FATHERS.
3. One class may be partially contained in 1. Animals, Cows, Dogs
another
2. Cousins, Males, Nephews
Example: HUSBANDS and BROTHERS. All
BROTHERS are not HUSBANDS and all 3. Women, Story teller, Liars
HUSBANDS are not BROTHERS. The Partial
4. Doctors, Surgeons, Musicians
relationship is as depicted in figure. So a
relationship can be of three basic types 5. Students, Females, MBA entrance candidates

111
6. Bedroom, Sitting room, Dwelling

7. Civil Engineers, Boiler Engineers, Engineers

8. People, Doctors, Cows

9. Typewriter, Reading material, Magazines

10. Policemen, Magistrates, Human beings 13. Which of the following statements is true?
a. all urbans are graduates
11. Mule, Father Book
b. all graduates are urbans
12. Family, Spouse, Husbands c. all the urban government employees
are graduates
13. Doctors, Human beings, Cows
d. all teachers are urban people
14. Social workers, Alcoholics, Teatotallers 14. Choose the correct statement?
1. there are some urban teachers who
15. Females, Mothers, Nurses
are government employees as well as
16. Dogs, Rabbits, Rats graduates
2. no teacher is a government employee
17. Females, Mothers, Sisters – a
3. all graduates are government

18. Beef, Mutton, Eggs employee


4. all government employees are urban
19. Students, Married people, Human beings -a
people

20. Husbands, Brothers, Males – c 15. Mark the correct statement?


A. all nonurban teachers are government
employee
Directions: Study the following diagram B. all urban government employees are
carefully and answer the questions the follow:
teachers
C. there are some nonurban graduates
who are neither teachers nor
government employees
D. all urban government employees are
graduates
16. Which of the following statement is not
true?
A. some government employees are rural
B. all teachers are urban

112
C. teachers who are government
employees are that
D. all govt. employees are urban people.

Directions: In the following diagram there are


three intersecting circles each representing
certain sections of people. Different regions are
marked a-g. Read the statements in question
and choose the letter of the region which
correctly represents the statements?

E. Which one of the following statement is


true?
A. all urbans are post graduates
B. all post graduates are urbans
C. all professors are urban people
A. Russians who are artists but not singers.
D. all rural people are professors
25. B
F. Choose the correct statement?
26. C
A. There are some professors who are
27. B
rural people
28. G
B. No professor is urban
B. Artists who are neither Russians nor
C. All post graduates are rural
singers.
D. All post graduates are urban
A. g
G. Which of the following statement is true?
B. c
A. all rural people are professors
C. f
B. There are some rural people who are
D. b
post graduates and professors
C. Russians who are singers but not artists.
C. All rural people are post graduates
A. a
D. All professors are rural but not post
B. b
graduates
C. c
H. Mark the correct statement
D. d
A. all urban people are post graduates
D. Russians who are artists as well as singers.
B. all rural people are professors
A. a
C. some professors are rural but not
B. b
urban
C. c
D. some urban people are not post
D. d
graduates

113
E. NON-VERBAL REASONING

Important Points

The purpose of non-verbal reasoning test is to find out how well a candidate understands ideas which
are not presented in words or numbers, but in figures, diagrams, pictograms presented in terms of size
or shape or position or quantity or other non-verbal, non-numerical forms. What is important in solving
the problems of non-verbal reasoning tests is to follow a logical procedure. This latent logical procedure
will help the candidate to find out the principle on which the problem figure is based.

Odd One Out


5
Multiple Choice Questions

Directions: In the given four figures, three of


them are similar in some respect, while one is
A B C D
different. Out of the four figures select the figure
which is different. 6

A B C D

7
A B C D

2
A B C D

8
A B C D

3
A B C D

9
A B C D

4
A B C D
10

A B C D

A B C D

114
11 18

A B C D A B C D

12 19

A B C D A B C D

13 20

A B C D
A B C D
14
21

A B C D
A B C D
15
22

A B C D
A B C D
16
23

A B C D
A B C D
17
24

A B C D
A B C D

115
25 26

A B C D A B C D

116
Completion of Series

Important Points

In this test, a set of 3/4/5 figures is given. Each figure changes in design from the preceding one in a
particular order. Candidates are required to choose the correct answer figure, which would best
continue the series. This test judges your pattern perception. You have to discern the pattern which
may be quantitative, qualitative, rotative, multi-relational, or ratio-based. The series-may also be based
on mixed operations, in which various elements change their directions/positions, increase or decrease
in number as well as change qualitatively. This test also includes questions on correct sequence of
figures. Students are asked as to interchange of which two figures would make the series correct.

Multiple Choice Questions

Directions: There are two set of figures. One set is called problem figures and the other set is called
answer figures. Problem set figures from some kind of series. Select one figure from the answer set
figures which will continue the series in the problem set figures.

Problem Figures Answer Figures


1

1 2 3 4 A B C D

1 2 3 4 A B C D

1 2 3 4 A B C D

1 2 3 4 A B C D

117
5

1 2 3 4 A B C D

1 2 3 4 A B C D

1 2 3 4 A B C D

1 2 3 4 A B C D

1 2 3 4 A B C D

10

1 2 3 4 A B C D

11

1 2 3 4 A B C D

118
12

1 2 3 4 A B C D

13

1 2 3 4 A B C D

14

1 2 3 4 A B C D

15

1 2 3 4 A B C D

119
Analog Test

'Analogy' implies corresponding. In the problems based on analogy, a pair of related figures is provided
and a similar relationship is to be established between two other figures, by selecting one or both of
them from a set of alternatives.

Multiple Choice Questions

Directions: Figures 1 & 2 in Problem figures are related in a particular manner. Establish the same
relationship between figures 3 and 4 by choosing a figure from among the four alternative answer
figures, which would replace the question mark in problem figures.

Problem Figures Answer Figures


1

1 2 3 4 A B C D

1 2 3 4 A B C D

1 2 3 4 A B C D

1 2 3 4 A B C D

1 2 3 4 A B C D

120
6

1 2 3 4 A B C D

1 2 3 4 A B C D

1 2 3 4 A B C D

1 2 3 4 A B C D

10

1 2 3 4 A B C D

11

1 2 3 4 A B C D

12

1 2 3 4 A B C D

121
13

1 2 3 4 A B C D

14

1 2 3 4 A B C D

15

1 2 3 4 A B C D

122
Completing an Incomplete Pattern

These are based on the Raven Progressive Matrices Tests (1938-56). They were developed in England
and used during World War II. They are non-verbal scales developed to judge your ability to
comprehend correlations between geometric figures and diagrams, and to perceive the structure of the
design in order to select the appropriate part from amongst a number of choices for completion of each
pattern.

Multiple Choice Questions

Directions: A pattern/figure is given, a portion of which is left incomplete. A few choices- a,b,c and d in
the answer figures are also given. You have to select the figure which fits into the blank space in the
incomplete figure so that the original pattern is complete.

Incomplete Figures Answer Figures


1

123
6

10

11

12

124
13

14

15

125
Spotting Hidden Pattern

Important Points

These non-verbal tests are designed to judge your sense of observation and analytical aptitude. In such
questions, the PROBLEM FIGURE comprises just one figure. The ANSWER FIGURE contains four
different blocks. The ANSWER FIGURE has a hidden figure of the PROBLEM FIGURE which is to be
spotted out.

Multiple Choice Questions

Direction: There is one block on left hand side marked (X) which has the basic design. The four blocks
that follow, i.e. A,B,C and D have a group of more complex figures in one of which the main/basic figure
given left hand column (X) is hidden/embedded in some position. You have to spot that particular
figure from the choice of answers A,B,C and D.

X A B C D

X A B C D

X A B C D

X A B C D

X A B C D

126
6

X A B C D

X A B C D

X A B C D

X A B C D

10

X A B C D

11

X A B C D

12

X A B C D

13

X A B C D

127
14

X A B C D

Identical or Similar Pattern

In these tests, the problem figure comprises only one figure. Below it are given four alternative figures.
Of the four alternative figures, one figure is similar or identical to the problem figure. The candidates
must study the problem figure and note its characteristic or peculiarities in detail. Then they should try
to identify or locate the alternative figure which is identical to the problem figure. The answer figure
should be identical to the problem figure, or should have the same characteristics as the problem figure,
which is to be spotted out.

Multiple Choice Questions

Directions: Select from the four alternative figures, the one with the same characteristics as in the given
figure.

1 Given Figure: 2 Given Figure:

Alternate Figure:
Alternate Figure:

A B

A B

C D

C D

128
3 Given Figure: 6 Given Figure:

Alternate Figure:
Alternate Figure:

A B
A B C D
7 Given Figure:

C D
4 Given Figure:
Alternate Figure:

Alternate Figure:
A B C D
8 Given Figure:

A B C D
5 Given Figure:

Alternate Figure:

Alternate Figure:

A B

A B

C D

C D

129
9 Given Figure: 12 Given Figure:

Alternate Figure:
Alternate Figure:

A B C D
A B C D
13 Given Figure:
10 Given Figure:

Alternate Figure:
Alternate Figure:

A B C D
11 Given Figure:

A B

Alternate Figure:

C D

A B

C D

130
14 Given Figure:

Alternate Figure:

A B

C D

131
Lateral Inversion 3. Key Figure:

Important Points

This test requires the candidate to observe


carefully how a given figure will appear in the Answer Figure:
mirror, if the mirror is held at a particular
position. For this test, you must remember that
the left side of a pattern appears to be on the
right side in the mirror and vice-versa.
A B C D
Multiple Choice Questions
4. Key Figure:
Directions: In the following questions, a figure
is followed by four suggested mirror reflections
of it. Assuming that the mirror is placed to the
right of the figure (except in questions 7,11 and
12), pick out the correct reflected figure. Answer Figure:
1. Key Figure:

A B C D

Answer Figure:
5. Key Figure:

A B C D

Answer Figure:
2. Key Figure:

A B C D
Answer Figure:

A B C D

132
6. Key Figure: 8. Key Figure:

Answer Figure: Answer Figure:

A B A B C D

C D

7. Key Figure:

Answer Figure:

A B C D

Direction ((9-26): In each one of the following questions, choose the correct mirror image of the figure
'X' from amongst the form alternatives a,b,c,d given along with it.

9.

(X) A B C D

133
10.

(X) A B C D

11. (Mirror is placed at the Bottom)

(X) A B C D

12. (Mirror is placed at the Top)

(X) A B C D

13.

(X) A B C D

14.

(X) A B C D

134
15.

(X) A B C D

135
PRACTICE SET 1

Instruction:
 This practice set consists of two sections. Quantitative Aptitude (Qs. 1-40) & Reasoning Ability (Qs.
41-80).
 All the questions are compulsory.
 Each question has five options, of which only one is correct. The candidates are advised to read all
the options thoroughly.
 There is negative marking equivalent to 1/4th of the mark allotted to the specific question for
wrong answer.
Time: 45 Min, Max. Marks: 80

QUANTITATIVE APTITUDE

DIRECTIONS (Qs. 1-5): Find out the approximate value which should come in place of the question mark
in the following questions. (You are not expected to find the exact value.)

1. 45689 = ? 5. (299.99999)3 = ?
A. 180 B. 415 A. 27000000 B. 9000000000
C. 150 D. 210 C. 180000 D. 2.7 × 109
E. 300 E. 2700000

(10008 .99)𝟐 117 × 117 × 117 − 98 × 98 × 98


2. × 3589 × 0.4987 ? 6. 117 × 117 + 117 × 98 + 98 × 98
=?
10009 .001

A. 3000 B. 300000 A. 215 B. 311

C. 3000000 D. 5000 C. 19 D. 29

E. 9000000 E. None of these

𝑎 4 3𝑎 +2𝑏
3. 399.9 + 206 × 11.009 = ? 7. If 𝑏 = 3 , then 3𝑎 −2𝑏 = ?
A. 2800 B. 6666 A. 6 B. 3
C. 4666 D. 2400 C. 5 D. –1
E. 2670 E. None of these

𝟐 𝟕 17 6
4. + × 19 ÷ 5 = ? (3.537 − 0.948)2 + (3.537 + 0.948)2
𝟓 𝟖 8. (3.537 )2 + (0,948)2
=?
1
A. 1 B. A. 4.485 B. 2.589
2
1 3
C. 22 D. C. 4 D. 2
4
9 E. None of these
E. 11

136
9.
112
×
576
×
256
=? DIRECTIONS (Qs. 16-20): Study the following
196 12 8
chart to answer the questions given below.
A. 8 B. 12
C. 16 D. 32 Villages % population below poverty line
E. None of these A 45
B 52
5− 3
10. =? C 38
5+ 3
D 58
A. 4 + 15 B. 4 - 15
1 E 46
C. 2
D. 1
F 49
E. None of these
G 51

DIRECTIONS (Qs. 11-15): Identify which Proportion of population of seven villages in


number is wrong in the given series. 2014

11. 2, 3, 4, 4, 6, 8, 9, 12, 16. G A


A. 3 B. 9 15% 13%
C. 6 D. 12 F B
E. None of these 13% 16%

12. 3, 4, 10, 32, 136, 685, 41


C
A. 136 B. 10 8%
E D
C. 4116 D. 32 18% 17%
E. None of these
13. 69, 55, 26, 13, 5
16. In 2015, the population of villages A as well
A. 26 B. 13
as B is increased by 10% from the year
C. 5 C. 55
2014. If the population of village A in 2014
E. None of these
was 5000 and the percentage of population
14. 24576, 6144, 1536, 386, 96, 4
below poverty line in 2015 remains same
A. 386 B. 6144
as in 2014, find approximately the
C. 96 D. 1536
population of village B below poverty line
E. None of these
in 2015.
15. 11, 5, 20, 12, 40, 26, 74, 54
A. 4000 B. 45000
A. 5 B. 20
C. 2500 D. 3500
C. 40 D. 26
E. None of these
E. None of these

137
17. If in 2016 the population of village D is DIRECTIONS (Qs. 21-25): These questions are
increased by 10% and the population of based on the table and information given below.
village G is reduced by 5% from 2014 and
There are 6 refineries, 7 depots and 9 districts.
the population of village G in 2014 was
The refineries are BB, BC, BD, BE, BF and BG.
9000, what is the total population of
The depots are AA, AB, AC, AD, AE, AF and AG.
villages D and G in 2016?
The districts are AAA, AAB, AAC, AAD, AAE,
A. 19770 B. 19200
AAF, AAG, AAH ad AAI. Table A gives the cost of
C. 18770 D. 19870
transporting one unit from refinery to depot.
E. None of these
Table B gives the cost of transporting one unit
18. If in 2014 the total population of the seven
from depot to a district.
villages together was 55,000
Tabel A
approximately, what will be population of
BB BC BD BE BF BG
village F in that year below poverty line ?
AA 929.2 537.2 567.8 589.9 589.9 800.1
A. 3000 B. 2500
AB 311.1 596.7 885.7 756.9 759.9 793.9
C. 4000 D. 3500 AC 451.1 0 320.1 780.1 720.7 1000
E. None of these AD 371.1 150.1 350.1 750.1 650.4 980.1
19. If the population of village C below poverty AE 1137.3 314.5 0 1158 1158 1023
line in 2014 was 1520, what was the AF 617.1 516.8 756.5 1066 1066 406.3
population of village F in 2014? AG 644.3 299.2 537.2 1093 1093 623.9
A. 4000 B. 6000
C. 6500 D. 4800 Tabel B

E. None of these AA AB AC AD AE AF AG

20. The population of village C is 2000 in 2014. AAA 562.7 843.2 314.5 889.1 0 754.8 537.2

What will be the ratio of population of AAB 532.7 803.2 284.5 790.5 95.2 659.6 442
AAC 500.7 780.2 0 457.3 205.7 549.1 331.5
village C below poverty line to that of the
AAD 232.9 362.1 286.2 275.4 523.6 525.3 673.2
village E below poverty line in that year ?
AAE 345.1 268.6 316.2 163.2 555.9 413.1 227.8
A. 207 : 76
AAF 450.1 644.3 346.2 372.3 933.3 402.9 379.1
B. 76 : 207
AAG 654.5 0 596.7 222.7 885.7 387.6 348.5
C. 152 : 207
AAH 804.1 149.6 627.2 360.4 1035.3 537.2 498.1
D. Data inadequate
AAI 646 255 433.5 137.7 698.7 112.2 161.5
E. None of these

21. What is the least cost of sending one unit


from any refinery to any district?
A. 95.2 B. 0
C. 205.7 D. 284.5
E. None of these

138
22. What is the least cost of sending one unit 15 9
28. I 𝑥
+ 𝑥
= 11 𝑥
from any refinery to the district AAB? 𝑦 5 𝑦 1
II + =
A. 0 B. 284.5 4 12 𝑦
8 6
C. 95.2 D. 294.8 29. I 𝑥
+ 𝑥
= 𝑥
E. None of these (14)2
II 𝑦3 - 𝑦
=0
23. What is the least cost of sending one unit
30. I 𝑥 2 – 208 = 233
from refinery BB to any district?
II 𝑦 2 – 47 + 371 = 0
A. 284.5 B. 311.1
31. A reduction of 20% in the price of sugar
C. 451.1 D. 297.5
1
E. None of these enables a purchaser to obtain 22 kg more

24. What is the least cost of sending petrol for $160. Find the original price per kg of
from refinery BB to district AAA? sugar.
A. 765.6 B. 1137.3 A. $12 B. $20
C. 1154.3 D. 1174.8 C. $16 D. $18
E. None of these E. None of these
25. How many possible ways are there for 32. Mrs. X spends $535 in purchasing some
sending petrol from any refinery to any shirts and ties for her husband. If shirts
district? cost $43 each and the ties cost $21 each,
A. 63 B. 42 then what is the ratio of the shirts to the
C. 54 D. 378 ties, that are purchased ?
E. None of these A. 1 : 2 B. 2 : 1
C. 2 : 3 D. 3 : 4
DIRECTIONS (Qs. 26-30): In the following E. None of these
questions, two equations I and II are given. You 33. Anish spends 25% of his salary on house
have to solve both the equations and give answer. rent, 5% on food, 15% on travel, 10% on
clothes and the remaining amount of
Give answer (A) if x > y
$22,500 is saved. What is Anish's salary ?
Give answer (B) if x ≥ y
A. $40,000 B. $40,500
Give answer (C) if x < y
C. $45,500 D. $50,000
Give answer (D) if x ≤ y
E. None of these
Give answer (E) if x = y or the relationship 2
34. th of Anil's salary is equal to Bhuvan's
cannot be established 5

salary and sevenninth of Bhuvan's salary is


26. I 289 + 25 = 0 equal to Chandra's salary. If the sum of the
II 676𝑦 + 10 = 0 salary of all of them is $77,000, then, how
27. I 8𝑥 2 – 78 𝑥 + 169 = 0 muchis Bhuvan's salary?
II 20𝑦 2 - 117 𝑦 + 169 = 0 A. $45,000 B. $18,000

139
C. $15,000 D. $28,000 What will be the height of the solution level
E. None of these in the flask ?
35. A tap can fill an empty tank in 12 hours and 𝑎 2ℎ 3ℎ𝑝 2
A. 3𝑝 2
𝑐𝑚 B. 𝑎2
𝑐𝑚
a leakage can empty the whole tank in 20
𝑝2 3𝑎 2
C. 𝑐𝑚 D. 𝑐𝑚
hours. If the tap and the leakage are 3ℎ 2 ℎ𝑝 2

working simultaneously, how long will it E. None of these


take to fill the whole tank? 40. A sum was put at simple interest at a
A. 25 hours B. 40 hours certain rate for 2 years. Had it been put at
C. 30 hours D. 35 hours 3% higher rate, it would have fetched $300
E. None of these more. Find the sum.
36. A train is moving at a speed of 132 km/h. If A. $6000 B. $8230
the length of the train is 110 metres, how C. $5000 D. $4600
long will it take to cross a railway platform, E. None of these
165 metres long ?
A. 5 s B. 7.5 s REASONING
C. 10 s D. 15 s
41. Which is the third number to the left of the
E. None of these
number which is exactly in the middle of
37. If 15 women or 10 men can complete a
the following sequence of numbers?
project in 55 days, in how many days will 5
123456789246897539876432
women and 4 men working together
1
complete the same project ?
A. 3 B. 2
A. 75 B. 8
C. 5 D. 6
C. 9 D. 85
E. None of these
E. None of these
42. In a certain code IDEAS is written as
38. Ashu’s mother was three times as old as
HEDBR and WOULD is written as VPTMC.
Ashu, 5 years ago. After 5 years, she will be
How will RIGHT be written in the same
twice as old as Ashu. How old is Ashu at
code ?
present?
A. QJHIS B. QJFGS
A. 15 B. 20
C. SHHGU D. QJFIU
C. 10 D. 5
E. QJFIS
E. None of these
43. If the alphabet is written in the reverse
39. A conical flask has base radius ‘a’ cm and
order and every alternate letter starting
height ‘h’ cm. It is completely filled with
with Y is dropped, which letter will be
milk. The milk is poured into a cylindrical
exactly in the middle of the remaining
thermos flask whose base radius is ‘p’ cm.
letters of the alphabet.

140
A. M B. N month was Tuesday, then on which date
C. O D. M or O did Rama meet her brother ?
E. None of these A. 24th B. 23rd
44. In a row of girls, Rita and Monika occupy C. 25th D. 26th
the ninth place from the right end and E. None of these
tenth place from the left end, respectively. 48. If it is possible to make only one such
If they interchange their places, then Rita number with the first, the fourth and the
and Monika occupy seventeenth place from sixth digits of the number 531697 which is
the right and eighteenth place form the left the perfect square of a two digit even
respectively. How many girls are there in number, which of the following will be the
the row? second digit of the two digit even number.
A. 25 If no such number can be made, give '@' as
B. 26 the answer and if more than one such
C. 27 number can be made, give '©' as the
D. Data inadequate answer.
E. None of these A. 4 B. 2
45. In a certain code language 'Ka Bi Pu Ya' C. 6 D. @
means 'You are very intelligent' 'Ya Lo Ka E. ©
Wo' means 'They seem very intelligent' 'La 49. In a certain code JOURNEY is written as
Pu Le' means 'You can see' and 'Sun Pun TNISZFO. How is MEDICAL written in that
Yun Ya' means 'how intelligent she is', In code?
that language, which of the following words A. CDLJMBD B. CDWDBM
means 'are'? C. LDCJMBD D. EFNJMBD
A. Ka B. Bi E. None of these
C. Ya D. Pu 50. If 'K' denotes '×', 'B' denotes '÷', 'T' denotes
E. None of these '–' and 'M' denotes '+', then –
46. Ankit is related to Binny and Chinky, Daizy 40 8 T 6 M 3 K 4 = ?
is Chinky’s mother. Also Daizy is Binny’s A. 19 B. 11
sister and Aruna is Binny’s sister. How is C. –31 D. 23
Chinky related to Aruna? E. None of these
A. Niece B. Sister
C. Cousin D. Aunt
E. None of these
47. Rama remembers that she met her brother
on Saturday, which was after the 20th day
of a particular month. If the 1st day of that

141
DIRECTIONS ( Qs. 51-55): Study the following known facts and then decide which of the
information to answer the given questios answers (A), (B), (C), (D) and (E) is the correct
answer and indicate it on the answer sheet.
In a certain code 'support the other group' is
written as 'ja pe la no' 'the mission gains 56. Statements:
support' is written as' ke ja zi la', 'gains other Some chairs are tables.
than money' is written as 'fu no ho zi' and 'more Some tables are drawers.
support and money' is written as ' re qi fuja'. All drawers are shelves.
51. What is the code for 'group' ? Conclusions:
A. ja B. pe I. Some shelves are tables.
C. la D. no II. II. Some drawers are chairs
E. Cannot be determined III. III.Some shelves are drawers.
52. What does 'zi' stand for ? A. Only I and III follow
A. mission B. than B. Only I and either II or III follow
C. other D. the C. Only II and either I or III follow
E. gains D. All I, II and III follow
53. Which of the following may represent E. None of the above
'more than the group'? 57. Statements:
A. la qi ho pe B. re la qi ho All trees are flowers.
C. re no la pe D. pe ke qi la Some flowers are leaves.
E. qi ho la fu No leaf is bud
54. What is the code for 'mission'? Conclusions:
A. la B. zi I. No bud is a flower.
C. ke D. ja II. Some buds are flowers.
E. ke or la III. Some leaves are trees.
55. Which of the following may represent A. Only II and III follow
'money matters more'? B. Only III follows
A. fu bu B. re bu qi C. Only either I or II follows
C. zi qi yo D. yo fu no D. Either I or II and III follow
E. la fu bu E. None of the above
58. Statements:
DIRECTIONS (Qs. 56-60): In each question All stones are rocks.
below are three statements followed by three Some rocks are bricks.
conclusions numbered I, II and III. You have to Some bricks are cement.
take the three given statements to be true even if Conclusions:
they seem to be at variance from commonly I. Some cements are rocks.

142
II. Some bricks are stone DIRECTIONS (Qs. 61-65): Study the following
III. Some stones are cement. information and answer the questions given
A. Only I and either II or III follow below it.
B. Only either II or III follows
Seven people—A, B, C, D, E, F and G are sitting
C. Only I and II follow
in a circle. Five of them are facing the centre
D. All follow
while two of them are facing opposite to the
E. None of the above
centre. C sits third to the left of D and both are
59. Statements:
facing the centre. E is neither on immediate
All flats are buildings.
neighbour of D nor of C. The one sitting exactly
All buildings are bungalows.
between D and F is facing opposite to centre. G
All bungalows are apartments.
sits third to the right of A and G is facing the
Conclusions:
centre. One of B’s neighbour is facing opposite
I. Some apartments are flats.
to the centre.
II. All flats are bungalows.
III. Some bungalows are flats. 61. Which of the following pairs represents
A. None follows persons facing opposite to the centre?
B. Only I and II follow A. A and F
C. Only II and III follow B. E and F
D. Only I and III follow C. A and E
E. All I, II and III follow D. Cannot be determined
60. Statements: E. None of these
Some spectacles are lenses. 62. Who is sitting second to the left of A?
Some lenses are frames. A. C B. G
All frames are metals. C. E D. B
Conclusions: E. None of these
I. Some lenses are metals 63. Who is sitting to the immediate left of E?
II. Some metals are spectacles. A. C B. G
III. Some frames are spectacles. C. B D. A
A. Only III follows E. None of these
B. Only I follows 64. What is the position of F with respect to B?
C. Only I and either II or III follow A. Fourth to the left
D. Only I and II follow B. Second to the right
E. None of the above C. Third to the right
D. Second to the left
E. None of these

143
65. If all the persons are asked to sit in a 68. Statements : P$Q; N#M; M@R; R*P
clockwise direction in an alphabetical Conclusions : I. P + N
order starting from A, the position of how II. Q$M
many will remain unchanged, excluding A? 69. Statements : D + T; E $V; F *T; E@D
A. Three B. One Conclusions : I. D $ V
C. Two D. None II. D + F
E. Four 70. Statements : T*U; U$W; V @L; W + V
Conclusions : I. V @ T
DIRECTIONS (Qs. 66-70): In the questions II. L #W
given below, certain symbols are used with the
following meanings: DIRECTIONS (Qs. 71-75): In each question
below, is given a group of letters followed by
 A @ B means A is greater than B.
found combinations of digits/symbols numbered
 A * B means A is either greater than or
(a), (b), (c) and (d). You have to find out which of
equal to B.
the four combinations correctly represents
 A # B means A is equal to B.
combination as your answer. If none of the
 A $ B means A is either smaller than or
combinations correctly represents the group of
equal to B.
letters, mark (e) ‘None of these’, as your answer.
 A + B means A is smaller than B.
Letter R E A U M D F P Q I O H N W Z B
Now in each of the following questions, Digit 7 # $ 6 % 8 5 * 4 9 @ © 3 D 1 2
assuming the given statements to be true, find code

which of the two conclusions I and II given


below them is/are definitely true? i. If the first letter is a consonant and the third
A. If only conclusion I is true letter is a vowel, their codes are to be

B. If only conclusion II is true interchanged.

C. If either conclusion I or II is true ii. If the first letter is a vowel and the fourth

D. If neither conclusion I nor II is true letter is a consonant, both are to be coded as


E. If both conclusions I and II are true the code for the vowel.
iii. If the second and the third letters are

66. Statements : B + D; E$T; T * P; P@B consonants, both are to be coded as the code

Conclusions : I. P$D for the third letter.

II. P@D
67. Statements : E*F; G$H; H#E; G@K 71. NABAQE

Conclusions : I. H @K A. 263$4# B. 326$4#

II. H*F C. 362$4# D. 362$3#


E. None of these

144
72. FWZERA E. None of these
A. 5D #7$ B. 5DD #7$ 77. Which of the following statements is not
C. D17#$ D. 511#7$ ture in the context of the above sitting
E. None of these arrangement?
73. HUBDIN A. There are seen students sitting
A. © 62893 B. © 2689% between K and D
C. © 6289 © D. © 62 © 9% B. G is the immediate neighbour of I and C
E. None of these C. H is the immediate neighbour of D and
74. EMIRDP F
A. #%978* B. #%9#8* D. K is between E and A
C. 7%9#8* D. #9%78* E. F is third to the right of C
E. None of these 78. To obtain the respective seats of all the
75. OREDHM persons which statement given above is
A. @7#8© % B. #7#8© % not required?
C. @78# ©% D. @7#@© % A. I B. II
E. None of these C. III D. IV
E. None of these
DIRECTIONS (Qs. 76-80): Study the following 79. Besides ‘E’, who among the following is at
information carefully and answer the given the extreme end?
quesions following it. A. K B. F
C. B D. Can’t say
i. Eleven students A, B, C, D, E, F, G, H, I, J and K
E. None of these
are sitting in the first row of a class facing
80. Which of the following gropus is at the left
the teacher.
of I?
ii. D, who is on the immediate left of F, is
A. AJB B. GCH
second to the right of C.
C. HDF D. GCH
iii. A is second to the right of E, who is at one of
E. None of these
the ends.
iv. J is the immediate neighbour of A and B and
third to the left of G.
v. H is on the immediate left of D and third to
the right of I.

76. Who is sitting midway between E and H?


A. J B. B
C. I D. G

145
1
PRACTICE SET 1 5.
9− 8
=
1
A. (3-2 2)
QUANTITATIVE APTITUDE 2
1
B.
3+2 2
DIRECTIONS (Qs. 1-5): What will come in
C. 3 − 2 2)
place of questionmark (?) in the following
questions? D. 3+2 2
E. None of these
1. [5 7 + 7) + (4 7 + 8 7)] − (19)2 = . . .
. DIRECTIONS (Qs. 6-10) : In each of the
A. 143 following questions anumber series is
B. 134 given. A number in the series is expressed
C. 72 7 by letter ‘N’. You have to find out the
D. 70 7 number in the place of ‘N’ and use the
E. None of these number to find out the value in the place
2. (4444 ÷ 40) + (645 ÷ 25) + (3991÷ 26) = . of the question mark in the equation
.. following the series.
A. 280.4
B. 290.4 6. 68 68.5 69.5 71 N 75.5 78.5
C. 295.4 N × 121 + …. = 10000
D. 285.4 A. 1160
E. None of these B. 1200
3. 33124 𝑥 2 601 − (83)2= (?)2 + (37)2 C. 1150
A. 37 D. 1180
B. 33 E. None of these
C. 34 7. 19 20 24 33 49 74 N 159
D. 28 N2 ¸ 10000 = ….
E. None of these A. 121.0
B. 12.1

4. 5
17 51 1 3
+ 4 52 + 11 7 + 2 4 = ? C. 1.21
37
D. 0.121
A. 303.75
E. None of these
B. 305.75
3 8. 51 43 N 30 25 21 18
C. 3034
N2 – 2N = ….
31
D. 303 4 A. 1155
E. None of these B. 1224
C. 1295

146
D. 1368 15. I. x2 + 4x + 4 = 0
E. None of these II. y2– 8y + 16 = 0
9. 2 5 14 41 122 365 N
2
N - 16 % of 5670 – (?)2 = 102 DIRECTION (Qs. 16-20) : Study the
3
following graph carefully to answer the
A. 7
question given below it.
B. 149
C. 49
Production of paper (in lakh tonnes) by 3
D. 7
different companies A, B & C over the
E. None of these
years
10. 510 254 N 62 30 14 6
40% N + ? = 92
A. 31.4
B. 29.8
C. 50.4
D. 30.6
E. None of these

DIRECTIONS (Qs. 11-15) :In each of the


following questionstwo equations are
16. What is the difference between the
given. Solve these equations and give
production of companyC in 2010 and the
answer:
production of Company A in 2015?
A. if x ³ y, i.e., x is greater than or equal to y.
A. 50,000 tonnes
B. if x > y, i.e., x is greater than y.
B. 5,00,00,000 tonnes
C. if x £ y, i.e., x is less than or equal to y.
C. 50,00,000 tonnes
D. if x < y, i.e., x is less than y.
D. 5,00,000 tonnes
E. x = y or no relation can be established
E. None of these
between x and y
17. What is the percentage increase in
11. I. x2+ 5x + 6 = 0
production of CompanyA from 2011 to
II. y2 + 7y + 12 = 0
2012?
12. I. x2 + 20 = 9x
A. 37.5
II. y2+ 42 = 13y
B. 38.25
13. I. 2x + 3y = 14
C. 35
i. 4x + 2y = 16
D. 36
14. I. x = 625 E. None of these
II. x = 676

147
18. For which of the following years the (in prime time) among different TV
percentage of rise/fallin production from channels.
the previous year the maximum
forCompany B?
A. 2011
B. 2012
C. 2013
D. 2014
E. 2015
Percentage of urban TV viewers of these
19. The total production of Company C in
channels (Total number of TV viewers is
2012 and 2013 iswhat percentage of the
12 lakh)
total production of Company A in2010
and 2011?
A. 95
B. 90
C. 110
D. 115
E. None of these
21. What is the total number of TV viewers
20. What is the difference between the
from urban areasfor all the channels?
average production peryear of the
A. 721780
company with highest average
B. 786486
production andthat of the company with
C. 811920
lowest average production in lakhtonnes?
D. 824864
A. 3.17
E. None of these
B. 4.33
22. What is the average number of TV
C. 4.17
viewers from rural areasfor all the
D. 3.33
channels?
E. None of these
A. 62178
DIRECTIONS (Qs. 21-25) : Study the given
B. 64680
pie-chart and tableand answer the
C. 66370
following questions.
D. 68484
E. None of these
Percentage distribution of total TV
23. Total number of rural viewers of Sony TV
viewers
is what percentageof the total number of
urban viewers of the same channel?

148
A. 51.68% 27. 40.005% of 439.998 + ? % of 600.020 =
B. 52278% ….
C. 53.846% A. 8
D. 54.272% B. 17
E. None of these C. 12
24. Total number of Sahara TV viewers from D. 20
urban areas iswhat percentage more than E. 5
the total number of rural viewersof SAB 28. 6894.986 + 5025.005 + 600.020 = ….
TV? A. 12170
A. 61.24% B. 13540
B. 62.83% C. 12950
C. 63.58% D. 11560
D. 64.57% E. 12520
E. None of these 29. 31.999 × 12.001 × 17.5001 = ….
25. What is the ratio of the total number of A. 6600
rural viewers of ZeeTV to the total B. 6720
number of urban viewers of Star TV? C. 6480
A. 7:23 ( D. 6070
B. 8:21 E. 6270
C. 9:25 30. (10.998)3 = ….
D. 11:32 A. 1440
E. None of these B. 1730
C. 1330
DIRECTIONS (Qs. 26-30): What D. 1640
approximate value should comein place of E. 1000
the question mark (?) in the following 31. Average age of 36 children of the class is
questions? 15 years. 12 morechildren joined whose
average age is 16 years. What is
(Note :You are not expected to calculate the theaverage age of all the 48 children
exact value) together ?
26. 12959.998 + 18.010 = …. A. 15.25 years
A. 840 B. 15.5 years
B. 990 C. 15.3 years
C. 570 D. 15.4 years
D. 680 E. None of these
E. 720

149
32. Profit earned by selling an article of A. 35%
$1,450 is same as theloss incurred by B. 32%
selling the article for $1,280. What is C. 17.5%
thecost price of the article? D. 22.5%
A. $1,385 E. None of these
B. $1,405 37. Two cars start together in the same
C. $1,355 direction from the sameplace. The first
D. $ 1,365 goes with a uniform speed of 10 km/h.
E. None of these Thesecond goes at a speed of 8 km/h in
33. If the compound interest on a certain sum the first hour andincreases its speed
1
of money for 3 years at 10% p.a. be $ 993, by2km with each succeeding hour.After
what would be the simpleinterest ? how many hours will the second car
A. $800
overtake the firstone, if both go non-stop?
B. $950
A. 9 hours
C. $900
B. 5 hours
D. $1000
C. 7 hours
E. None of these D. 8 hours
34. In a class, 20 opted for Physics, 17 for
E. None of these
Maths, 5 for both and10 for other
38. 24 men working 8 hours a day can finish
subjects. The class contains how
a work in 10 days.Working at the rate of
manystudents?
10 hours a day, the number of
A. 35
menrequired to finish the same work in 6
B. 42
days is
C. 52
A. 30
D. 60
B. 32
E. None of these
C. 34
35. How much water must be added to 100 cc D. 36
of 80% solutionof boric acid to reduce it
E. None of these
to a 50% solution
39. The sum of digits of a two digit number is
A. 20 cc
15. If 9 be addedto the number, then the
B. 40 cc
digits are reversed. The number is
C. 80 cc A. 96
D. 60 cc
B. 87
E. None of these
C. 78
36. Successive discounts of 20% and 15% are
D. 69
equivalent to asingle discount of
E. None of these

150
40. Three cubes of a metal are of edges 3 cm, C. 4998
4 cm and 5 cm.These are melted together D. 8949
and from the melted material,another E. 8994
cube is formed. The edge of this cube is : 44. Saroj is mother-in-law of Vani who is
A. 8 cm sister-in-law of Deepak.Rajesh is father of
B. 10 cm Ramesh, the only brother of Deepak.How
C. 9 cm is Saroj related to Deepak?
D. 6 cm A. Mother-in-law
E. None of these B. Wife
C. Aunt
REASONING ABILITY D. Mother
41. If the following series is written in the E. None of these
reverse order, whichnumber will be 45. A directional post is erected on a
fourth to the right of the seventh crossing. In an accident, it was turned in
numberfrom the left ? such a way that the arrow which was
7, 3, 9, 7, 0, 3, 8, 4, 6, 2, 1, 0, 5, 11, 13 firstshowing east is now showing south. A
A. 0 passerby went in awrong direction
B. 5 thinking it is west. In which direction is
C. 9 heactually travelling now ?
D. 11 A. North
E. None of these B. South
42. In a certain code language ‘ne ri so’ C. East
means ‘good rainyday’, ‘si ne po’ means D. West
‘day is wonderful’ and ‘rijo’ means‘good E. None of these
boy’. Which of the following means ‘rainy’
in thecode? DIRECTIONS (Qs. 46-50) : In each question
A. ne below, there are three statements
B. si followed by two conclusions numbered I
C. ri and II. You have to take the three given
D. so statements to be true even if they seem to
E. None of the above be at variance from commonly known
43. If SMOOTH is coded as 135579, ROUGH facts and then decide which of the given
as 97531 andHARD as 9498, then SOFT conclusions logically follows from the
will be coded as three statements disregarding commonly
A. 1527 known facts.
B. 1347

151
Give answer (A) if only conclusion I follows. All lions are goats.
Give answer (B) if only conclusion II follows. Conclusions:
Give answer (C) if either I or II follows. I. Some goats are roofs.
Give answer (D) if neither I nor II follows. II. All goats are figures
Give answer (E) if both I and II follow
46. Statements: DIRECTIONS (Qs. 51-55) : Study the
All shoes are pens. following information to answer the given
Some pens are razors. questions.
Some razors are desks. P, Q, R, S, T, V, X and Y are seated in a staright
Conclusions : line facing North, P sits fourth to the left of V.
I. Some desks are shoes. V sits either sixth from the left end of the line
II. Some razors are shoes or fourth from the right end of the line. S sits
47. Statements: second toright of R. R is not an immediate
Some benches are windows. neighbour of V. T and Q are immediate
Some windows are walls. neighbours of each other but neither T nor Q
Some walls are trains. sits at extreme ends of the line. Only one
Conclusions: person sits between T and X. X does not sit at
I. "Some trains are benches." the extreme end of the line.
II. No train is bench.
48. Statements : 51. What is the position of Q with respect to
All brushes are chocolates. P?
All chocolates are mirrors. A. Fifth to the right
All mirrors are tables. B. Immediate neighbor
Conclusions: C. Second to right
I. Some tables are brushes D. Third to left
II. Some mirrors are chocolates. E. None of the above
49. Statements : 52. Which of the following represents
Some pencils are knives. persons seated at the two extreme ends
All knives are papers. of the line?
Some papers are books. A. P, V
Conclusions: B. Y, S
I. Some books are pencils. C. R, V
II. Some papers are pencils. D. Y, P
50. Statements: E. R, Y
Some roofs are figures. 53. How many persons are seated between R
All figures are lions. and T ?

152
A. One
B. Two 56. How many players are neither artists nor
C. Three doctors?
D. Four A. 25
E. None of these B. 22
54. If P is related to Q and S is related to T in C. 4
a certain way, towhich of the following D. 29
would V be related to following thesame E. None of these
pattern ? 57. How many artists are players?
A. Y A. 22
B. P B. 3
C. R C. 25
D. S D. 8
E. X E. None of these
55. Who amongst the following sits exactly in 58. How many artists are neither doctors nor
the middle of thepersons who sit second players?
from the left and the person whosits fifth A. 22
from the right? B. 8
A. V C. 25
B. Q D. 30
C. T E. None of these
D. S 59. How many doctors are neither players
E. P nor artists?
DIRECTIONS (Qs. 56-60) : Each of the A. 4
following questions isbased on the diagram B. 25
given below. Study the diagram carefully and C. 17
answer the questions. D. 20
E. None of these
60. How many doctors are players and artists
both?
A. 4
B. 7
C. 3
In the above diagram, rectangle represents D. 8
‘artists’, circle represents ‘players’ and E. None of these
triangle represents ‘doctors’.

153
DIRECTIONS (Qs. 61-65) : In these E. More than three
questions, relationshipbetwen different 67. If the order of the first twenty
elements is shown in the statements. letters/numbrs in the abovesequence is
These statements are followed by two reversed and the remaining
conclusions. letters/numbersare kept unchanged,
Given answer which of the following will be
(A) if only Conclusion I follows thefourteenth letter/number from the
(B) if only Conclusion II follows right end after therearrangement?
(C) if either Conclusion I or II follows A. B
(D) if neither Conclusion I nor II follows B. 6
(E) if both Conclusion I and II follows C. 2
D. 1
61. Statements L > M, M > N, N > P E. None of these
ConclusionsI. L > P II. M > P 68. Which of the follwing letter/number is
62. Statements A > B, B B = H, H > G the eighth to the leftof the nineteenth
Conclusions I. A > G II. A > H letter/number from the left end?
63. Statements H < J, F < H, I ≤ J = K A. N
ConclusionsI. H > I II. I ≥F B. T
64. Statements A > B >C ≤ D = E C. 1
ConclusionsI. B ≤ E II. B < P D. D
65. Statements P > M > Q, Q > Z > N E. None of these
Conclusions I. M ≥ Z II. N < P 69. Four of the following five are alike in a
certain way withregard to their position
DIRECTIONS (Qs. 66-70): Study the following in the above sequence and so forma
sequence carefully and answer the questions group. Which is the one that does not
given below: belong to thatgroup?
M E 5 P B 2 A 7 K N 9 T R U 4 6 I J D F 1Q 3 A. WIQ
W8VISZ B. PAE
66. How many such numbers are there in the C. NR7
abovesequence, each of which is both D. 4JR
immediately preceded byand E. D16
immediately followed by a consonant ? 70. How many such vowels are there in the
A. None above sequence,each of which is
B. One immediately preceded by a consonant
C. Two andimmediately followed by a vowel?
D. Three A. None

154
B. One
C. Two
D. Three
E. More than three

155
GMST
Latihan Quantitative
Questions 1-15: Each question shows a quantity in Column A and a quantity in Column B.
Compare the two quantities and indicate whether:
A. The quantity in Column A is greater.
B. The quantity in Column B is greater.
C. The two quantities are equal.
D. The quantities cannot be compared from the information provided.

1. Ann's class has x students this semester. Paul’s class has one less than three times the number of
students in Ann's class.
Column A Column B
𝑥2 2𝑥

A. The quantity in Column A is greater.


B. The quantity in Column B is greater.
C. The two quantities are equal.
D. The quantities cannot be compared from the information provided.

2.
17 Column A Column B
8
𝑏2 289

A. The quantity in Column A is greater.


B. The quantity in Column B is greater.
C. The two quantities are equal.
D. The quantities cannot be compared from the information provided.

3. Isosceles triangle ∆ DEF has a perimeter of 24 and side DE has a length of 12.
Column A Column B
Side EF Side DF

156
A. The quantity in Column A is greater.
B. The quantity in Column B is greater.
C. The two quantities are equal.
D. The quantities cannot be compared from the information provided.

Questions 4 and 5 both refer to the figure below.

B
C G D

E H F

4.
Column A Column B
<CGB <DGH

A. The quantity in Column A is greater.


B. The quantity in Column B is greater.
C. The two quantities are equal.
D. The quantities cannot be compared from the information provided.

5.
Column A Column B
<CGB+<CGH <BGD+<DGH

A. The quantity in Column A is greater.


B. The quantity in Column B is greater.
C. The two quantities are equal.
D. The quantities cannot be compared from the information provided.

6. Recorded attendance at weekly seminars in the first quarter of the year has been: 117, 103, 109,
100, 125, 115, 120, 126, 130, 106, 109, 125

Column A Column B
The median The average

157
A. The quantity in Column A is greater.
B. The quantity in Column B is greater.
C. The two quantities are equal.
D. The quantities cannot be compared from the information provided.

7. 6 – 4𝑥 > 3

Column A Column B
𝑥 1

A. The quantity in Column A is greater.


B. The quantity in Column B is greater.
C. The two quantities are equal.
D. The quantities cannot be compared from the information provided.

8.

Column A Column B
½ times the perimeter of a triangle The perimeter of a squre with a side of 3 cm; 8
cm; 14 cm; 3.2 cm.

A. The quantity in Column A is greater.


B. The quantity in Column B is greater.
C. The two quantities are equal.
D. The quantities cannot be compared from the information provided.

9.
Column A Column B
The number of new three digit 6
numbers that can be formed
by using the digits 413 once

A. The quantity in Column A is greater.


B. The quantity in Column B is greater.
C. The two quantities are equal.
D. The quantities cannot be compared from the information provided.

158
10.
Column A Column B
The probability of throwing 6
an even number on a fair, six- 3
sided die

A. The quantity in Column A is greater.


B. The quantity in Column B is greater.
C. The two quantities are equal.
D. The quantities cannot be compared from the information provided.

11.
Column A Column B
The least common multiple of The least common multiple of
267 and 1782 252 and 1330

A. The quantity in Column A is greater.


B. The quantity in Column B is greater.
C. The two quantities are equal.
D. The quantities cannot be compared from the information provided.

12. Consider these two expressions:


4𝑥 − 3𝑦 = 8 − 2𝑥 + 5𝑦 = 10

Column A Column B
The value of 𝑥 + 𝑦 9

A. The quantity in Column A is greater.


B. The quantity in Column B is greater.
C. The two quantities are equal.
D. The quantities cannot be compared from the information provided.

13.
Column A Column B
The length of a rectangle with The perimeter of a square
a perimeter of 40.8 cm with a side of 3.5 cm

A. The quantity in Column A is greater.


B. The quantity in Column B is greater.

159
C. The two quantities are equal.
D. The quantities cannot be compared from the information provided.

B
14.

A is the center of circle; BD = 17 cm


C
A

Column A Column B
AD BC

A. The quantity in Column A is greater.


B. The quantity in Column B is greater.
C. The two quantities are equal.
D. The quantities cannot be compared from the information provided.
15.

A. The quantity in Column A is greater.


B. The quantity in Column B is greater.
C. The two quantities are equal
D. The quantities cannot be compared from thye information provided.

Question 16-23 : Choose the letter that best answers the question.

16. What is the su, of the integers fro, 13 to 75, inclusive?


A. 2184

160
B. 2279
C. 2332
D. 2376
E. 2385
17. The ratio of empty to full parking spaces is 3 to 5. If there are 940 full parking spaces in the lot, how
many spaces are empty?
A. 156
B. 376
C. 470
D. 564
E. 602
1
18. Charless is able to husk 216 ears of corn in 22 hours. What was his rate per minute?

A. 1.44
B. 5.4
C. 14.4
D. 54
E. 144
19.

The perimeter of triamgle ABC is 84


The perimeter of isosceles triangle BCD is 110
What is the length of CD?
A. 30
B. 35
C. 40
D. 42
E. It can’t be determined from the information provided.

161
20. Which two of the following fractions are 25. (x – 2)(x + 4) – (x – 3)(x – 1) = 0
equal? A. –5
0.040
I. B. –1
0.108
0.40 C. 0
II. 0.180
D. ½
0.40
III. 0.018 E. 11/6
0.004
IV. 26. −24–(x2− 1)2=
0.0018
A. − x4 + 2x2 + 15

A. I and II B. − x4 − 2x2 + 17

B. II and III C. − x4 + 2x2– 17

C. I and III D. − x4 + 2x2 − 15

D. II and IV E. − x4 + 2x2 + 17

E. I and IV 27. The smallest prime number greater than 48is

21. If 3x + 9 = 15, then x + 2 = A. 49

A. 2 B. 50

B. 3 C. 51

C. 4 D. 52

D. 5 E. 53

E. 6 28. If a, b, and c are consecutive integers anda < b <


𝐴2 c, which of the following must betrue?
22. If a =3b, b2=2c, 9c=d, then
𝐷
A. b2 is a prime number
A. 1/2 𝑎+𝑐
B. 2
=b
B. 2
C. 𝑎 + 𝑏 is even
C. 10/3
𝑎𝑏
D. 5 D. 3
is an integer

E. 6 E. c - a =b
23. a + b + c/2 = 60 29. (42 − 6)(20 + 16)=
-a –b + c/2 = -10 A. 2
Column A Column B B. 20
b c C. 28
24. 3 – ( 23 – 2[3 – 16 ÷ 2]) = D. 30
A. –15 E. 36
B. –5 30. (4x)2 –
C. 1 A. 24x
D. 2 B. 4x+2
E. 30 C. 22x+2

162
D. 4x2 B. 0 < x < 2
E. 22x2 C. –2 < x < 2
31. If 813=2z, then z = D. x < 2
A. 10 E. x > 2
B. 13 37. If x is an integer and y = –3x + 7, what is
C. 19 theleast value of x for which y is less than 1?
D. 26 A. 1
E. 39 B. 2
32. ½ of 0.2 percent equals C. 3
A. 1 D. 4
B. 0.1 E. 5
C. 0.01
D. 0.001
E. 0.0001
4
33. 1
+1
3

A. 1 Note, Figure not drawm to scale


B. ½ 38. In the figure above, triangle ABC is
C. 2 isosceleswith base AC. If x = 60˚, then AC =
D. 3 A. 2
E. 4 B. 3
34. If x + y = k, then 3x2 + 6xy + 3y2 = C. -4
A. K D. 14/3
B. 3k E. 30
C. 6k 39. A unit square is circumscribed about acircle. If
D. k2 the circumference of the circle isqπ, what is the
E. 3k2 value of q?
35. 8x2 – 18 = A. 1
A. 8(x2 – 2) B. 2
B. 2(2x + 3)(2x – 3) C. Π
C. 2(4x + 3)(4x – 3) D. 2π
D. 2(2x + 9)(2x – 9) E. 5π
E. 2(4x + 3)(x – 3)
36. For which values of x is the
followinginequality true: x2< 2x.
A. x < 0

163
40. What is the area of the triangle above? 46. How many ounces of water must be added
A. 20 toa 30-ounce solution that is 40
B. 24 percentalcohol to dilute the solution to 25
C. 30 percentalcohol?
D. 54 A. 9
E. 64 B. 10
41. If the average of 2x and 4x is 12, then x = C. 15
A. 1 D. 16
B. 2 E. 18
C. 3 47. What is the value of the 201 st term of
D. 4 asequence if the first term of the sequence is2
E. 24 and each successive term is 4 more thanthe
42. The average of x, y, and z is 8 and the average term immediately preceding it?
of y and z is 4. What is the value of x? A. 798
A. 4 B. 800
B. 9 C. 802
C. 16 D. 804
D. 20 E. 806
E. 24 48. A particular carmaker sells four models ofcars,
43. If the ratio of two numbers is 6 and theirsum and each model comes with 5 options.How
is 21, what is the value of the largernumber? many different types of cars does thecarmaker
A. 1 sell?
B. 5 A. 15
C. 12 B. 16
D. 17 C. 17
E. 18 D. 18
44. What percent of 3x is 6y if x = 4y? E. 20
A. 50% 49. Define a@b to be a3-1.
B. 40% Column A Column B
C. 30% X@1 X @10
D. 20% 50. Define the symbol * by the following equation:
E. 18% x* = 1 – x, for all non-negative x. If ((1− x)*) * =
45. y = 3x and x > 2 (1− x )*, then x =
Column A Column B A. ½ B. ¾
10% of y 40% of x C. 1 D. 2
E. 3

164
165

You might also like